CORNELL UNIVERSITY LIBRARY BOUGHT WITH THE INCOME OF THE SAGE ENDOWMENT FUND GIVEN IN 1 89 1 BY HENRY WILLIAMS SAGE Date Due JUN 1 950 TH^ ^wr°z $s& a .t 1?g$-MP~ 1^72 F iiUV 1 DEC. 1 4 497^ ■titi^r&mf Cornell University Library GV1451 .M64 Chess gems: Some of the ( finest examples 3 1924 029 919 663 olin a Cornell University J Library The original of this book is in the Cornell University Library. There are no known copyright restrictions in the United States on the use of the text. http://archive.org/details/cu31 92402991 9663 BY THE REV. HORATIO BOLTON. White to play and mate in ten moves. IYANHOE. BY WILLIAM BONE. White engages to mate with his King's Bishop's Pawn (f5) in twenty moves, without taking the Black Eook, or compelling it to move. CHESS GEMS: SOME OF THE FINEST EXAMPLES OF CHESS STRATEGY, BY ANCIENT AND MODERN MASTERS. COLLECTED AND SET BY JOHN AUGUSTUS MILES, FAKENHAM, NORFOLK. WITH THE SOLTJTIOIsrS. " Qualis gemma, micat, fulvum quae dividit aurum, Aut collo decus, aut eapiti." Virgil's ^Eneid, X. 134. "A thing of beauty is a joy for ever." Keats. Jjukenljam: E. W. SOUTHWOOD, BOOKSELLEE; AND THE EDITOE. 1878. & Price Eighteen Shilling s . ' WK'IVI Kur, y I I : !• I, h V /\.U^\4-\ M)|0 HU) YI.IJ'.M'.-HMMU TO SIR WILLOUGHBY JONES, BARONET, OF CRANMER HALL, NORFOLK, THIS BOOK, AS A TESTIMONY OF SINCERE EESPECT, AND HIGH ESTEEM,. is trtbitaitb' BY HIS FAITHFUL AND OBLIGED SERVANT, THE EDITOE. PREFACE. Eighteen years have elapsed since I made an attempt to collect and arrange some of the finest Chess Problems extant. During that time so many beautiful compositions have emanated from authors of all countries, which are scattered among various newspapers and serials, that it appears to me very desirable to collect and arrange the choicest of these, and, at the same time, to make a better and larger selection from the works of our two great English Masters, Bone and Bolton, the decease of both of whom we have now to lament. Messrs. J. and W. T. Pierce have given us an excellent collection of Problems by English Composers only ; but this leaves ample scope for a more extended selection, such as I have endeavoured to make from the works of the old Masters, and of the composers of every nation. In doing this I have deviated from the usual course of separating the problems of each author, and arranging them according to the number of moves in each ; and have, instead, placed them together in chronological order, wherever I have found it practicable to do so : thus exhibiting the progressive change of style which may have taken place in any author's compositions, I have given in Chapter I, a small selection from the works of the old Masters of the 16th, 17th, and 18th centuries, in studying which we are struck with their simplicity, and freedom from the depth and intricacy of the problems of the present day : yet the conceptions embodied in them are very fine. Ercole Del Rio (Hercules of the River) was, indeed, well worthy of his name ; one of his problems (No. 13) exemplifies two of the greatest beaiities of strategy, and in the next one the modern style is foreshadowed in a most remarkable manner. "With the present century commences a new era in problem construction ; Mendheim in Germany, and Bone in England leading the way, closely followed by Bolton, D'Orville, Anderssen, Brede, &c, the great Masters of their day, during whose reign, so to speak, Chess strategy made great strides. With the brilliant and forced sacrifices and constant checks of the earlier composers there began to be mingled the quiet coups de repos, which so eminently characterize the stratagems of the present day. Bolton was, undoubtedly, the great pioneer in this track, and it is worthy of notice that, in his later compositions, these fine waiting moves occur more frequently than in his earlier ones. Of these great Masters many retired from the arena some years before their death, and were succeeded by another race, among whom we find (facile principes) Bayer, Healey, Grimshaw, Andrews and many others, the style of whose works is seen to be still gradually changing, hard blows giving place to quiet manoeuvres. Compare, for instance, the problems of Mendheim, the hard hitter, with those of T. Smith, who seldom strikes a blow till he gives the final coup de grace ! Although many of the authors of this epoch are still living, some of them, alas ! seem inclined to retire on the laurels they have won, and to leave the field to be filled up by yet another race of strategists, the heroes of the present day, whose name is becoming Legion, and whose motto appears to be " Difficilia quae pulchra;" for the simplicity of the olden time has given place to subtlety, depth, and difficulty. Instead of the problems in many moves, in which the old school delighted, and which, for me, will never lose their charm, we have difficult problems in three and even in two moves. As examples of extreme difficulty the following problems of recent date may be cited ; No. 433, which gained the prize for the best three-move problem in one of the tourneys just brought to a close; No. 581, No. 726, and No. 732. A fine composer and strong solver calls this last the most difficult problem in so few moves that he ever saw. Let these and others such be compared (after solving them) with any of the problems in Chapter I, and the enormous increase of difficulty in modern compositions will, at once, become apparent. In arranging the problems of William Bone I have followed the order in which he arranged them in his own collection, and which we may fairly assume to be that in which they were composed. They are selected from many hundreds, and several of them do not appear ever to have been published by him ; the fine stratagem on the frontispiece being one such. The Eev. H. Bolton's are arranged in the order in which they were published as nearly as I have been able to ascertain. The beautiful position "Camilla" which adorns, the frontispiece would have had its place near No. 160, but some years ago it was discovered to be unsound, and I had to omit it from the collection ; I believe however, that a slight alteration has corrected it, and I feel great pleasure in being able to present Camilla to my readers, sound in wind and limb, as the great huntress should be when she " scours the plain, Plies o'er th' unbending corn, and skims along the main." I have inserted so large a number of the works of Bone and Bolton because one of my chief objects was to make a fine collection of their problems, and to offer it to the world as a tribute to the memory of two dear friends. Indeed, without this inducement, I should probably have hesitated to undertake the work at all. The problems of J B of Bridport and of T. Smith must always take rank as highly finished and beautiful compositions ; and those by E. A. M. M. of Mhow in India are, in my estimation, gems of the first water. No. 245, which I have named the Koh-i-noor, was considered by good judges at the time of its publication, (about a quarter of a century ago) to be the finest problem extant. E. A. M. M. is, or was, an Englishman : I know not whether he is living or not, and I have therefore located him between the last of our native composers who has passed away and the first of the survivors. Of the problems of other composers, deceased and Jiving, I have made as large a selection as my space would allow. Of two-move mates I have been sparing, giving, with a few exceptions, only one such by any author ; and the same remark applies to suicidal problems. In the selection of the problems of living English composers I have been most kindly assisted by many of the authors themselves ; and my thanks are due to Mr. H. J. C. Andrews of London for much valuable help in sifting and scrutinizing the problems of Bone and Bolton : and to Mr. John "Watkinson of Huddersfield for kindly placing at my disposal his fine chess library and voluminous collection of problems. The notation I have used for designating the squares in the problems and in the solutions is the German one, which I consider to be especially adapted for the purpose, being more concise and clear than that still in use in English books, which I hope ere long to see superseded. This is now so generally known that an explanation is scarcely necessary, but for the information of young students I here give a diagram. Problems have been called, not inappropriately, the Poetry of Chess. We might carry our fancy even farther, and say — they are the flowers of Caissa's garden ; charming us with their variety, delighting us with their richness and delicacy of structure, and appealing to the aesthetic sense, in common with all else that is beautiful in Nature and in Art. Viewing them in this light, I may say, with Montaigne, "I have gathered a posie of other men's flowers, and nothing but the thread that binds them is mine own." a8 b8 c8 d8 e8 f8 g8 h8 a7 b7 c7 d7 e7 f7 g? h7 a6 b6 c6 d6 e6 f6 g6 h6 a5 b5 c5 d5 e5 f5 g5 h5 a4 b4 c4 d4 e4 f4 g 4 h4 a3 b3 c3 d3 e3 f3 g3 h3 a2 b2 c2 d2 e2 f2 g2 h2 al bl cl dl el fl gl hi [ Eakenham, June 1st, 1878. LIST OP SUBSCRIBERS. The Honourable and Keverend Kenelm H. Digby. Sir Willoughby Jones, Baronet. (Four Copies.) Abbott J. W., London Andrews H. J. C, London, (Three Copies) Andrews T. J., Fakenham Atkinson Rev. M. A., Fakenham Baker C. T., Holt Ballantine W. A., New York Barclay H. F., Woodford Beck Horace, Harpley Birkbeck H., Stoke Holy Cross Birkbeck W., Thorpe Bolland Rev. W. E., Bedford Bolton Mrs., Thorpe Hamlet Bone Mrs., London Boustead H., London Bridgwater Walter, Birmingham Brown Wm., Titchwell Bryning J., Blackburn Bucknall R. C, London Carr Mrs., London Chambers Thos., Colkirk Chambers T. 8., Fakenham Charlick H.. Adelaide, South Australia Claremont, — London Coates W., Cheltenham Column J. J., M.P., Norwich Conroy J. A. Londonderry Cook B. B., Hoboken Cooper A. H., London Cowles W. B., Fakenham Derby Free Library Derry T. R., Belper Dewing R. W., Fakenham Duffy P. T., London (Two Copies) Dyson Edwin, Huddersfield Fakenham District Library Finlayson W., Bridge of Allan Finlinson J. H., Huddersfield Fluck C. J., London Fornival W., Wolverhampton Gilberg C. A., New York Greenwood W., Croskills, Leeds Grimshaw W., Whitby Grummett G. V., Fakenham Gumpel C. G-., London Gurney John, Sprowston Gurney Reginald, Fakenham Gurney Someryille, Lynn Hardingham F. R., Fakenham Hamond J. B., Stockport Hoffiday Thos., Huddersfield Horsley Chas., Fakenham Huggins F., Hawstead, Bury St. Edmunds Jones Rev. H. W., Sculthorpe Kent Rev. F. W., E. Barsham Kidson H. E., Liverpool Lee Warner H. J., Walsingham Lee Warner Rev. J. , Thorpland Levien Rev. J., Bumham Mallock D. M., Clinton, Ontario Manby F.. Rudham Manning J A., London Mason J. W., London Meyer H. F. L., London Middleton E. P., Hindringham Miles J. G., Saxlingham Miller T. J., Jr., Fakenham Murray A. K., Bridge of Allan Nash Wm., St. Neots Neame Chas., London Onions G. F., Bradford Overman John, Bumham Page G. W., Fakenham Pearson J. H., Field Dalling Perkins F. V., Manchester Phayre Rev. R., Raynham Pierce Jas., Bedford, (Two Copies) Pierce W. T., Brighton Pope S. R., Colkirk Ranken Rev. C. E., Malvern Ridsdale Rev. G. J., Raynham Robinson John, Cleckheaton Roe T., Jr., Derby Sewell P. E., Norwich Shaw W., Montreal Sheringham Miss, Fakenham Sheringham W. G., Fakenham Sillett Miss A., Fakenham Simpkinson Rev. J. N., North Creake Skipworth Rev. A. B., Tetworth Rectory Slater G. J., Bolton Smith J. D., Mayor of Norwich Southwell Thos., Norwich Southwood E. W., Fakenham Steele Joseph, Croydon Studd A. E., Oxton, Exeter (Two Copies) Taylor J. 0. H., Norwich Taylor J. P., London Temple-Lynes C. J., Blakeney Thomas S. H., London Thompson F., Derby Thursby J., Burnley Townsend A., Newport, Mon. Ward W. G., Mayor of Nottingham Watkinson J., Huddersfield (Two Copies) Watson G. A., Walsingham Wayte Rev. W., London West J. C, London White Chas., Brighton CHAPTER I. TjHIIE -A-ZLvTCHEaSTT MASTERS, To THE END OF THE EIGHTEENTH C/ENTURY: Numbers 1 to 48. GALAHAD WHITE TO PLAY AND MATE WITH HIS PAWN IN NINETEEN M0VE8 r WITHOUT MOVING EITHER HIS KING OR HIS ROOK. : And there was one among us, ever moved Among us in white armour, Galahad." Tennyson. The Holy Grail. No. 1. From an Arabic Manuscript in the British Museum, upwards of 500 years old. 1 1! B mt ^W; :. w I Hi ^-* ill HI » HHi i#a ij H .1 ?:--'% m J mm vm ~~ HI fa \ . W/xwM W/////A €ri mm m m ■ ■ ■ White to play and to give checkmate with his Bishop in twelve moves. No. 2. From an Arabic Manuscript in the British Museum, upwards of 500 years old. a& mm ^m mm W/a mffifo 4Mii White engages to checkmate the Black King on the square he now occupies in fifteen moves. 1 No. 3. By Damiano. tHHP _JL^ P r*>rW :Ti M~^JStk .&uUB am Jm, MW'\u g * ■jy""- ....A w mm J White to play and checkmate with his Pawn in four moves. No. 4. By Damiano. ^P# Wm, y *JmmyJmm f I m mJ m m mm ml, mm ill White to mate in four moves checking every move, but not to mate with the Bishop. No. 5. By Damiano. J * i Z- d -9B, ■ - "mm / wm. UiO m 'mm 'mm m t 6 ^ 1 White will checkmate with his King's Pawn in five moves, without moving his King. No. 6. By Damiano. 'm C„„„„,«H fSSS i wm§^ mm i HHP PHI P JUL si V-'.. tf i mm up"" 3 White to play and mate with his King's Knight's Pawn in ten moves, without taking the adverse Pawn. No. 7. By Captain Bertin. mm. W ■IP— - . r. up % w//M, ill HI M ■ 111 i i-ft-iSl "p^i s™ "White to play and mate in eight moves. No. 8. By Stamma. BLACK. ^ i M i ....M,^r„mA m w J m ' " w, White to play and mate in three moves. No. 9. By Stamma. m . iiL ; = : iri i jBf 1 II ^ IP m mm,,,.,, WuA mJL.gm m m *: ! I ■^J54 m '.■■/. w, »s H "White to play and mate in five moves. No. 10. By Stamma. ///7/A '//////////A mJt Hi lJLhi asp ,^§Sls3^ ^HH %%mw s%§HH w, 'mm, nm ^ mrn ^^S White to play and mate in six moves. No. 11. By Stamma. n IJIJ IllJlill 11 mxwii. ~ az -' w/mM. ty/z/Jmy, tff%Z%>/, musm. -///////////,. '////sZy/A. . m "1 My//////;. ife r 1§ s up ^ vyy/////> %t White to play and mate in nine moves. No. IS. By Del Rio. 'mm mm « |fl§ %f% mm ^-////////J , .....;■■"■ m m /%. //////////: msk mwmm. mm w%m w, W.„ §» ^m w/.-^^mm wmr ^'liiteto play and mate in three moves. No. 13. By Del Rio. \ W III wmk ^ illl ,_„ , Jill llli H © H i fll «^^P * a F2p i „„.ftn 2^? ? 1 f fwTl White to play and mate in four moves. No. 14. By Del Rio. wgk mm m mm mm Jam warn m. m III M m^i,Wm ■w ^ m WM y0- wm mmt ^^«^« J White to play and mate in four moves. No. 15. By Del Eio. ; *11*I1 11........ II •PUP A, 'WW? HHIP w mm.,. PI IB w, 'mm. ttmMM ' i wrTmm 'mi iJZLMM "White to play and mate in four moves. No. 16. By Bel Rio. j -Hi /- > i iii W////Z %%., 9, m. m W wW WHITE. White to play and mate in four moves. No. 17. By Del Eio. -W-LJJ1 f • m X illl ■%%m&,, 4m!m4i„ m *^/z*.!„.TTL»l«8s*8P ^« Mil PHI ! O ^41 iSI White to play and mate in five mores. No. 18. By Del Rio. « i 1' W JlJSfJilllL ifa IP I ■. HI »l. P^ M^ IIP J»N S ilil ffffil Ilil /5a HI Hii ^§Hl tfH *£J m m,„„„„„^Pi^_^^*^^^^^^ White to play and mate in five moves. No. 19. By Del Rio. BLACK. ■ m «l mm m wm^ >y- W, !«j*Hili JL ill m t/ JMM WM 7 WM White to play and mate in six moves. No. 20. By Del Bio. PPP P ■ if 1 b f ^^^^ww^l ■'-^^P^'—M^-^^M , yfo& m wm mi mm. WHITE. White to play and mate in seven moves. No. 21. By Del Rio. BLACK. Hi§ 'jmk wm ill ill w /m w ,wm^ mm. mm,. m—j* m HH B..1..I1 11111 "^ ifyl ^« M MS White to play and mate in ten moves. No. 22. By Contarelli. i • 1 W" ^ W"'* "White to play and mate in five moves. No. 23. By Contarelli. WM W, m. f*1 Wm? w/////// '-mrz/ //Z//// ■'////«'/■■' L-Mi .wmk '^^ / - ■////////// ^^^'^....Wifo I H&JP tw up n liH SI y////y////fi /////// "'-//// m. w ^h H iHf H ^ White to play and mate in ten moves. No. 24. By Lolli. m wmi mm 'm m. mm, ^a„ .. fJH Hi ill urn mm nj mxffii ■i ■..^.■^„.bL ii^ i % JML mm B^^Hw ' I ill WB^\ \m^~m mj±mm w$, W; W>. wm Wlm, White to play and mate in five moves. No. 31. By Ponziani. mm mm mm, 4nA mm 'mm, '■mm. mmt mm — i mm k mm k *m\rTm m mm mm^/mm mm. mm. m m Wi/i mm wm^=*wm. White compels Black to mate in five moves. No. 32. By Ponziani. w mm m m m m mm. w mi. 'i %^A,,,,,,,,,,fm3,„J^A j™ a m. mm ' A m mm mm Wm 'mm Wmb wml>, mm -...mm WHITE. White to play and mate in seven moves. No. 33. By Ponziani. ^w ^m il^# fis ^jl8 ftp White to play and mate in seven moves. No. 34. By Ponziani. wf% 'WW}< mm WM wm fit ill m mm, mm?,. mm, warn WM, 'WM HH1 H w, mm. ■ '■; ^m mm, mm WM, wm_ White to play and win. No. 35. By Allgaier. |PI ;.;.! s ill pip n^^ in i "WV^W ,W/////////.„ in OV////////A « WM™ 4m z>:X, Ill §§p "White to play and mate in five moves. No. 36. By Senupah. %%? 2% pv ■>■ - » ■§ I mm. *L««J^«L«. ; ■ W?A m ' ■•; 1 m§I1 tllll White checkmates with his Pawn in five moves. No. 37. By Senupah. ■mm WL 3m^m flip '" a A ^^HTB 4M, « r §,/fi f ^M. ■I ■Lr ill 111 ■ jM| Jpn WHITE. White checkmates with a Pawn in five moves. No. 38. By Senupah. HP 1 mm^Wm w -MiM,,, m m warn ■■ W fa. il ^^l^^fc^, i a !8 MM W//M "White checkmates with his Queen's Pawn in nine moves. No. 39. By Senupah. w.„ ■*¥?■ m m m y 'w&m, Ja^A n -I it ^ w^wm. . ; r ^ w <-> wk JL™ r ™ iHl ^B ^ Jill * i. HI IBP * * m i WHITE. White checkmates with a Pawn in eleven moves. No. 40. By Trevangadacharya. y mm % %HH W0W/, zs&gg? VBX&% 1 „■ I IB iff MM 1 Jf&^^St^im 'mm warn m, m II ^^ WHITE. White to mate with his Pawn in four moves without taking the Bishop. No. 41. By Teevangadachaeya. ai.' m& i ^z^r^mM^^wmz ^ mm, 111 w v wk v mm. mm, % wr White to mate with his Pawn in five moves without taking the Bishop. No. 42. By Teevangadachaeya. iifei Wl wmi.. =-„^m. B Jfcjj i i 1 JL ill I mm,, ■mm. m mm m mm. ^ ; White will mate with his Pawn in five moves, or compel Black to mate in eight moves. No. 43. By Teevangadachaeya. BLACK. Hi '• Ww% A HI IIP jl ill ■ 1 ml im .„ nm ~ ■ ■ B y/M, IK, ^ ^SPil #81 ill ill fii « 0//////>M WHITE. White to mate with a Pawn in seven moves. No. 44. By Teevangadachaeya. 'mm m ww 6 i ''M'tiit ■■/■' #////////%>, wm ' "w ■v?\ ww mm wr&\ I « IBjIJilL ill. iff" ;;,;,: Im i» i?M Hif tiPi ^ W//////A ^H§y %^Si Bil 'WvW wWW ^fe§S§?^„ :■■/■'■/". ww. ww. wm *Wm White to mate with a Pawn in eight moves. No. 45. By Trevangadachakya W% Hk HH lisp 8 fi ■ML wn. *L~JBIA. ill fMk p JP m m r m White to play and mate in nine moves. No. 46. By Trevangadacharya. ^^ Ml HP ■1 HI ^ i ^»„ jdi HI ■ ^ ^p «p «p jiji #1. ■ "V *■■ ISli ^, W//////ML '** ,r; ,^^i v/y/Mwfr mh^ r : " WHITE. White to mate with his Pawn in eleven moves. No. 47. By Trevangadacharya. : ^ « I* m wm m Wk.. <\MA ^^%^ M W ^M '"^ %//■"■ m "HP ■iW^A vxwM W W i Wf wM mm ^ - B /,..'/. WHITE. White to mate with a Pawn in 'snLteen moves. No. 48. Hercules. By Trevangadachaeya. W WM, ■ mm . m mm vmz. §§§i HS5H , . 'fiGZfr. IfiSI Mm?/, if im mt \ i If i B^\ 7//,.,,, 'A ..»,/;. ..III lU M In (*) i ■ ■■ ^^ 11 ( White to play and mate with the Queen's Bishop's Pawn (c2) in 82 moves, without taking any of the Black Pawns, or allowing any of them to move. Given in 81 moves by Trevangadacharya. Solved in 89 moves by Rev. H. Bolton, and in 32 by J. A. Potter of Salem. Mass. CHAPTER II. EUGLISH COMPOSEBS : Deceased Numbers 49 to 248. Living Numbers 249 to 448. THE PROPELLER. BY THE REV. HORATIO BOLTON. i i & f«I • #S5^ ^fe^i ^^M ^™ %fe«M H a fl ill £/^F.' Mi ffi^?///i § H mm - Mi ■WHITE TO PLAY AND MATE IN TWENTY FOUR MOVES. " monumeutum sere perennius ; Quod non imber edax, non Aquilo impotens Possit diruere, aut innumerabilis Annorum series, et fuga temporum." Hobace. Odes, III. 30. No. 49. By William Bone. .'■■ ^ 1IIF mm mm x lip §» mm White to play and mate in four mOTes. No. 50. By W. Bone. WT mm,, Jm ill il 7: m M ^mK^ Bm §Sml s ^m JL 03/A ft if ^5»ft -fi^m Mmffl-, "White to play and mate in six moves. No. 51. By W. Bone. Ill was* vJ^'-mw ^ / w / w^'ww /,y MWA . '''VMM. ''WilMZ"' "" / ' / MW/, I * ill * ™L w nU^iW ■ ™ fe ^ H W'M.'^~W//Jm V" White to play and mate in six moves. No. 52. By W. Bone. mm i PPI ^ n I i m tm wm. m fill w PW '1 I m si M WHITE. White to play and mate in seven moves. No. 53. By W. Bone. BLACK. mm. mm. m »l m #1 ■ ■ W fi 111 111 WHITE. White to play and mate in nine moves. No. 54. By W. Bone. m i§P\Ui/ mm •/////////A PUP k JKmJLj HP 111 6 HP «1 jB_JHL 9, Jili iAI iPM i wma mm. jflL m ■■ White to play and mate in ten moves. No. 55. By W. Bone. PUPP W WM. m WWA el mm *% m J. wMm. vEm, ■ WTdbP* =— ^gS^rt® I A £1 ffll IB. mm White to play and mate in eleven moves. No. 56. Agincourt. By W. Bone. i i ail /« VZr////ss/.s 4m III- ''WOffl', W%%%?, W//itf//, ?//, .if M 1^ *" wry Mitpfi H '^ ,-f^™^-™ ^» ^mm//. White to play and checkmate in twenty moves. No. 57. By W. Bone. ■ £3 ■ Pi flB ws/m. '-mm. wm In mm mm *%m A Wjk A fifi i ■ HP i ^ Jill 8 sell jiw % /■ White to play and mate the Black King on his own square in seventeen moves. No. 58. By W. Bone. W3 ¥ PV i Ill ^ m mJ m mm m Wm. , wmk,,„, i wr" m ;wB,,,ymm ^ m J Km mm M,.,.,..,,„?mm £Q)A White to play and draw the game. No. 59. By W. Bone. ill 'mm. JSy--"^"'"' ' 4a ■&■/////&„ f ■f-^a i HI lifi ■ B i ra #yj m A 1111 White to mate with either Queen's Pawn (d2) or King's Bishop"s Pawn (f2) in fire moves. No. 60. By W. Bone. w, w I S 'w/w/FT"m wAMJsLjm. mm ffsAf mm mm wm, mm, White to play and mate with the Pawn in five moves without taking the Queen. No. 61. By W. Bone. BLACK. W&w/M mm i i i f ' W£W% mm.,,,,,, wmM, m i r — I »1 ^^ White to mate with the Pawn in twenty-seven moves without taking the Rook or Rook's Pawn, or compelling either of them to move. No. 62. By W. Bone. Ml _fc m -J Km m 11 B ■ ////,9 / //// dm pf WW mw WW I £§ mm, i White to mate with his King's Bishop's Pawn (f5) in eight moves without taking the Black Queen. No. 71. By W. Bone. BLACK. "5ZZT/Z v/y// / // // //////// isli %a_m&sm Mm* m rf*ili3 wa m m White to mate -with the King's Bishop's Pawn (f2) in eight moves. No. 72. By W. Bone. ^, ^^H, Wow,., , v , VmmM ™ "■d^aJf^B ^ V////////M PI 11 i§ gi* lMi Uli iHH. «T ^ 9m,,^wm. w . mm. ■ White to mate -with the Pawn in eight moves without taking the Bishop. No. 73. By W. Bone. BLACK. ■ «■ Mi I i B — w in' §§ip ^8, WttZM ''warn. ''mmd'"' ' m m y/ ^^m,, ^A m White engages to mate with his Qneen's Book's Pawn (a2) in nine moves, without moving his King or the Knight at Queen's Knight's third square. No. 74. The Triplet. By W. Bone. PmMvf mmj m m iH jmj \ m% m mm?/, wm, „„JMm V wT 11 jjjl P ^ Jill J White to mate with the Pawn at h4 in nine motes, without taking the Black Pawn or allowing it to move. "White also engages to mate with the Pawn at g3 in 10 moves, checking at the 9th with the Pawn at h4. And to mate with the Pawn at h4 in 11 moves, checking at the 10th with the Pawn at g3. . No. 75. By W. Bone. 1 mm wm W" ia /mm. White to mate with his Queen's Knight's Pawn (b2) in ten moves without moving his King. No. 76. By W. Bone. Hi . wm^'wm, i * JgLj^fji ; '- j w, JB ■*"** mum J^ tKt ^J^^J^ mm J m m ■L M ™ „„$/. ft w White to play and to compel Black to checkmate him in ten moves. No. 77. By W. Bone. wm„^J4Bm. ill II H e« ■ H ffli up HI X ^^ ™.„, ^p I » ■ Hi ■! HI « ^ White to mate with the Queen's Bishop's Pawn (c'2) in eleven moves without taking any of the Black Pieces. No. 78. By W. Bone. Ill F I HlM White to mate with the Queen's Knight's Pawn (b4) in eleven moves without taking the Black Queen. No. 79. By W. Bone. 1 W, 'WW M H^r* ww, » mm *L0mL*JHM warn % JNM I 111*111 ^1? WW/ warn. % iM*mL-M ''WW' r~^ HP w. ■ft iiil t—* WW, WHITE. White to mate with the Pawn in twelve moves. No. 80. By W. Bone. m W"* I s '///////'A '/////////M tmo mm mmm jmm ww. w M m ik e. HH §■! H§H §111 WtM, a UTaH 'WW mm fi fit § W/W /\ w, -'mm 'WW, % White to mate with either the Bishop's Pawn (o3) or Rook's Pawn (a3) in twelve moves without taking the Bishop. No. 81. By W. Bone. m m wm.. %.... *«"«■ %i#„ ^^r^ i B 1.. r ^^m r .^ > W^ M "White to play and to compel Black to mate him in twelve moves. No. 82. By W. Bone. % w. ■JLPW". m m. bJLs 'ill fSl HI I ^L.'^L ^ White to mate with the Queen's Knight's Pawn (b2) in thirteen moves, without taking the Knight, or compelling it to move, 6 No. 83. By W. Bone. up m mm. ytfe ^ wnjfwm 11 |§J I ^H ijBB White to mate with the Queen's Knight's Pawn (bi) in fourteen moves, without taking the Black Book, or compelling it to move. No. 84. By W. Bone. 'S' m 111 . 'Mm, W warn &-wm m IP mm mm White to mate with the Pawn in seventeen moves without taking the Queen or Book's Pawn, or compelling either of them to move, No. 85. By W. Bone. ^ill . mm mm, mm. warn P •■ Ill i an* mi mm mm '/ A V //. - HH w It White to mate with the Pawn in eighteen moves, without taking any of the Black Pawns, or compelling them to move. No. 86. The Ambuscade. By W. Bone. BLACK. s "'- npfi ' i ■ s a I p 1 ; ^^^.„„„„„ r ^^^,^^ / J^ ! ^' W" it W", rrrr . r ,S' / '"""^ White engages to mate with his Queen's Pawn (d3) in eighteen moves, without taking the Book at e3 or compelling it to move. No. 87. Richard Coeur de Lion. By W. Bone, mm*- V////////A m m i£!.: HI m m^ m L ^"% ; W 2£ HP, iHH ^H " sum R g%« W, ess i^ ess i wok ■ m w White engages to checkmate with the Pawn which now stands at his Queen's Knight's 2nd (b2) in nineteen moves, without moving his King or taking either Queen or Eook. No. 88. By W. Bone. / 'y////i///// . ip ■iA 4 WW m 1*..: til K^&7%, W/////X/, elJ.i H ^^J «=S=. L^ 1 ?? 8^8 H« iH Hi Hi White to mate with his Queen's Bishop's Pawn (c5) in 2 1 moves, without taking Queen or Pawns. No. 89. The Tower. By W. Bone. m. mma iHH iHH IB %i%S<2 *^ s= ':,.~7%ii {f777^r:.. , wm, mm m ■ ^MaH^ r*yJB6m r White engages to mate with the Queen's Pawn fd2) in twenty-four moves, without taking the Rook or the King's Pawn, or compelling either of them to move. "White further engages not to interpose any piece between his Queen's Pawn and the Black Rook now attacking it. No. 90. By W. Bone. White to play and mate with hia Queen's Pawn (d3) in thirty moves, without taking the Black Rook at d4 or compelling it to move. No. 91. By W. Bone. BLACK. r'.'n * If H m ,,,,,, ,,wm. % warn w mm W„ mm i ^^ ^ €Hl White to mate witli the Pawn at Queen's Knight's second (h'2) in thirty-eight moves, passing the adverse Pawns without taking them, or compelling them to move. No. 93. By W. Bone. ■ llSi II il r ■'■--■-■ mwMM 117Z&7&. W/////MM WHITE. White to play and mate in twelve moves. No. 93. By W. Bone. ySl *" m mK J Km ■mm V"™ 1 mm. *3 I mm mzdi. ilH White to mate with either of the Rooks in fourteen moves, without moving either of them. No. 94. By W. Bone. m m #jS ,,„iwi ill ■ mm ii si 4k ■ !■ ■ ii a tt&f HHP m m, A iM^i, White to play and to give checkmate with his King's Knight's Pawn (g6) in thirty-nine moves, without taking the Black Queen. No. 98. By W. Bone. 'WM W k g§§|| f§§|§§ m mm.. m mm SHHl vw 'jam mj± mm,;j-^;w, m 'W mF** fc White to play and mate in eleven moves. No. 99. By W. Bone. pH wMm w& HHy ''/MM. 'W$M>. W%M. % /; ^\il w ww Wa M ; WW /J\ WW WW WW WW, W WW, m White to mate with either Pawn in fifteen moves at most, checking the previous move with the other. No. 113. By W. Bone, 'WW 'WW WW PIpW •11 wB, ww mm h ww, mm. ., la i« ^^ It. m-jm^^^^mKk % v///Wl WwJ 1 WW, m SJ mm 'v W&i , m i ■■-'/////a m White to play and mate in six moves. No. 115. By Rev. H. Bolton. s ■ in r ' wmm T"wm ill ■^Sb^mJ^h H fen. 181 MP MM WM///M White to play and mate in seven moves. No. 116. By Rev. H. Bolton. HH ^^■/////////// / '"'" '/// Mw%% MAm w,„ mm.. m_m &MJm€m mm White to play and mate in six moves. No. 117. By Key. H. Bolton. ■ .mi fc -%^™_~™~ ™ ifip i H _1 7 11 i 1 11 ■ ■ • « 4 ill « ■ i ■ I Iff ■ % y fe«>i ilii !&■ White to play and mate in eight moves. No. 118. By Bey. H. Bolton. IBf 'mm. Ill m m PI L Jf^J--^" ** ill &i n. «!„..._ ^B pp 1 «n WHITE. White to play and mate in seven moves. No. 119. The Escalade. By Rev. H. Bolton. BLACK. '■"■iTw p ■ III i iS 81 "3 H ■ P HI mA-%. ..... P Ml Hi '#$3, Wa .^w White to play and mate with, the Pawn in thirteen moves. No. 120. By Rev. H. Bolton. j IS » , w^L^m. /////j V//S//////A Wa,.., warn m ^^ WM VM White to play and mate in six moves. No. 121. By Eev. H. Bolton. MB mm »1 JM±M m flLJ"^.( i 9 MM H» H White to play and mate in seven moves. No. 122. By Bev. H. Bolton. -mm. -=C-*"- §iffl H H JL™ii im n Mi =*!&; mkMkM "WHITE. White to play and mate in seven moves. No. 123. By Key. H. Bolton. %H H •bfli ilk . ill WSKT wamt m w. 'mm.. /m %, iB Ufa 'wm White to play and mate in twelve moves. No. 124. By Key. H. Bolton. BLACK. m 1SI —"mm/-, HK III ymm. i^ :. iff A Pfl ' 'P « mmJ±M fM IE m wJPLM m White to play and mate in three moves. No. 125. The Arabians. By Rev. H. Bolton. —j™^..™. HP ■AJtePL ' m''' A wd"' A wf m % ^P ill Mil i HI P ■mm,w'" ™_P"« m m ^- jm M .mm Wliite to play and mate in fourteen moves. No. 126. By Rev. H. Bolton. »™t**J*™**»*^M« « MM tfr/M w////////. vrzzr.s '„/,-/.„■:£ WM W. HP IIP k IIP k I mm. mm. * ™1 * wm&m W0M WV if PL w 'ww ww ' 'I ■_^™^^"^ 1 4m, 4M,,p.:wB, ...m i Ws, ^ . 'HI' ^ ^^ fill White to play and mate in four moves. No. 13S. By Bey. H. Bolton. "iiW™" , *f : IrI mm..^Z...j„ ...m mfflk 4^1 SI J III pip - « ijBI. i%tmi %■//////?/. White to play and mate in ten moves. 10 No. 147. By Rev. H. Bolton. BLACK. ', W/, A^J m '— / P Jm 1 Warn wvwi. i ISP HP Vi&&. mm White to play and mate in eleven moves. No. 148. Ghuznee. By Rev. H. Bolton. ■ iJB IB iM WML WMLWm mm Kt^wm m i m^L W®*WM ^^^ IllP ^MM* W ill mm mm White to play and mate in eleven moves. No. 149. The Projectile. By Key. H. Bolton. fill ill mmm i 1 WB x MSB /s »i ^ HP ill i H^fii 1 fif I ^P Iflli *JP PUP Pw HH HHI §111,, « |1 p| VwssssSJSt / //////////V., w m p mm//,,. mm a , r £l l jh B bi §i #iii While to play and mate in fifteen moves. No. 150. La Chasse. By Rev. H. Bolton. ■...\^±.mm.^T,. PH m ^F m ^ lill & IS ■I ip ftp llf WSSk . wm, IS pp§ j i i iff! * Ml HI I. . /; ?>:^ HHSgj ^H § » White to play and mate in seventeen moves. No. 151. La Pucelle. By Rev. H. Bolton. wm, mm P wm w Wm WB &\ HI IBi ' ill iH «i .. «S i III B. * ..si 1H^ m 'mmFFm mm j&m fSi^ill 8 iiiw j£i m Hi H| White to play and mate in eight moves. No. 152. The Torpedo. By Rev. H. Bolton. *~w™» i Hi*!. A m *m Y//////M fm Hi 1 4y r ' ^ iUSi, .JHH1.. m «E1 I " H HHI s iHH White to play and mate in nine moves. No. 153. By Eev. H. Bolton. «&& JEHU. 111 H 4m i . mm_wm, V6 - 9, . H '/■■'>. White undertakes to compel Black to checkmate him on his Queen's Book's square (al) in twenty-five moves. No. 154. By Rev. H. Bolton. W - i fc P§|U H & B » 4m P jilt hi i i H ^H mm. White to play and mate in five moves. No. 155. By Eev. H. Bolton. WWM pp mm^WM_ mm mi. ?&&>, w mm mw, §a £&f Warn fiBjpl ilflll 4MM> fi osL'* ^p as ■■'*% *■ I iBf »lf illl White to play and mate in six moves. No. 156. By Eev. H. Bolton. BLACK. m km MMA i i^ei ■^ISI^ §1 ill 1 #Jli H5i ^m V////////// //////////// W////////S, o If * ni ■■ »« 1^1 «f m ww M WHITE. White to play and mate in seven moves. No. 157. By Rev. H. Bolton. vW- mp n mm. mm. P wzk~\ %~~\u~\ 'Z ■- .■ . '' ™«£=LiK»J: m„ wy/////fa III ■_*■ m 4mm ^^p J wB, White to play and mate in eight moves. No. 158. By Rev. H. Bolton. ""% 77Z.. %:■ "■ - ■ wm 'W§r^n t: & .. - . ..-.■/. y///v////^^^'''My^Z //, ^^'Z///"//y flM^^j/m 0A m I Ml ^1 ^H SI Ill IflP H White to play and mate in ten moves. No. 159. By Rev. H. Bolton. P w>. Uta mm m mJ m Wmk - mm fH. 111 wA -mm w m w Fm * « i bJ^VJW" i ^P jfej « i&J White to play and mate in twelve moves. No. 160. La Finesse. By Bev. H. Bolton. wet *t* ^P ^ ii II ^B ^ m .„. . i^ rii ML*. HH » « Mi £ 1 **JkM iL, Hi ■ ■ . |»! H Hi ™t^> y/ /,,~,., < ^ / " i W///// ™^W///W ™™™- W///M&,. mm. 'WM& WW>, White to play and mate in nine moves. No. 161. The Revival. By Key. H. Bolton. ■mm wm_ am * i III H&' mm, £11 w HP 111 mm. mm. ill WB. WHITE. White to play and mate in five moves. No. 162. The Enterprise. By Key. H. Bolton. 'jIB m ■ IB III wzMi..Q..,0m. m 111 w mm (■ ill "ip p mMi ,m 9 White to play and mate in ten moves. 11 No. 163. Moultan. By Rev. H. Bolton. %& ■ Hi mm i $ mm 111 Iff iP Hi JUl iO Lwl mm i^p p^^ ^^ ^ m^^Jmm, in . Hsfe 2^ YA—Jmm.., m.r mm White to play and mate in seven mores. No. 164. The Attack. By Rev. H. Bolton. '//A ■//. mm. Wfe»iff US i ! '/by///////, yy////////,. W 1 , W^jfll 111 mm i j^» " r „„„„„ v// "" //M ZJ*^«: * ^H. m 'MB. WHITE. White to play and mate in ten moves. No. 165. The Rivals. By Rev. H. Bolton. I 'W3, WM H wmk ■twm. ... ip J nw m' mm, • i M lifl I mm,.*mM &® ph MP HHH X IP A • ^» ##i White to play and mate in ten moves. No. 166. The Propeller. By Bev. H. Bolton. m x W* WW' mM fA I of ^ iUB „ mm. II i HP I HPf ^HP : ■ i In m, „,^5^M White to play and mate in twenty-four moves. No. 167. The Electric Battery. By Rev. H. Bolton. w -ww j^B,^ Jwm.jMm. "mm hi wm a mm 'W i» .ML i I ^ Mi wmj*jfflm 'MM?,. W 'wm& x *g® Y///WM m *M m W> lis :,„. «1 Isi White to play and mate in fourteen moves. No. 168. The Research. By Rev. H. Bolton. m *«"*i £1 Wi ■ m Hill »J^J8B» ^^ H IHP ill 1 ..„.§!I Q lill White to play and mate in six moves. No. 169. The "Warriors. By Bev. H. Bolton. "§=■=* ■ w ill S ill^lSJ M ™ — ™ * H wk WJk 'wmk ?;'?■■% . ^^^ ^?%% ^-^ IIP fiPF Wm mi wot fUH g;S ^ White to play and mate in sixteen moves. No. 170. The Leviathan. By Bev. H. Bolton. ■d™« ™- ^ ». HP ^ m^ A- m yz. Mm.M- mm fli & III 111 lb m& W/foms/, l White to play and checkmate with his King's Knight's Fawn (g2) in twenty-four moves. No. 171. By J. Brown. (Temple). w, & iHH #JH§ warn. WW, AiU m -jM W JM Hi s U -mJF m m m..u.ww, p in m, %&& WM 'Wk% m m "w, m^^,& WWy? '=' WmW, WWW White to play and mate in seven moves. No. 172. By Elijah Williams. ■ mm JaiUlI WrW, WW, *§H* WWW £Qi, mm . ■■. mm.. mm m mm. WW mm WWW ?/////, i „„„.iSI,„,*../^ kl wm. ' W *** m WWW. &. WrW an! i„,xr„ ^S^ White to play and mate in five moves. No. 173. By R. A. Brown. BLACK. WMA ^^ WM 'Wm ... m mm. warn WHITE. White to play and draw the game. No. 174. By J. Barnes. 'y/, '#/, m £#J fgJi illt ■All fift ffliJI ■i m W"w 1 & |Sj m^ nm White to play and mate in five moves. No. 175. By W. Lewis. m w,. ^8BL ' Hi * ■ III s «l §lsi , ppf| ^am •Ml i 1111 »l U "-"it-* ^ pil ■ m l «p nm 1 White to play and mate in four moves. No. 176. By W. Lewis. WWM ts=s 'WZM W/ ''ww- ■ i |H§§ m. ifftii o "IB i * KB * P. ^S^J ^^5 t« ■ : til HH White to play and mate in eight moves. No. 177. By W. Lewis. W HiP' iMik 4m, ml i £ uBj ill si IS ■ B i B fi ■ HrrrfTL mm .my/M4fr,„ w = «i White to play and mate in four moves. No. 178. By W. Lewis. 2, Jill "UJ^V HI I mm,. HHP" ^^^ ^ h^—jwk ' 4^ WW/////// V//TX//// Wliite to play and mate in five moves. 12 No. 179, By W. Lewis. WZMA tH m mm ; m , <||> p mm. Bit! Ill 4 m ■§ §§ §3^ % Mm c m~B-wm mi n» i§ m£A wmk mmk ^ 4^A m HiB P 1111 HH ^ IIH White to play and mate in eight moves. No. 180. By W. Lewis. ■tW&l HHf HH -mm, 'rim-< f WHITE. White to play and mate in five moves. No. 192. By T. Smith. wm fa "W 1 i • TaTL M White to play and mate in four moves. No. 193. By T. Smith. i§§§§ g§§§| p !■ *m IIP ' P "■L, mm. ■■■'■■'■■■■■ fm \ iaPW : ^W/jmT' V m III ■p*"- w % r ,Jm ill White to play and mate in four moves. No. 194. By T. Smith-. /ft* Sib m. w,***Wz%m.. \ S" ^.,'~J™"' wy////////. r mm & m §lil I Pis pi HP '^B 3 W ////////////. 4?/////M WHIIE. Wliite to play and mate in three moves. 13 NTo. 195. By T. Smith. ty////////?. rri&^^////f''Z / '''WA :■ HP mm, m9, ft m wmtfm^ i !i % * Mill ^J p «1 MM H '; White to play and mate in five moves. No. 196. By T. Smith. BLACK. ML- A. w mm, «L~. #> mmm m m III ?//////%. ill Pf '/7//SSJ/SAH WLJbL %^/^ m r wwm mm m mm HI White to play and mate in four moves. No. 197. By J. B. of Beidport. 'WW B I W0""i""W, mm —mama w mm M7M WES!?/}}?. mar ■UFL ^„„„,„jiiiL„„ Su 1 If IIP MM White to play and mate in three moves; No. 198. By J. B. op Brxdport. "■i! 1 " 1 ■ WW/ m mm. M mm P^l W-% W, Wm. " mm, 'mm. w. m p M w^' mm WHIIE. White to play and mate in three moves. No. 199. By J. B. of Bridport. M fci VlH-flJ" m m & wM „,.„„mm, mm w ^ v £!L$™A,,,,,,,J®^ m i m "WO0""*' 'HIP if w ij§ij^j 4w< mm nm §1 Ict mam,,,,, mm. mm. m b « n White to play and mate in three moves. No. 200. By J. B. of Bridport. fe Br y - , mm, mm, m HI mm ',■ M4 Wmk tllll * ^ ^« . .. I mM, HH llil WHITE. White to play and mate in three moves. No. 201. By J. B. of Bridport. 'mm. nn ^ « mtfrnSJ^ im. km ]■ i ! ^ «1 * SUi"^* i jjgywLjMj^ ISI m Wffl8^^W ^^ White to play and mate in three moves. No. 202. By J. B. of Bridport. BLACK. ■p ■ s ■ H iKi ^^"^ ^tlflilP ^i$HH WHITE. White to play and mate in three moves. No. 203. By J. B. of Bridpokt. ■ -««f« ■„-4-P. P It ; H % WHITE. White to play and mate in three moves. No. 204. By J. B. or Bbidport. m ■ i ml i H 111 w/////;yZ>., m Mb mwA wk mm mm Whito to play and mate in three moves. No. S05. By J. B. of Bridport. M ij H 4™^—™= « Pf2f mssm m m i msm ■Usf m ■ ■mmiw- WHITE. White to play and mate in three moves. No. 206. By J. B. of Bridport. 1' WW*/, w,. m w J Jm m itiiiC mw, WW? ^ i i r WW ^ iww m // 'yMmw-J^ /////// ^'- iwB,§. WHITE. White to play and mate in three moves. No. 207. By J. B. of Bridport. mvsmx, »^- V////S7///, M m wmT m m i si ^ „«t mm Hi my, w////////f. HI WM €l§J m mi »\1 aL. fat W/, "White to play and mate in four moves. No. 208. By J. B. of Bridport. BLACK. P; #J^ WM% .. : m w v/ mm,,. W£':% H Hi •,—■■1 ^J^JiiilL W tli ii ^ WHITE. White to play and mate in five moves. No. 209. By Silas Angas. BLACK. y ///>. vu W, y//////////, mam — - «^ W"~ i r 'i '^'WW^^''W7///"~ WHITE. White to play and mate in three moves. No. 810. By Silas Angas. i iH§ MP * PS IL 111! w 88 10 WHITE. White to play and mate in three moves. 14 No. an. By Silas Angas. ■i i r iy w iM '■P &^mf7i IIP wM WHITE. White to play and mate in three moves. No. S12. By Silas A.ngas. P i ib. Hi w, ^e E w= r ■ 4™"^ %^f^ M 'r^. White to play and mate in four moves. No. S13. By Silas Angas. m mm,. pif mm x mm x mm m...^.MmM. 2^1 i„ ei^M « [ mwMMiz^Mi Jr** »f±*— WHITE. White to play and mate in three moves. No. 814. By Silas Angas. mm tBi mm M \S\, 'Hi mm w mm ram mi WHITE. White to play and mate in four moves. No. 315. By F. H. Deacon. »\1 %t III Wl m 3~m i im 1 i ill ill xl „ jhSL. *: . ... mm ^ mni g * mm, HBHL ; ISP i3| .;.;; d« White to play and mate in four moves. No. 216. By F. H. Deacon. :%is W////////X y/s, wWa ^ , III wk,.. m W, #1111 mm. WHITE. White to play and mate in five moves. No. 818. By F. H. Deacon. „ """""■„ / . ///// ,,y // " / ^ / -r // , / - / ., / -, / """""'■ /"zztT. mvmmv. M» 4m fMB Wm. W%M„ ,^JH, vytXky/j %r£\4 ySsSmxi wrJtxi ^11^ '///.LtI?, W^,'. '///.L^i 5 ^^«»«««^^ :: ^^j««««^^ // ^^^>«««»^^^^^ wmr* w pi OV/////////. 1 ■ White to play and mate in five moves. No. S19. By F. H. Deacon. mm ^■AII m kmm 11 i§ IHIJ % ■■ White to play and mate in five moves. No. 220. By F. H. Deacon. ML, IB! SB p?^ ^^^ «m Pff P ill ■ 1BV i. ™ iii iJLi i® ■/////////// /y/ 7 %_^_wm mm. White to play and mate in seven moves. No. 221. By R. B. Wormald. BLACK. ■ til < iij.il Wv tip .Mr//////'/. W/////M&,, *w White to play and mate in three moves. No. 222. By R. B. Wormald. k §§§ k 'WW Mi - H mm .■//, '///,///////, WHITE. White to play and mate in three moves. No. 233. By R. B. Wormald. BLACK. m y 'W^% HIP f$M 1HP H s jmm W//////M 1 ,,, mm 1 mm- ■W*" 2 WHITE. White to play and mate in three moves. No. 324. By R. B. Wormald. i m Biwiq m III WHITE. White to play and mate in three moves. No. 225. By K. B. Wormald. H I§1 WjgtM m w w"V WHITE. White to play and mate in three moves. No. 826. By B. B. Wormald. mm i §111 * --Tip ■ m ... Wt, -. •..••; Ill 1 an ■WHITE. White to play and mate in. two moves. No. 228. By R. B. Wormald. BLACK. .. Ill w fc ^. iSa, ^Sfei <^^2 'mm W mzm, HH H ;t> 1ra m/ fflf :f #l i Pi ■I EljK ■mwJ^^^Jt^m m i mi HII II- ^ fc«^ .i§ P «l White to play and mate in three moves. No. 229. By R. B. Wobmald. ■ m ®- J* v'/mv/x: ir^rrr', r. 1 HJ MjmAJm^J^ White to play and mate in four moves. No. 237. By E. A. M. M. ■PtfM mm M mWk 4 wM,=-„mm, jBL^ljhnhl \% mm ; fi 111 ■ i W%M Wwk White to play and mate in five moves. No. 238. By E. A. M. M. im -mm. m wm mmi 8 ! ^ a vB?A c A Si lite ww, , fm wm% W% I wSTETM —— „w//////m ii#Hsl! . PfWlO 8 pi wmd. vrnf ' II WWM. MM W fTj ^P IS A I to ■ iptf FBI White to play and mate in six moves. No. 244. By E. A. M. M. ty/' 'm-'-S'- <4 %% WA.Tr'wmi M"J wmU 'mm m ww^mm m WKK^^mm mm y ~~ WHITE. White to play and mate in six moves. No. 245. The Koh-i-noor. By E. A. M. M. flat ''■mm S#///S/SSA ""'" V/////S/A Wk W/Z&>//. %WAW///a 'Py^'ffi WZWffi ^jftwm Si^il 11 \tbM 111 WW WW V . ... j WHITE. White to play and mate in six moves. No. 246. By E. A. M. M. 4M Wa) '%j9%fr 't^&as?'' z*w5g% m &7//J, ,„„,„„ %///////£.,„ 'WW, r WW ,//s////7777/,„ mm mm JmWL WHITE. White to play and mate in three moves. 3fo. 247. Mhow. By E. A. M. M. m, ifcf mm A Si mm Mm m Hi W//////ZU,, m Wm WHITE. White to play and mate in four moves. No. 248. By E. A. M. M. iW™ "W if w mM>, m%M wm. w> mzm. fen wwm mm. a 'mm, _r_ m ", _-i | ^?« l- « K m White to play and mate in five muves. No. 249. By F. Healey. WM v ; "'"'% mm /A m im I -vFT'w, wSk mm. mm w White to play and mate in five moves. No. 250. By F. Healey. t^jy / BLACK. ' ■ PI % w ; ^^ ^p 1M ill fli a i * i i ^ i my. ,„„„„„^M^^ White to play and mate in five moves. No. 251. By F. Healey. A w, M wmF mm, mMM,...^!, ,,.■?//,■/,/////. 85^^. a H m I#I ^P ^ P ra III ■MfcL mm, mm, ^u- iO White to play and mate in three moves. No. 252. By P. Healey. *.mm, \ m m Ill % I 3 P P m ^P * .1 8 WHITE. White to play and mate in four moves. No. S53. By F. Healey. w m -mT m m KXOMMi „ . S^H^g., ^ jSII ^^ ^„.:r,,,.^%^ b White to play and mate in four moves. No. 254. By F. Healey. AiL wm^mw. WM-mw wd^'m wmrTm m m^m warn mm mm w — mm//, „„„„ wwM.„„ r '&%&>}.„.„ wsm. . WM. m§ II _„0M j^^ mm„,,,„,,,,fa ;. '■i H ^IP p II »lll«j^yyj 39 fy\1 mm. I White to play and mate in four moves. No. 255. By F. Healey. w> ww3... W. m mm. w. mm _ mm , mm, i 1 * mm H, #iBSi ilBl m. m m m^mi m ■f ■ H iiii 1 ™ # Hsif p3"| iUPP 3% ggggB^ Vy/////Mt, twwZy, WHITE. White to play and mate in four moves. No. 256. By F. Healey. "I I ■ w pi 4^ pi III A i#Jls illl r , ^ PPf ^ 'WM WW- White to play and mate in three moves. No. 257. By F. Healey. H PP "WB, "^ iHi ■t - - isi B ■ ■ ra « M w 1 i ■I ■ b Isl am,,,, White to play and mate in four moves. No. 258. By F. Healey. -mm wm iHf ill 'W% ■ Mm y/ w/////////., aw. s & ? ,-^u I ^$ I « 111 wm White to play and mate in two moves. 17 No. 269. By P. Healey. #1 m % %-PVi §1 'WM wm tmfr,„wm,, jjpijji i *■ iH ft a£jBS» BjfcS WHITE. White to play and mate in four moves. Wo. 260. By F. Healey. HI n II 1 .WW//. VA pit mm m , %mm. ^^^ B W, gffa WHITE. White to play and mate in three moves. No. 261. By F. Healey. Hi WPWi &\i s' e =».« §0 / '-v m - //////// - Warn. * %wm m '/m fy/////;y/?. mm ran mm % m (*) i ri, b White to play and mate in three moves. No. 262. By F. Healey. 'wm w A ftM m ^™ — —^ "p. ■■« H Wm. White to play and mate in three moves. No. 263. By F. Healey. BLACK. ^rnJ^m M **. WW, VMM up i i „„„^*..; "'■> ''//////////, ■ yy/////y///^^^ m ww, y/ to 'ww, v ww. WHITE. White to play and mate in four moves. No. 264. By F. Healey. '■SWiffllt. '////, w m v ww, m m in Wm. WM. WW/, WW/, B, , m o **-* WW, -s- 1 P ,t, ^^ & MM W/mw/,J±%mi , mm ^j^jb^jB^^i ifll 111 gl^ White to play and mate in four moves. No. 265. By W. Grimshaw. "WB I ||§§| As ^ m ?'■••■; m mm it White to play and mate in three moves. No. 266. By W. Grimshaw. m« ^ ••£--•/ ■ ifi fl »» v^m. m r/ m mm /y/ n illy Pi *% »/////////. ^^ mMM ■»/////////, ggassfei WHITE. White to play and mate in four moves. No. 267. By W. Grimshaw. - HP Iill„ ■ f— ™ ap J" 111 |"« Hi 4W™^ ?i A mm //////A w?" % w//M, i mi i;m * in ■ri ■ MB. ^ SI ##Jf H ^■^ ^SS>% ^^^ iT ^ '^j JBJ JHHf HHH Ml « White to play and mate in five moves. No. 368. By W. Grimshaw. \i\k\ ;•# WM A & MM w^Sziz. ■■'"■"A mis. W 111 : - m i,o ill JHL JH pi tm White to play and mate in four moves. No. 369. By W. Grimshaw. ™H*_« W m ,,„....0i00. %B^ i 9 LU f'wmfw. * mm, ill i fai iB ki m i r^ i mm ■m0 §p ip WHITE. . White to play and mate in three moves. No. 870. By W. Grimshaw. ■ i IK *■ W \m V ,„ ft/////////. 't%MWV, WMM/: in w Mm,.,, %?,. .... ill I Ml PPP P¥i P B « p f White to play and mate in four moves. No. 271. By W. Gbimshaw. mm % ■I Bi y/mz ^'wzzzzzzz^^'m- w mm, wm^/ww, mm, a mm, ^ '-mm, * y WS/S////Y/. Ifl i fan VV' pi pi H ^ W//////A Z///////Z. SSSfe^ "White to play and mate in three moves. No. 272. By W. GrKIMSHAW. 'W'ZZZ^ "-"-f W&$%!> W///////& fa is i i 1553 pj*i i .,,,///A '/////S////A pi x k IB I m iHl White to play aad mate in three moves. No. 273. By W. Geimshaw. %Wa mm. m a m if (S WBSr mm. mem *y//////AY/. W/TT; ;;///, ppsp ffpl m mm iHI mml wm. wmf'^'''Wm . W4W, W/, %Mm ■A M iBJBi Smi^^wA HI Wkwmm mm, ■ III i ¥ M PS *H €sl„ Jill ™„„__,„,w//////fa 'oiTT'.., m m i Hi W'mwZ- ■ww/ ////// ^wfmr^^w& White to play and mate in four moves. No. 282. By H. J. C Andrews. m. 'mm W///////A ty////////i »» , ■X^t^/':- y.&£% warn. 'VMM; ^ WM I PI I V/////S///S> , '■mm w. Hf ^1 4%zfA White to play and mate in four moves. No. 283. By H. J. C. Andrews. J""* w, £» ^B,...§....^B. IS i ■ m ■I fej i'*H HHI ■ $1 ■ § IP ||f§t #2_ 1| if™ ,1, ^»| White to play and mate in six moves. Wo. 284. By H. J. G. Andrews. ±s "■%&% y MW% WZM m ?Mi mm Wm^m y/////////, , v/yy/7// „y/sts////%>. m wm mm''"' fmwm m til ii Jt «L ™l , 1 ^mm fe#i ira H fe*l ™ 4MvA, Wmk, mmk Wm mm ?m VmTZZi. WW/0, <£t%SZ&. White to play and mate in seven moves. No. 285. By H. J. C. Andkews. ■ w m " m w^r- '■ZWM *m m „H§ mi f#J v mm. PI i mAJmW.WB_. B ISif «§! Jill P^M Ji^AWf& y/zy/J^^', ///////// ^^ White to play and to compel Black to mate in eight moves. No. 286. By H. J. C. Andrews. //-/A ¥M vm WM„,,'%m, v/m iJiii „,,,...ii§ii mil i jy ^ ■(- §■ , jiii 'V/1'.L'V'/, v'/l'^'/J'/, ■///''''>-■ W, '"A, ■'■ yam A//////Aii o Yflks''. WyW, a WZW/, mim m a WB, ■/, r— •? w//A//i White to play and mate in three moves. No. 287. By H. J. C. Andrews. m mm. mm ^ W//////A.. III 181 ^ww- mm mmJ am m "Wliite to play and mate in three moves. No. 288. By H. J. C. Andrews. II. mm 1 fHH up ■?»"— "™ ill ^M W//////Z& w$% ^ Hi iMI 'fa' i 181 WHITE. White to play and mate in four moves. No. 289. By H. J. C. Andrews. BLACK. . %®4 mm m tmmf™mmJp*™»*mJ '^^ JSm. 8 KM. ill up a m WM mm %^ ciL. wM/7/, WMt; Ml fHf WHITE. White to play and mate in four moves. No. 890. By H. J. C. Andrews. ^B. isky Wm k wlm^M^ k I ^3i * ^^^ ms Warn. * * . %mi*WM. w Mm, iJLil _■_ BL H Us? White to play and mate in three moves. 19 No. 291. By H. J. C. Andrews. ^ISii W//M/-, ^ ^^ 8 1 , w/w/-, //////ft. mm i ■ mJkm m ill ^^; wp ill 1 fill p n . i§n§ ^ ^^^^^j^^' Wlri*e to play and mate in four moves. No. 292. By H. J. C. Andrews. H M ■ ill I WW; ■-J" M* 1 ^ si > ill ffif © H M Wm\ A H#r iJJfl^eJ ill IP ■■ / //7////////. '//,^* / /. M, iS White to play and mate in five moves. No. 293. By H. J. C. Andrews. ■"■■ IP mm... m <&„„„ i //////////. v/Mmm m&M ^sf'J X ■ H : ^X ^H' #-^ jfl! ffl ■S m H vwm, White to play and mate in four moves. No. 294. By H. J. C. Andrews. ill :k^ y/!55!!?M. ™% mm -*- wm_, mm, mm ^wJ^'mwF m 'mm. m^m m i %8„ 'sfTffsTTXZ I/////////:., W//////M,, #1 m S^^'m^ m w""'- " 111 H kJ±Jm WHITE. White to play and mate in four moves No. 295. By J. C. Roll. ^ 'WW)^x,Mw3. mm. wm y////////^^^y/^////^ WW, WW. v££ WW. «h_ H _- m^bk w jm^ w jmm m* m ww, M, mW, White to play and mate in four moves. No. 296. By. J. A Coneoy. i W m ww, ww, mm ww. ~M „,.,,,..., Wm, S3, 81 mm mmm jB8m »j^.i Jill 4mw,^..ww,, // ill WHITE. White to play and mate in four moves. No. 297. By J. G-. Campbell. fawm w ■m. x m :;-%—• m muf l ' m ^gg^ % f|p ## ; HE (f ■mm. i tM m f "if" l «| ill. White to play and mate in three moves. No. 298. By J. G\ Campbell. MP M mm/ hm ^n WW>. 1 M -*■ M Pi i pi pi %7&7?7ti, V////S/M..,. ~ ,,■%/////////.„ -»//////,//.. ' /M Wt ^Ifil M White to play and mate in three moves. No. 299. By J. G. Campbell. 1 i ■ n ^ mma. iffm^/.. wSffity,. s ^ im Jmm^ Mm. \\ i lAlil mAm(M^M m w/w mm '^^ m ^' White to play and mate in five moves. No. 300. By J. GL Campbell. H§J ma mm. © a mm 'W&& 'WM WM ^™ - iii mm, WJm, 4M J**J£J* . ^SsfM W777777A Mv/awj '■Ztt^FtttIl WHITE. White to play and mate in five moves. No. 301. By J. G. Campbell. PP| pip pll lih Mom m t-jmzsm^jm m^ mm * A ; X '-'.JSm "HI""' m WWM WHITE. White to play and mate in three moves. No. 302. By J. G. Campeell. BLACK. HP ^ r^i m% a if km h m V/////////>. 'mm wm fll ill 11 W&& x 1111 tit SiP H ' m Aim iff 9 m i ■21' '■ a iKJ & wmr^m White to play and. mate in five moves. No. 303. By J. G. Campbell. mfimm '/StZtSiz w,„„„ /M mm H ill HP mi n "«t L. WHITE. White to play and mate in three moves. No. 304. By J. Gr. Campbell. mm I 1 ? a H, ISJ b. ■«v~ illl. wMk„. i£m " bSJ %„„. jsam White to play and mate in four moves. No. 305. By Rev. C. E. Ranken. m wm m JL III ■ mt I ill I ill iSi HHf^. ^ ^^■mw/^^^w//^/J^ s ^'w % ^^ ^*s» WHITE. White to play and mate in four moves. No. 306. By H. E. Kidson. . V/////S//M,,. 1 wm « .iH li III *9p i : ■ ■ ■ wm wm w//////', IIP m WHIIB. White to play and mate in three moves. 20 No. 307. By H. E, Kidson. nm M mmTTm '///,, '////////yy/. m WiM. W. i pi & p| IB > * m ■■- H ■flfisffl, White to play and mate in three moves. No. 316. By H. Turton. Ussf Jfil III * #» . Ki gfeafa H. v/s///ss/yh j^^fw^?> ■%/%%% JvAtCCwvs H. ill H III White to play and mate in four mores. No. 317. By H. Turton. J^mm/^ggJ^i^™ mm w. ft , w///////; WM WMk WMM y iW?A 1 fli w. wk PI KS«v^%a timy/M?. m ii si m imi wm vm ww, §;.:./ WHISTE. White to play and mate in five moves. No. 318. By H. Tueton. ..4«" ■" ^•? ^S mm m mm, mm, mm, w f FBI "if b mm,, . mm, «i m ill 111 1* 1L till White to play and mate in five moves. No. 319. By H. Tubton. WW. mm>A WW4 im fm #1 WM.fBA^P?^ A if W* 4 wm mm WsB m WHITE. White to play and mate in six moves. No. 320. By H. Tubton. J HP IP m l», ^—i Mi i WW "WW P^ ill mm ffl i w////// ,,imm 'w-v/// '' m ^\^//// m//m ''y / »%*p^ '"^w/maf^^vsm H Hi White to play and mate in four moves. No. 321. By H. Turton. Jft, Mb m. V///A m nm mm m r K ' « — IT " ; W//////Z4, ■xy/////zZ>, ^HH W&M § H „1W1 i^i ^^ JB pi n "r B mi « ISS IHf WHITE. White to play and mate in three move3. No. 332. By H. Turton. WW4 IP * i^n m M*m.,, #HI w If ill ^wsm r , n 1 Pugf H %^si a rfl WHITE. White to play and mate in three moves. 21 No. 323. By C. W. of Sunbury. ■ r «■ W"if if -< J MJPtJBL «1 White to play and mate in three moves. No. 324. By C. W. of Sunbury. i§ iai w £11 w m ^ m J"% ^ (22iiSK2 m m w i„. h White to play and mate in four moves. No. 325. By C. W. of Sunbury. ■ /Wi WZM ■P IIP %%23 tf777A^7*7. <& mm Mm. mm %m% White to play and mate in four moves. No. 326. By C. W. of Sunbury. m mm mm mm §|||p wk t™' i mm wm. H^HL 'mm,,.._.,..M^m. V/////////. il * J B if Wwm,,. i. ■r WHITE. White to play and mate in four moves. No. 327. By C. W. of Sunbury. im mm... HI mm& W" m y mm. ^~- : P%^ ^ ■ k HP ■ i m^M m JBWiZ£±jBBM m m mm 111 2% Vsss//ss/y/. White to play and mate in three moves. No. 328. By C. W. of Sunbury. III HI M . ^_^^«_— ''7777/7777Z V/////?/?/* White to play and mate in two moves. No. 335. By I. 0. Howard Taylor. HP ill ill ifa " i mm, *mv ■>, w t wm m I .mm «= SSs i^f JSLJam^^Sm^. ^.^ ■ ^^^ HI White to play and mate in two mores. No. 336. By I. 0. Howard Taylor. / Wk-.M l5W# f^M §ffl§ I j ny im M m m m ■ * a White to play and mate in two moves. No. 337. By J. W. Abbott. W. \J*JHk iSJ. ww' m imm ^^ ... Ml e - WWA Wwm ^^^PiP--^ HI *t\ Hill 1 W wmy-iwm mm .■■ i» ;'-T';l »M pw WM m m i ■ White to play and mate in three moves. No. 338. By J. W. Abbott. i MM m m m llBlli Hi WM ■V -1 mil H i wB, W. wm zmi ~ — ~ %•/.'////£ ~ — " vxfiwA, % A Si V-E-- ^P 1 ^p HP 2 J lib "^- j i White to play and mate in two moves. 22 No. 339. By J. W. Abbott. 39 Wm PP IP ill «S> WM.,,,,,,,,'WM,,„„WM. mm, wm i ■ (Hi 1 1 m,.m.wM m m «^-- *^^ V/ ' %m^k mm mm, %mkj*Jmm. i I ft » ®i * HI 1 I 1 %8 '///////////, 4M y/ JB,. m Wliite to play and mate in three moves. No. 340. By J. W. Abbott. 'wm m m ww? m W - Wm HI HP, & w mm WM Wi§ PPP* Hi Hi A Wmk WMk WWk fSi I isll ^m^-jak. White to play and mate in three moves. No. 341. By J. W. Abbott. musJm^Jm. m wm-\u/^, 1 Hi sb i wmi \ mm X wMlk w/////m W///////A Bl A :a ili . Si . (■ iImI ^M w rwKT m ■-■■11 ■■"■/'///, u %?%J: k 111 * rs^ H llli %fei?J White to play and mate in four moves. No. 342. By J. W. Abbott. 'wm ms Ws ■ % Mil i i lllli k'A lllll ffif W:M ''/////////A WW"'- '^^ Ml ^^„ ..im iii H IS til % y JW//M, mm,. wmm mm. White to play and mate in three moves. No. 343. By J. W. Abbott. HI n ..iHH WWS/, Wmm WM% Wm WiM (m mm m W»M %5fegKrf UMM . V/////////. "White to play and mate in four moves. No. 345. By James Pierce. MP ML mm m m, ////////M ^m. Ill iiii — . H 1 I . ag« — B r r - "■ W, White to play and mate in three moves. No. 346. By J. Pierce. H mm... mW, - wr~iV WW '^ Wf HI Iff ' ™* .wwm M^% #18 A m •i. • ^as. * III i ijm. HP White to play and mate in three moves. No. 347. By J. Pierce. an m m ^ '///. i a ^ «« * m Yy 11 a 1 ^ WHITE. White to play and mate in four move3. No. 348. By J. Pierce. — IP ll is IIP ill _ilil 3 mm wm il ^ w/M, "White to play and mate in four moves. No. 349. By J. Pierce. ::^HM mm ' mm ism* m mm *mm mm a wm wm a a kVi wm wHm "White to play and mate in five moves. No. 350. By J. Pierce. "pTgTl 11 ^ M 'mm M wr WM 'WB, W1M ■ ■'////%!', s£f5zH ? *W J White to play and mate in three moves. No. 351. By J. Pierce. hi mm mm^jm . ■■Tfr-i I H I ■r • » til iH !l«i mm fAl '*w> wmfim MWZM y/////y//Z; '■/////////■ w%m. o ^^^ 111P ^^ .....1 ^ iHI llil BHi 1 gp mm «* «r- White to play and mate in two moves. No. 36jf. Z- By J. Pierce. ?% I§PeJs3§§P y/M&..,^/mA MH.. If ■■ ■ ■ 'VAvtttt,/, tfiSBSSSit iiti it m *SJ m wtJ® m w''" M Eft ■■ 'mm *§ a mi up i:^«IjJh Hi lllf ■ I lUl HI White to play and mate in three moves. This Problem has not been published before. No. 353. By Rev. W. Wayte. IIP HIP IIP IP warn, , "W WSB WM. " WW' WM' flbi mm. mm.,,.,,, \ w, J....JSI ^HH w//////.... r/ zzz/zy/Zb. IIS hp ^sn lay « ■/■y.: %:.,z -%y//zy//): wfiffim zZXY/////. wm * * m i ill III i,- White to play and mate in three moves. No. 354. By Bev. W. Wayte. WM I' C "Wi Zz\wM L — ' Z7/M/M * W" V">,////,% ^ppi A ^p X % * I ,'yyy/////i% ., :.,.... J I J J V lill i§il l^P «*» ^^ ^ IP IB HP i «ii g #11 White to play and mate in four move3. 23 No. 355. By W. Timbrell Pierce. iwffti ■! ' 111 mm W III ■ Ik ^L 1 , i JLfHi - "H ■-^H fc M "w^aHr ^^ ..MAmLili «i ^^ W////M" ''WMk %m White to play and mate in three moves. No. 356. By W. T. Pierce. illlJ ilil ill MimmP^waJ^m m " ^w^ .*. i ,-/,.;. .v, A Wm i a. WHITE. White to play and mate in three moves. No. 357. By W. T. Pierce. m m m s ?1 Iil§ f: ■lg| « H P ■ ■ '<&% 'mm §Pi| « ^^^. IfeM fe^l _ Jite u ™^- mm pM wM x mm, lb WHITE. Wliite to play and mate in three moves. No. 358. By W. T. Pierce. MA %S^ -J**- #~^ L — ' W%m 1mm. mmWiM H HP ■ ^ W V# rl^ WHITE. White to play and mate in five moves. No. 359. By W. T. Pierce. %-P.A ■■ fl .M ,H JML m m w///////f^ m m W I in 111 ■ B %-/////////. 'd/OOzm. mm mm, m i ill White to play and mate in two moves. No. 360. By W. T. Pierce. mi m Br i 4 »* w™^ A w "-- ^ ===i t/////////k y///////// W////MM^^^Tfa/#ASffl f V/W7??/// m wt/ m - ,•&•/////. W '//WW, Www? f~^ W"M> WHITE. White to play and mate in two moves. No. 361. By W. T. Pierce. §WI I W£M -, WW W i m 111 10 j» ill jM iSl m*tt& i^*** wmtmfa ft£vS5xAa White to play and draw. No. 362. By John Watkinson. m , 4>wA ....warn.. iff ?W. i H^jdLjHl White (Mr. Watkinson) having to play, won the game. No. 363. By Rev. A. B. Skipwokth. mm m , w*m fm i I * HI WM - ■ : S i<^fli III *Ji ^ IM w mk *m mm m 4. 4%zm Li us W^^=^=' '/>//////%i wm m mm LSI 11.., H pip mm wm wm. ^m^^Y-- ■"■■■"■"■ til J§p f&f ¥ White to play and mate in three moves. No. 370. By Eev. G. Mc. Arthur. 'MM. WW4. W, W£%?,. mi Wk mm WMk ^H ■ Hi 'mm. mm. -mam. wem i W^'-i" 1 - s H Mill "5^-» ^^ Pj P H Hi » Hl^ wm%, mm. m White to play and mate in four moves. 24 No. 371. By Rev. G. Mc. Arthur. m 4/ wi m w §m w mm »8^r . - Ill fill p/^A Jt^Szi. W//////&, .„&m5mi i mm i m White to play and mate in four moves. No. 372. By W. S. Pavitt. v m> m ■ "111 JP IIP "ail Jem. Wb ww iHHf fi__rai__il§il WHITE. White to play and mate in three moves. No. 373. By W. S. Pavitt. i ■ I ill m^jmmmjtm^ ■ JE.il I Hi 01 n* 4Km r ^ mm WM m m Jm\ mrn^m n mi WHITE. White to play and mate in three moves. No. 374. By W. S. Pavitt. MJ m as jj 11 ■&!! ^ g;**... White to play and mate in four move3. No. 375. By W. S. Pavitt. '|jp k Pil^^P i 111 wii «y 131 i4t,u eH .. mm mm 111 §111 1111 1 S White to play and mate in three moves. No. 376. By W. S. Pavitt. A m J^< w -.jMm.,M w, mm,. 1 i •■ i IBI H. ^^ SBEbb. 11 IB ^^ WHITE. White to play and mate in three moves. No. 377. By A. Kempe. w//. w/;;y/'//. H i ■ ■ ■ 'w//.'//A M wmT «i w, I i « wmamt ; %km B ■ Bi-I -mm -ww !R III ^ dfll Mi White to play and mate in three moves. No 378. By A. Kempe. BLACK. WM MM/A . =L^llii.. wm kwm Wmk . ^^.— MM— ,0^1, iii pip mp S b n fli i IHt tsi i ^ A PHI tsa ill m Ji !M. WHITE. White to play and mate in five moves. No. 379. By A. Kempe. J I8J f «J^ I i WW4 «T\f ^ „,%?/////,//>. Wm rMe mm mr 'wv/WP X fillip ■ Sir 1 PI ■ i tM WHITE. White to play and draw. No. 380. By S. H, Thomas. Ed f»cS/i K W//////A '^//////A. ^HtH 1 fi Wt Mi ». ^K„ ^ n 1» A El 'T m ™— M — J "White to play and mate in four moves. No. 381. By S. H. Thomas. 'm ft i \\i •■• m m m m ', w. ' 'mm- " m ^^ m 'mm mm 'mm^^ m ' m Up f4£ "White to play and mate in three moves. No. 382. By S. H. Thomas. ■ waum ftei "wm -^ %s?i %MW zmmz wm&, mm: i tri, 111 m, %//////A 0S% w mW m_m White to play and compel Black to mate him in five moves. No. 383. By S. H. Thomas. mm pn m m. mm. I -*|HI wk ■ Wk » v * 8mh« -mm Z M 'w I Iflff ^W • fiHI I im ail * ^p -■Wwi warn us wm IIP i IIP iff P : * i^ io ife nn V///s////JZ> mm, wm. a s in © ^ WHITE. While to play and mate in three moves. No. 384. By S. H. Thomas. IIP ■ jfB. HH fill ,1111 Ml a mm: rf i mm mm mM fjp "White to play and compel Black to mate Mm in eight mores. No. 385. By A. Townsend. w H Wmm I I ■Pi WW n§ wml , mm mm * m mi i m,B...wM L;,..J i "m H wm. I m w w mm H I^J White to play and mate in three moves. No. 386. By A. Townsend. §M, mm. m S%^^% %%%%%%>. vwMM 9m. Lifi ^P ^^ ^^ ^P "White to play and mate in three moves. Conditions — That the White King is to be in check, bat able to escape by any of the three methods, viz. interpasing, capturing tha checking piece, or moving out of check. 25 No. 387. By A. Townsend. mm warn Ws fc « i i m m ■f ■ ■ ■ .as, I * ^Si I a ||||§ *^ Jul wm%> White to play and mate in four moves. No. 388. By A.. Townsend. «2 ■ Mi • ess-, WWM ^a W°W //////////A Jiil if IB a , , ist 1: P Bl ...mm. mm, hi ^/mgummwm mm WB% W^M ?mm White to play and compel Black to mate him in six moves. No. 389. /Li/u*(- crvtcMtj By A. Townsend. warn nn 'Wm\ w. WMM *■ HWH J ■ ■ pi * i i mm *;wm.. Wb, mm, wm m m mJ m White to play and mate in fourteen moves. No. 390. By A. Townsend. X & P IP ! wM,„ ML fc;„ m ww MM ww ™ " HI z — ' vh»0/fi ^feSal mmw. WHte to play and compel Black to mate him in fifteen moves. For the first correct Solution of this fine Problem a prize was cffjred by the Composer, but no solution was sent in. No. 391. By G. J. Slater. BLACK. »W ess WMk HP m -mm ~'wm£ /jm ' 41 wF^rn^ H wm& „,„psk TpFW p J ill 111 ill ip §§§ g pi ■% m wm. White to play and mate in four moves. No. 392. By G. J. Slater. I ^] it! ■ A ■ " iMk M y Wm%L ^88%^ ** Warn, ^§ Ifejl \J~,....MMw. White to play and mate in three moves. No. 393. By G. J. Slater. BLACK. '■W, m J^ m JB\-& — «l HP » ^m % 1 ggug B8 ^ I k mm mm i« « an i «f ~wm ■I * Ci HI i ^ ^.„.™..^iii i ■"■ in m ■ . . 777777. 49///////A <%//. H WHITE. White to play and mate in four moves. No. 394. By G. J. Slater. WW- 'WM0, BH mm?, WHOM ■ Bill II if 19 B Bi my//////:, / /V//////'//, ■r~B~ 0TB m ■ ■ 1 ^ HSI ^^^ up. White to play and mate hi five moves. No. 395. By G. J. Slater. m. mm. wmk Hi m. - wm». wwm mm. ffl W£ mm m mm mm 'T-m^?~-- ^-^ye^ "»* mm mm. W. - if^ill^^ "up* "White to play and compel Black to mate him. in. six moves. No. 396. By Edwin Dyson. White to play and mate in three moves. No. 397. By C. W. M. Dale. YW3 WM P ■—■L ' ■ m AA gilf || Ml i i ffl m Hr-- ^ i Qfe"< Hi iff &„„„„„!».„; B p W"g^p" — CtHl; 0" ! ' i 1811 White to play and mate in three moves. No. 398. By Fked. Thompson. i/' K WMMWM,. ■ HI ■ 111 HP mm, s&f fli In m ■ Hi mm. Wm-*~ White to play and mate in three moves. No. 399. By Feed. Thompson. # IP il ■*J1 1. .11 i v* <%%% mm, Us m B ^ ^» ^H M "White to play and mate in three mores. No. 400. By Feed. Thompson. ""wI /A I 1 I ■I mm m+* C\lll ? ^ i ill ■ ■ «... Ill III (Hi Jwl£jOL^-jl §mm—_Jmsm WHITE. White to play and mate in five moves. No. 401. By J. Paul Tayloe. mm m Wffl% Ww/ mm mm> 1 * '"prp™ - Y mk^mk r mm, mm. mW: WHITE. White to play and mate in two moves. No. 403. By J. P. Taylor. m* W/ZT??. H ^ .mm. I mm .wm 'warn w/m, W/Wi W2M lal H m m « i m mm,, wm mm_ rJ\ mm, wSxt %m f*i ■iijsJw WHITE. White to play and mate in two moves. 26 No. 403. By J. P. Taylor. wm m wm ww, I ww, 111 ww, wm wl a. 'S/, '4s/.>,&& =,.;^ m i III M^wm WW, WW, White to play and mate in two move3. No. 404. By W. Greenwood. **l - # ^l:^-WW Mstjijl «il W^ ^ww, Jill ^^ iiip ^ H. Jill m White to play and mate in four moves. No. 405. By W. Greenwood. ■";■/ i HP lilt. VMifPA W&jsysi fliflL 11 dSM wMfr,.,,,, ISI a fM, mm. w §H^ ?A'< «T J»i MH MSI u p HP iff HP —■^JsUfL-^ WHITE. White to play and mate in four moves. No. 406. By W. Greenwood. , ■ ■ if W ilf •' if 1 pI m -wm § 7 A m^\- m i t^MA nn ;. K'/zy//////. mm WHITE. White to play and mate in four moves. No. 408. By Rev. A. Cyril Pearson. WMfr W$$& WM I ill i is i <\ii White to play and mate in three moves. No. 409. By R. W. Johnson. BLACK. wm. / jm% _t™ mm. v/^y^osy ± 'fiy^z&s f\ W/, Is/ PI A @#jfi W/////Z& ////////////. t2/////M, / mmy/a §llf„ ilii Jill ,„„0m. I n«i mm, mml " iy! r- II M^ HSB 8BI ^§i ■ lH: ■ ■WHITE. White to play and mate in four moves. No. 410. By R. W. Johnson. BLACK. ' "**. '1 Jk H-'"^ &«r# ^P f€m; » ? I ... nn ... till J*. Mi aril i \ - 3 WW/ IS 36m Jtm t* 'Wm mm i ■ in ft * »AW IPil IktW id §a jM IP pni White to play and mate in four moves. No. 411. By C. Callander. K y< ^^ 'W^ '^^ & WMfi .„,* JiHt x fell w// mw/ wm, wm wm aSI White to play and mate in sis moves. No. 412. By C. Callander. fcH Va ... ?%"/ IP i '■/.T7T.,,. ',.. ■ A ■ i J Wl UP fpHI km,: fiBXi '■ (£» ; ■ ■&■ H ■Y///.// /.'. ty/////////. OY//A////>. :-■- -,-*/. 0£- &% w mm q wm m ; . II ;: ,, "White to play and mate in four moves. No. 413. By C. M. Baxter. ISP 'HP m mm, * §§11 *■« -J^ wm ****** U'/M////'. W////M& """■■ 5g%g? ■ #IH ^^ _^^ White to play and mate in three moves. No. 414. By C. M. Baxter. ^ H bH -si 111 ifti iHI iiifl rmi w i r mm « ^» is ip^ 1 I i 111 !§P i w,„ im WHITE. White to play and mate in four moves. No. 415. By W. Furnival. A i mss mm .!■ IB ^P HP. IV : P 5 HP 6 H1F ^Itl i^ifi jh iBL -^ !«'• mihmw. mm m p & ■ p'T'p H §mi ippf rat 111 Hi M iM White to play and mate in four moves. No. 416. By W. Weatherston. ir^nA. i%HI%. b: : i wm.>B~wm. tin tt ^R B i ^JN iff Hfl §i§ m^wk Jm mm Jm mm %y\.^§h> \ Q uw mm, , mm j** mw.,^~ i8I Wm ft ill * iMjlOEj(-i WHIIE. White to play and mate in two moves. No. 417. By E. Ormond. IMP • «^ HP i ? wkjks mm @ i* 111 j til ■ White to play and mate in four moves. No. 418. By R. Ormond. m %W™W"fi W7V n nyi *„i§ii - %"# - ii^ iHf ffl mA ■Z: H 'Warn „, fflSZ:.,, w. HP PPP is mk m if*' 1 WHITE. White to play and mate in three moves. 27 No. 419. By W. C. Spens. ^ maw. ■fflawM. x. imi,m Wt. „ 'tCWA fH mm wfflm fe=s??i ''' i '' M Ill H H. til ill i White to play and mate in four moves. No. 420. By W. Coates. 'WM W -a ■ Hr mm. i X "W*" ,W™W m wmT m m sfeggp^, ■/ i 1h illi IP3» « 4 ■ i 11. >A 111 A ■ WHITE. White to play and mate in three moves. No. 426. By J. Menzies. '/////sss/A : mm mm. || 1 »« 'mm £ fBllP^i^ Warn' '''-mm. 4 wr~ mm | _^ | jy"i |r JBai H ill I Us i^M s^ ^H sW"" White to play and mate in four moves. No. 427. By F. W. Lord. mm mm mA^^JLi iPj% HP 1111 k fill bI H MAm 'WZM 'W$y/ Pa# ^^^ H 4M JM, , — ^ WHITE. "White to play and mate in three moves. No. 428. By F. W. Lord. HHP PUP W%m W^m mm ^t ^il * khb 'mm. fSi * 111 £ Iff Pi IP A IP i ....J« fgBk m '^BtM mm'JmAfmi^m WHITE. "White to play and mate in four moves. No. 429. By J. H. Finlinson. m wm 4% 'W; £*■ ■ WW W S88 ly//////M ■ f m&,- i m WHITE. White to play and mate in four moves. No. 430. By J. H. Finlinson. Sffil 'W/Mfi, Vv7i77T& is; PI WM mm iH r ^., r ,. |f: £ 1(1 mm Warn vow/M vSzEsa ip ph »**» » -^ ill MM; q Hm w White to play and mate in two moye8. No. 431. By J. H. Finlinson. Wm wim WM ^P tmmit * w r« W/W/> Ji, 'W%W/ S3 ^#2^ a fiJt§l# jy fi n £SS5>;3rf - wl - W//Wa vmmA #W?M ■ ,.w//////i„ ^ == '/////,/////. ;S : ^s ^« Si si g Ai WHITE. White to play and mate in three moves. No. 432. By J. H. Finlinson. mp up "1 i 'ZZZV^ZZZ- WaaSli 1 IHM?- SP^ iSl, feU i« «# zzzzzvzyz/ 'wm h wm A 'JtU i; "White to play and mate in four moves. No. 433. By J. H. Finlinson. IIP wJw ■^^'^^"w^ m r wm ' m HH mm mm. „^^.,f*r.M fa ;w m i m m m fyy, -//////M//. mm/////, feai "*■ ■ > y/y//'///^ % White to play and mate in three moves. No. 434. By J. H. Finlinson. mf 'W/M WyWk WyW>. W \ mm. mm. a hA^L"I. up Li #8# m " Ik JSBM.^.JKwi IB H : "White to play and mate in three moves. 28 No. 435. By A. E. Studd. W0M Ws 1 HH Wm mm W% #k mm mm mm mak ** mm mm wim\ mW Wm mff\ -«m- fci White to play and mate in three moves. \ No. 436. By A. E. Studd. jh wm ?ip «p™ y Jm. JET Wm, i wm * ; f# mm $ ^» M^Jj^m^mmjL M mJzh. mm,,^ mm. mm Jm\ wm * & m mm m %m mU m m mm 1 WHITE. White to play and mate in three moves. No. 437. By A. E. Studd. §111 ilHl A H^ ■ u H 4L_» IJI.J1 if S ,J«|ii ^ WfM% -WSW; WiW% 'w. r\ W/. Y *^ %M/»A y ^-* mMj. r WHITE. White to play and mate in two moves. / -7' No. 438. Studd's Challenge. By A.. E. Studd. 1HP HI ..— ..: i %/////A. #^^i ill 1 \^J^^ W - " ni i ? Z: VmwXA White to play and compel Black to mate him in eleven moves. No. 439. By Colour Sergt. H. Woods. ■■ ....11 mm I www, ' 'W4W>, V'V'W/f'. ft m im ,if ill S 'WW/?', <* ^ »* WWW- WWW HUP Wv, wJA — M^. w// „jmm. i c& Ml "White to play and mate in three mores. No. 440. By Colour Sergt. H. Woods. HI M 1 bL. $\§BP wm^wM. £bl III ^111 ft fH W X WW??/. . ^ -//■■ 1 fe * ' B Bi i 111 ■■ El WHITE. White to play and mate in three moves. No. 441. By J. A. Miles. 'Ok "7\Wm mm mm.., mw^. mm WW, WW* a m mt m w ==■ WW, White to play and mate in three moves. No. 442. By J. A. Miles. •*. : '"■■:■■ \ mm mm m if *m ta ' 111 Hi 111 WM #%%%% ^ WHITE. White to play and mate in four moves. No. 443. By J. A. Miles. eg n wm mm Www WHITE. White to play and mate in three moves. No. 444. By J. A. Miles. :WB, WM, WW, mm Bf B 111 i s. 1 Y/y//////:., f/A 'Jfimw,,. 'HP Pi- ; ill n »j iiM ,;^-^ vim *—m* WZzM p B 181 White to play and mate in four moves. No. 445. By J. A. Miles. M„^,dMm£*^ wwA .. iH fc n " mm, 'wm ■ Si WW "^ ' w 4M.1 r r White to play and mate in five moves. No. 446. By J. A. Miles. Hg p"p m 9W jam m wb, m?///////., HI HH. b White to play and mate in three moves. No. 447. By J. A. Miles. 61* „ m M M ' "SW ' "fMTw- a m wm mm *» mm § W... Km mi Wm mm White to play and .mate in. four moves. No. 448. By J. A. Miles. ?;;^=y ■p pp if i ^« »s 3 ^ r ; A"; mm mm W3 w Mil LSI m?dZ. ^ ^51 ,,,, iwm, WM 9m w*W, wkT m m J WQW 46m §Bm mm §§i a jpni White to play and mate with his King's Knight's Pawn (gC) in twelve moves, without taking the Black Queen or compelling her to move. CHAPTER III. IFOIRIEIGKET COMPOSBES : European and Indian Noa. 449 to 640. American and Australian. . Nos. 641 to 720. Addenda, Various Nos. 721 to 736. Dedicated to CHAELES A. GILBERG, of New York. BY HEINRICH F. L. MEYER. white to play and mate in twenty moves. Edle Kunst, die sich bewahret, Wachst im Stillsten unvermerkt, Bis, gesteigert und bejahret, Sie des Freundes Fest verstarkt. Goetue's Artist's Song. Wahrheit sei im Spiel des Schonen. Hermann Lehnek. No. 449. By Julius Mendheim of Berlin. Wmm wmM WM& wwm & HI mm iH 111 Mm^mm m mJr m wM^ *ir W" 3 ^^^ WHITE. White to play and mate in seven moves. No. 450. By Mendheim. 4 4 Hi l'i ;: i i III III i HIP d P 4wtA ■ H ■ ». White to play and mate in eight moves. 29 No. 451. By Mendheim. wm TmvsT^m iWw WB B HI mm mm mi tzmfflM, mfiwwito, vtvpTZMt. M mi ■■■MJr ^ mm &A v/ *MM. White to play and mate in seven moves. No. 453. By Mendheim. m§ w /M mim I n. §3 mi i W *i # #1 w...~M.. ■B i.. ^„ asm, <2 ■■■ m ^ tiiii WHITE. White to play and mate in four moves. No. 453. By Mendheim. mm///, '==' mm// *==> ssbJbsss , mm wm & p ill wmp> wM„ ^^ f HUPP K ikr •§ 6 fill w, ..ill , Wm3.'0m% w, 'WW' mm P» r; i till Ml if White will give checkmate -with his King's Bishop (a2) in twelve moves, without taking the Black Queen. No. 454. By Mendheim. — Wwk !^§y^ Pi BMiP m///y/.„„ ///////m.„„ rr ,>. 'HPIP IIP Willi #£# §yy§ ill to =tH l»l fOl ^ IP 1 MJBLjil HI mm, W4WA ££ ( W4W, V/6WA a 22^^ fell fi susses 'WM/."^... v ^^M.-^^W/m»- \ WmmWfM mm mm Worn m wmW/ WMW/ White engageB to mate with his King's Bishop (g2) in ten moves, without taking the Black Bishop at hi. No. 455. By Mendheim. BLACK. ML wm ■ HP My%%9%, 't/MS/M ,„ty//////M: ■ Kl H II H „.JH „„Miy/////. rr v//M/,..., wsiWt. WW- B HM Is •Ml I 4 ^^"i"^ iilj?k m vm JRm vm m m fit l White to mate with the Knight at King's Hook's 8th (h.8) in eight moves. No. 456. By Mendheim. BLACK . i 1JMM *■» nasi . ^^ mm f s| ill i n &s^ ZZWM„, M?m»t. Wmm HI PI WB v//',<»< ■ W '■ mB, w -mm jmfr.,j^ ?M§, g™p mT A W m "White to play and mate in three moves. No. 464. By Beede. BLACK. MM I %B, Wm H HI ■ i ; '/////.//«■. ■%%%?%. Wzam %&&? . mm §§p mm Tn p, mm ■ WHITE. White to play and mate in three moves. No. 465. By Brede. W, 'm ^K -"* *" iiit ^ m m -■mm ■ i' V/y^jwww.,,— ««. / warn Wm, mm „,„„ mm. WHITE. White to play and mate in four moves. No. 466. By Brede. ™9a %%W%. .... WW/,,, m m mm JBm WAkmm wm ■mm Pi W WwJ I ,///M w I^i '■www I White to play and mate in four move3. 30 No. 467. By Brede. <&*WM WB HP WB wffl y/Wwi Wzmfc Wmy// imEki mm, .mm ,„i» w i« iJL ; pa ^1 „ V.W5A M «l #M# #111 i lis II wm, WM * '//////////. White to play and mate in four moves. No. 468. By Brede. 1 f» WS. w, ww//// w///////// m ^P ^P li &'1 Hi" V//////M m iH pi WmfM,,,, HHH illl 1^ i WHITE. White to play and mate in five moves. No. 469. By Brede. 'mm wm Jk mm- : <&4 — I: :; in W 1 * mi m !#i 4 i am el 1^ * €M W ds PIP Pii i ■i H H A M «i. ^ f A § (# m m&m Ill Ill jib I Slip White to play and mate in five nioves. No. 470. By Brede. yy/z/s.'/,//, 1 n. til .'//////////. V/////////A 9. " ill w HP in ■///////M :■ Bl Wixtm i HLJHI « fi Wm 111 p ^^i WHITE. White to play and mate in five moves. No. 471. By Brede. Iff """ J wm A wwi" / "" A WM wt mm Wm '^■„,,„„„P^K„„„„f^A M ^* $0m'%0' ' 0/// a §^j ^mmj^'" W, . iJH White to play and mate in six moves. No. 472. By Brede, HUPP . wmk Wim WM White to pla}' and mate in four moves. No. 480. JtL By D'ORvitTE. Mi %>./■ ■V- ** Wi ~^— '%/W/m -ZZZTA m ■ m ""iL %"'::". m wKmF m m WB m m JMRTmr Wm% HP WM U wW/?/ wm% Wtw WM Avt 4 wM. v ^ mm w 'mm Jmm iHf mm White to play and mate in four moves. No. 481. By D'Orville. ^i? mm 111 !«l « HI A 111 iM IP 4m, i ^^ ««„ ^jmm^. White to play and mate in four moves. No. 482. By D'Orville. HIP 1 illl '•,/?$?. ' $/////$. %////;/,//. 1 ^ IjI; ■ ^ "si - ? J il VvMrf/. WftMi 'J?/////////, WWi 'WW m..B..3Mm. 'm/. % fm fm P mm m, m mm,. White to play and mate in five moves. 31 No. 483. By D'Orville. ""S* .////My///?.,, % %/////////.„ ,„,(»«! m m Pi m «p mm i i mm wm -V0S/0A V/SSS/SA m m m. White to play and mate in six moves. No. 484. By D'Orville. 1 MM i ...iff '^ wSb. WMk, HSH i 4 HI Hi i " W Iff' al flH ■ » ■ ■ White to play and mate in six moves. No. 485. By D'Orville. S/////S.'///. wM mm WM6 m y. ■ B "**■ wm, pa §m mm ^ air a 11 ..-.^ •==!=» %JS^ %///M r ■wyV i in imp A mni .■W/////ZV. ■white. White to play and mate in six moves. No. 486. By D'Orville. w 'fgf"" A 'WB wM^4Kk m mm mm m w ■ §111 lilii *=LiMi, JO ill ill 151 f if£! HfiH H 48HI1 "//Mm W//////A m *■ i- White to play and mate with his Pawn in ten moves. No. 487. By J. E. Kies of Stuttgardt. -ill 111 S y /i&ay fr v;.?'??', i''.'/" / '"^ ■,-■■."■-, "^ ^^ ■ i ill iii I II mm '.■/.-' wm wW mm, mm mm, mm x W'"''y *^WW4 1 ■ ■ ■ ■ ■ ...... /////. %/////!%; White to play and mate in four moves. No. 488. By J. E. Bies. P thHbG" W El ^--w—W™ m mm w""" A w Wi Y , ■ 11 i m » ^P c fif i«„„„ wm a£s, ^m I. WHITE. White to play and mate in five moves. No. 490. By J. E. Ries. : W% J ,f#l |fi . s=g S_- #2_K I1I§ V/////M ~'— i HlH iHI ^3 '-'/A '; A < ^^...S„^H„ w i L 11 l El 1 IlH i Ws> wm wmk WHITE. White to play and mate in six moves. No. 491. By J. E. Kies. !§■ * wmi. Wm„, 4m '/'/■'/Kg?/ W m mm. ml. ™ §BA mW,„„,„„„ -mm 'mm mm Ki mm. m 4m, ■WmF m m p w mm w wmf, White to play and mate in four moves. No. 492. By J. E. Kies. a ww, mm, Wkw w Jm '■/,/'■■ , mm wm i in m m^%. 'Jmm. wmm . W:,//s-6. ■ i WM HIP A ; X. 1 lilAi ■ White to play and mate in five moves. No. 497. By Kieseritzkij. <0^A w,/A//, %7^A ^^ Wm& < W3 ww% '%%wi wzm M M til M mm LssJ^L^j^m ms& i mm 0jy& m J0i mm , H wm WxaM WtWfi. WHITE. White to play and mate in sis moves. No. 498. By Kieseritzkij. _ HffC I i t i ;• gj I mm,.^,&A Jm^>, y/// jhHL I i 8 § ill a "job , m White to play and mate in four moves. 32 No. 499. By J. Kling. BLACK. w///// 3 WM WM ®® ™p^" "White to play and mate in four moves. No. 502. By Kling. ■^J^-HpA' I 1 illl i WW WW?' m -wmf m m ...WW, W WW, Li WW, WHITE. White to play and mate in five moves. No. 503. By Rling. ^mm wi mi mm ! mm ~mfr^ fta pIx JliJt|S| JiH Jin White to play and mate in five moves. No. 504. By Kling. ^ ,,,,,?i2^&.„„,, r ,„JZW%%y, W%0 m nm mm #282? '4% Wi 'WfflMk. Y <^yM, y #mwM< mJ**WM.. WWa l.. T B ■«■ White to play and mate in four moves. No. 505. By Kling. «-" W ^_» 4. W£$ * Hip fiy * ill JHHftJJW White to play and mate in four moves. No. 506. By B. Horwitz. WHITE. White to play and mate in five moves. No. 507. By Hokwitz. m m m m^ mm V/////////. «ii» m ¥M, I* I 4M. mm... PW WsM Wfo m m w%m .11111 mtvA mW/, <1 wM mm, WHITE. White to play and mate in four moves. No. 508. By Hoewitz. >=■ , m -^Jf^- m w mmi / ^4&m^.,.mm. ■mwr-a m mmF m vmm Km "■*■ White to play and win. No. 509. By Kling and Horwitz. IK I ,,,,„,,wm mm, W//////M wmm, 'Wm WM W mm, "-'; , Wff ■ WMi, t ^fH8%[, '/?////%#//,, %%W//A ■P W ifi f^-jjjj** m§A mm w. mm, WHITE. White to move and -win. No. 510. By Kling and Horwitz. WKk_ m w/M. mm p mm, ™,±±.J mm m w j0k m jm u m wmf m m ...m mm, mm, m 'W§ WHITE. White to move and win. No. 611. By Kling and Hoewitz. " **?! MWi. wm„ 7?'"'rz-. m -mi? m mgF m ma^ fi irt WHITE. "Wliite to move and win. No. 512. By Kling and Hoewitz. Vs. ■P I ^ ^ «11 m mK J am ■■"■^ M 'wmf m m m. Warn p fa "PPf 4 w$ 1 4 '///////Mi,. m mii W//;/////j WHITE. White to move and win. No. 513. By Kling and Horwitz. ; m * i Bl s. IIJI^Hi ^tiBk W"% 4k m iM^ f im m mm mm mm WHITE. "White to move and win. No. 514. By Kling and Horwitz. WHITE. White to move and win. 33 No. 515. By A. Andeessen, of Breslatj. Wm ■ Hi i m *Jr- m wJ m m^ m ^ mm mm. White to play and mate in three moves. No. 516. By A. Andeessen. fill w |Bi JI H lii ill m -,J^^ mm w. W//s////yyh H warn, iill % WHITE. White to play and mate in three moves. No. 517. By A. Anderssen. ^ , , BLACK. illl 9 §111 iHI iHI X HH mm. 'W!W;. W/Mb ■ x 1 1.... 8 i * fat W&3 Wtw WMfr w mM,., •M4 Y%V///////. m:zz, m TVHITE. White to play and mate in three moves. No. 518. By A. Anderssen. m> ■ H ii ii mil mi y/ , w mM r mm, 1 ■ m m ■ mm _JBL White to play and mate in four moves. No. 519. By A. Anderssen. g8*gi 1 % ill 11 II fc„ - ' J8jL pi pqJ i§H§ H w 'mm, mm. m. w mm ..mm. * II3J b, mwi.A..dM MM White to play and mate in ^ve moves. No. 521. By A. Anderssen. M ] WLifo\ wm*,,.,wm , mm ; M_4^M» 111 % Q I '/y///// - lr~.i&^ ill ill ill i ill im # m *jra». wm,^ mm. WHITE. White to play and mate in five moves. No. 522. By A. Anderssen. 3 '%%%%?% ii iff 1 J ./' II. „ if i_ ju *jB M H *J8w. Hi ■■ *WM. A W\ S H "^ HH * SI White to play and mate in five moves. No. 523. By A. Anderssen. ■ mm,. lb flJLpJ#fli w HP ■ HP m w J ii . fc^ S JT^%^ White to play and mate in five moves. No. 524. By A. Anderssen. W$M WW, 'MM mm mm mm % V/////M HIH HI imm mm, I III g| [jj wm. j White to play and mate in five moves. No. 525. By A. Andeessen. JS& is Ill iS tilll HHH HHi ^ ,„,, ^« mm... in. H 8 i 4 HP '_ ; »/?,,'~^'%MY/%i, i ...m y/ w, WHITE. White to play and mate in six moves. No. 526. By A. Anderssen. ,,,.,. i ^^V^ « mi ^ mem Wa Jill jillL. m m White to play and mate in eight move3. See No. 98, by Bone. No. 527. By Conrad Bayer, of Olmutz. mm JH mam wjm. mm, Wk w, mm' . 1m **, mm m9. m v//y/,7"'. i wm A iifJ mm m 'mm ff" mA III „ . WMM, • PP IIP §1 mi mm WHITE. "White to play and mate in four moves. No. 528. By C. Bayer. BLACK. w^SlfiJIt, m w////m W//////A 1mA mm. mm, Q ifSi mm 'mm , p 'mffl. y//fflyyy% X «? HI ^^ " «§I «1 rn^^wM 4M WBi WHITE. White to play and mate in five moves. No. 529. By C. Bayer. '////'..'.■■//■'..'. ;...-..... ,.. . si;;........: White to play and mate in three moves. No. 532. By C. Bayer. « »s, ^ *«: \ k\4m |K (f / m„„„„„, $&^//Zi. Mi • P w mm 1 HH! HI WHITE. White to play and mate in four moves. No. 533. By C. Bayer. Ill & ill « i i /.•^r^^i—*^ !"->■, ■■■////*/ S^ „ HH AH! BLJBUbii JH « ^ fli ^^ «« HP WZ&'A WHITE. White to play and mate in four moves. No. 534. By C. Bayer. H i HI 111 ■ ■ ■ fMf §K Hi ^ peg M m "«... ~..,.^^/'...tZZ,,w"M'M mr////y//. . "- i ^ i^iM Hi "White to play and mate in five moves. No. 535. By C. Bayer. . '/W% S wuJF m wm* 3 P ^«W rf P rf ^ 41 S im i m mm a €m m i m mk 1 jfSA w jtm fm fa. White to play and mate in four moves. No. 536. By C. Bayer. m HH Hi nm i m 'mm, m nil wmb, m m ■ i. H iiIj5?LilSi ™ v iai - ^W™ ^ P? WHITE. White to play and mate in four moves. No. 537. By C. Bayer. BLACK. hhp ww> www www . ~*i 4////////A WfflMMi VXMZiZ,,, 'warn. mam wwz. % WWW, iSi. i III H i w ■: ■ww ///A mm ^WWWW, WHITE. White to play and mate in four moves. No. 538. By C. Bayer. nsr WW \ \ i % WW i mam w%"" I fit §■ a * 111 wtm wow, „ ^ d whtW W,W: A IP ■ Mi Mm WHITE. White to play and mate in four moves. No. 539. By C. Bayer. HP HP 1 - 1 — m^-M m « m mm. mm w^m , JJBBBSi %?M%% illlllP ^sUP^ Wf WW 1 w tf"mw a wm' i''S iM if ill i ■mm/' A fflffi A " ^fflm* m II P WHITE. White to play and mate in four moves. No. 540. By C. Bayer. ::.'.'.;•>'; 1 m M wd: m i IBS ^ i^j ...M ml mm mi^ m * HH ^.mm. . wsak mk mm, ** y SL\m mm HP mm, m White to play and mate in four moves. No. 541. By C. Bayek. WmW WW's Wb •I.i.i.:.: M . fe : S*/SSSSS/Ai. mm. .,.#S§ mm ill ill ill ii JK '%m. r/ ,.^m^ mm White to play and mate in five moves. No. 542. By C. Bayee. ¥M M msm -mam mk% mWzk m HI m ■ mm §^§»: ml wm mm. Hjl.1 '■:, if W^ — ~™|^ White to play and mate in three moves. No. 543. By C. Bayer. il mi ■ ■ Al m * is at fAl <%* W//////Zfr WHITE. White to play and mate in five moves. No. 544. By C. Bayer. ■I mm. y mm mm, m mm, mm, dj\, i am i wm, mm mm mm i i "White to play and mate in four moves. No. 645. By C. Bayer. *mm *. A WB® A A 'WM i** M mm m i ■ in. HI i III M fl m^Wf^W^W i n nn WHITE. White to play and mate in four moves. No. 546. By C. Bayer. TBI ^ p ^ 2 " w mm W/////A M. ■% mm. HH * ILl -fli i«i *-j- %»*> 1 Hlb ^H e ^» iM White to play and mate in two moves. 35 No. 547. By C. Bayer. .Ill wm. w. ■ ' I y/Jm «-*- Hfedy JmH WM, mm ..wm,.... mm. r ^ mi * is »J, n ; Hi^ HBP <** ^ Oy////S/7Y/. mm wm.,,^www,. mi tm m wm "mm WW%. /\ WWw. WWm. » 8% White to play and mate in three moves. No. 548. By C. Bayee. ft wm, 7/ mm. mi. m:^wm. im, m mm ww, HU, WM,,,*JWWa^,'wS... \ i = ^AJ^WB,, W, '— ' WW?/, BSO WWZ, nm warn, ^ » --:\ ,* m, WM White to play and mate in four moves. No. 549. By C. Bayer. mml » wim mm% f WMfr '%mW W%% wm mm wm% k W$xk k 'mm\ JmAJm ill ^M x 1 S i Is**' £ ■aL i#.. J ^ • H 1 1 V I ^H ; ■=* ilii ! I^Pi IB*' ^ ... . White to play and mate in five moves. No. 550. By C. Bayer. ■ j jUB£i(B = i i||p ilP^^IP > J 'wm J ' ■WW- iA» i ffi Vl pi i s ■ IF mm ' y ' '■ sis 'mm ~« m. Mi "White to play and mate in five moves. No. 551. By Rudolf Willmees of Vienna. WSB PUP Pi S If W/ f/r^ m «l.Jf ■ m w^wm mm - *■ €j i ill wm. WM W9 ■" " i ill gft *«• iiii White to play and mate in four moves. No. 552. By R. Willmees. v: _ '''///a w W III ■ mi m mm i i mm mJ^m mA m my/M ^ ■■■r"W w - White to play and mate in four moves. No. 553. By R. WlLLMERS. HP n ap Hil |§lll 1 : =:. ^■■'■■-y/sJ™^'., ill lil i V////////3. W I*... si WHITE. White to play and mate in four moves. No. 554. By R. WlLLMERS. «... ,* iiii ?.^ .< 1 p . ..mm 'm IP i = r ^ W"" J "' ^ wm.,,,,,,,,,'^M„ HH B mm m mm mm x A wwM mm mm i 111 iBl WHITE. White to play and mate in four moves. No. 555. By K. Willmers, I Mm Wm ife III m WP Hi 111 Hi i n n§ m in iiij ,fc w m ..." m. : 4v///////A P W>/%& V//////M& mmH& |f| IK V///S/SSML .*». i WHITE. "White to play and mate in four moves. No. 556. By R. Willmers. ^;: ■ B ■ I p K^ V/M/Mi WiM W*W, W?M> W&Wa §1 §1 *■ ** pi fil Hi" "^■A^JL** ■^ ■ ^\ m ■ i "White to play and mate in four moves. No. 557. By E. Willmees. JiPPI ww- HI mm mm. ifl m*mm. M ..mm s mm. WSsfc 1W^ m m 1 i mm\ mm WHITE. "White to play and mate in four moves. No. 558. By E. Willmees. 'wsm H h Hi RIaI i ill! P. ill ,iBL 1 §|^^ ^^^ ^5%%% #1 Jfeff WHITE. White to play and mate in four moves. No. 559. By E. Willmees. V///A Z'/A **.m H " J ^mnsd"*' m^m, mAM. JBf all fijf WHITE. White to play and mate in four moves. No. 560. By Ivan Kos of Vienna. */ tSS 'SSS//s 4rJBm. mm i ff/SSS/SS/M M mJ mm mm mm g- ^ ^gSgifej MM%M WHITE. White to play and mate in five moves. No. 561. By Adolf Rothmaler, of Nordhausen. m MdH mm wm wmrsx'i ■ "™J Wmw. . V~M- ■P — * A HIM ■ I ^m]j B^ i &A/MW// ^/79A// ■////'' ■ ^K 'S3!' ^^ ■yWsmM %ym»y%, WxWZ w/mmw. HI tJlli :. e3 WkW, mm ■ iff HP ill 9 §o iw is *%HHI %////////>. m mJ m '-mJ m ' m WHITE. White to play and mate in five moves. No. 565. By Anton Novotny. JBL ..PH. WMi m jBSl W/////y/A /$y////////j ///Zv7?///, wwg. v-^ wm -,i ■ m wm mm ip WHITE. White to play and mate in four moves. No. 566. By H. A. Stavenuter. "%Mw/'"' ip^p ft. mm i m HUJi ■ Hi- A Hi JLra H Hi I V///s /.-//, A '///s/s/s/// ■I ■ ■ ^w ' ai mm mm^ mm m . 1 fil ^^ p»~ '»» «p' WW/, tpMmyS,. «§ White to play and mate in three moves. No. 567. By J. G. Schultz. mm ww, ww. HI HP ■ i ill Si liL WW WMfr W9 mm Wm w i m * ■■ '/■Tip'!! HH * la Q HH,. ^ M Wf M M il . .. ^SJ H IO. WW mm White to play and mate in three moves. No. 568. By P. Klett. BLACK. MP HP SI ■ w///////,r"'"'\M ww, ww,^ ww. i Wi I P 1 £ g -sir 'mm, 1. 111 lira fii "-Sip m ^f M - w ,,,JM Jm... w^wm White to play and mate in three moves. No. 569. By P. Klett. Ill Ili m HHi.. mm. PUP Wm fm wm % M. w #§H1 x HHi i§ i — 1 mw/%: I ^ ^§•1 I \£hw wm^ mm mm mm. White to play and mate in four moves. No. 370. By P. Klett. H Wl 1 * III ° IF I Ill i ^/wm * ^^ ■%y^%a r %?////////k, .......mmM, «s§ ...:m ■P " il nn White to play and mate in four moves. No. 571. By S. A. Sorensen. I B Wm. m wm II ■ • 'mm mi i iWW> 'WW>, WWi W% tim .mm mm,.... Jm Pif WM i, s HU J ^p B'I'B ■" «K. M w"'"" /M WM A *W"' M M M ii wmmm i] gj Wi m ,„„„, WM hhp M' "White to play and mate in five moves. No. 575. By J. Plachtjtta. «» wm 'wm. i -mm v&m m m fr^' 9//// #JH„ ¥ \J w f / ''mJ m 'm i • » i» White to play and mate in three move3. No. 576. By J. Plachtjtta. '-:'/, m», „,„„,„ W//////M, © tm 'W.,^. mm «i. '■W"W '%W' / /%' "/■''jffl?/ '■%%■ nm lui ■ 61 M HI .„; White to play and mate in four moves. No. 577. By H. F. L, Meyer. HP HP H P m Hi . , Hi r „ Hi„ H r „ IB HiJi B ^^ ^^ ^^ n^f • *l 7 H ■ H ill §=.. ill in n ^» WHITE. White to play and mate in four moves. No. 578. By H. F. L. Meyer. I WM 111. 9,,,„„wm \ wm y mM mm i mmi*zMm. m H ^ «lf JH1 WHITE. "White to play and mate in three moves. 37 No. 579. By H. F. L. Meyer. WW w. jm ■ ■ m m w: y/ m wsm mm ; m m il m mm ■ A 1 White to play and mate in five moves. No. 580. By H. F. L. Meyer. W,,„„ „jll^ Jill vs5tJ™^'» mk wk ■ ft ■ g L i MP tt Bl * «^ mi im - ill «i m 'w%m} WW§ HP^ ° w if §1111 Hm wzz wM,.. W WHITE. White to play and mate in five moves. No. 581. By H. F. L. Meyer. W. %Si mm m a: ""K~ww4 HIP iPiP 5 X ^^P A ^^# * PI Pllf m*Jm£A White to play and mate in five moves. No. 582. By H. F. L. Meyer. 1 m , WB, mm 4m, mm, ...:,; llP^iii W. mm. mm. ^ Si vkMh mm ww IP i ill W P& ; 'i JOjl IS. m w//////m. White to play and mate in five moves. No. 583. By H. F. L. Meyer. m m m ■i i Bl ..mw/'M...^.... W//////A 1 i Pi ///////////a &J *■ 4M>, mm t mm iCl iSJ m H ' y 4£A ^^mk, fa^ 'WW* m ■M>////#M , 5* IS in PL,, .'//////Mb iai ■§■ is w ■ ■ €#J nn i« nn ■r Hi ii IS mi Hi m! WHITE. White to play and mate in four moves. No. 588. By Kohtz and Kockelkorn. iH w/////.'/^. WWi & mm Mi HJ^W JIJ ml ■ * CJ §■ H mi ill ','/<>, .-■ ■ *■ *%^ K Km 1 'ft—///: w, Wm m 1- PS ^^^gjgjg^^^jgjjg^^^^^ ^ L White to play and mate in five moves. No. 589. By Kohtz and Kockelkorn. HP - mm mm mm I ill '"■;•' ; < * :.^; * t« i & I* ' ._ ' 4m fij ci IP Plf ~§r White to play and mate in five moves. No. 590. By Kohtz and Kockelkorn. wf WWM^'W" ~~' ifli llfl i ill _ ill .jbI h I 1 ^ I il i fa i ■ if" 'mm u Ill White to play and mate in four moves. No. 591. By Kohtz and Kockelkoen. WM. j*v WSM %msm — mm. : I m a IB! A ^^ ■ ■! m wm*m §1 ^» White to play and mate in five mores. No. 592. By Kohtz and Kockelkoen. JM ■ fi ^,,.,,,,,..,0^, ills r i si nn a fill PS pal w&m. mm a . i - ^ W/////M& wm, mm,< White to play and mate in five moves. No. 593. By Kohtz and Kockelkorn. ml : m m o n «i i ^'^M 'Wm. 'Sag' SO ms pip Si ;jl; ;.,. 2 '.; as «1 HH ^H WHITE. White to play and mate in four moves. No. 594. By Johann Berger of Graz. WMk^% fe (■L iH Hi . Bl til .,.., ... ■ JiL'#By V///fS77ssA ■-' Wife}}. ;4MM ' BL Hi. HLJH White to play and mate in three moves. 38 No. 595. By J. Beegee. % ftp? i ^^ m m ii ■ iBdill H J is „ ■ HI I Ill I '//////M„ Wi, mi White to play and mate in four moves. No. 596. By J. Beegee. r «v, — I ■ ■ill in lii V///////A., " /a " wm ^mtm. . Hit wm,,£jvLdi ■■ £#J 8H H« ^1 ISpl «L iK ,;< I H HHP P I ^ "White to play and mate in four moves. No. 597. By J. Berger. YW'% i fp mmV^ mm. mm, WW, W a !i W//////Y//. V///////'/i V///S//J/Y/. 'ft W///MW. V^/////////). ?//////////. 'Mm WM mm wm • m isi "" j ■ m^m Y A w 'wW, mm mm mm WHITE. White to play and mate in four moves. No. 598. By J. Berger. BLACK. lit" 1H m ii*.. Ill "■81 m -wJF A m m §^-» ? l» gii urn WHITE. White to play and mate in five moves. No. 599. By J. Berger. BLACK. w£m, wzmm. h. ^HH ill "m k I : ^ mm A W; mm wm ':. if 8 IO •sJJU/I HHF 'HP "" i ft///////?/. White to play and mate in four moves. No. 600. By J. Berger. V i 1,3. .3 M„,„ m ,jmA „„„„ X s : m. ■ m. Jm i H^i H&H n 1 w»k llll , * Jill „,,,,„„ ilK H IHI SB ^i gig White to play and mate in four moves. No. 601. By J. Berger. Ky ™ ' m "wd' w. ■ ill i H H * 111 ■■ ■ R R A ll^Jig^liJLlI ^A^'wM mm. ^.....jBl. WW, W//////A t S 'l£m MM m. kZ%^3. White to play and mate in four moves. No. 602. By J. Berger. BLACK." J"LPLJPLJ" 4— Aj^LA WW % asmr/, in m White to play and mate in five moves. No. 603. By R. Bkaune. /A w itm mi y?>-/////yy/. j wa 'mm. w/m wm , i «^\H MM 'MM *^^=» V/y/////7//. / M7/X//M -ifT^TPli , «gg% ^i-^ M^ ^ mm, mm. HH E#l ... IS. III m -mm m f% m mm. i s White to play and mate in three moves. No. 604. By E. Brauke. sT« — pTr _ r J« -JBBL >; 's/s/ss/s/A mm tfH "m^r&m. ■?' mm m : Hill wmi IS^I kW% HP i v H III i ■ a ■ nm lllJl iUfa ^^ ^ JgHi. § ^.**??T ■x^^i. im%. White to play and mate in four moves. No. 606. By J. MlNCKWITZ of Leipsic. 111 ^wi iO ill ■i Ml Pff mm Wm ■-pJM^M. wm& 1 MBH H Wfr WW/- i»-///////^ WWyyV/A White to play and mate in three moves. No. 607. By J. Minckwitz. my/./////. m WW/ "■ *mr$ ;#! 11 B ft H S 1 is iWJ Wwm m. ^■mm, mm... H Wmk /Wk% White to play and mate in four moves. No. 608. By J. Minckwitz. ■AAa — 1 fij J» J|| Jfai. iHH JUl r#$l W/////A , „„„„^^« ,,„,,,,,WWw/, "«H«45W , «S: Eli ■ IHIiviM * 1 fll .»,; yJ~'y/mm^ //////A wm*. m White to play and mate in four moves. No. 609. By Rosa Fessler of Vienna. ...m 'mm I ill III iil H i .3, §M * ^m\^\ HH fet. « * « *Z-J i iil w lfil m I -_— « m w J V////////A White to play and mate in three moves. No. 610. By Bela Szerenyi of Pesth. ,„.^» ^»__^» ipm 4mi ' * A 'wm,.,x f'.m W/. 4qA wm% %Q)A White to play and mate in three moves. 39 No. 615. By Franz Della Toeee. ■■ m "" \ 'WW, m wk ww%, * Jilt. 'mm : m% A ww wm f^> %z£%g2i «^ S3 =* WW//, , v^M„,,,,,,,w///////A m iwfli mwl k wm *** mm fm,*jww ■ mm iszJU., %5zmM. . „ %BiJ 'Wwl. V%77T7s//S. ws/ss/jm. 7y L, ,,,} /, VM. WHITE. White to play and mate in four moves. No. 616. By Constantin Schwede. W% ^? 111 ; HI ■ mi l wm, iH ll 1 m IS z ' ''■■/////////%, ^m j & mm vww. % H WW/, "White to play and mate in three moves. No. 617. By A. Baybrsdorfer, mm M 'i mm ■ ""•""Wf wM,„-0M. *. IM I HI i iSi W> V/7?7/77sJ, W& ■ //m m, ifil <§&»: H §1 mm m A White to play and mate in five moves. No. 618. By Oscar G-elbfuhs. £3 *■«■* ~ -*RJPi w,B'wM H pm^tiSm^ WHITE. White to play and mate in five moves. No. 619. By WlLFRlED MOSEE. W, y 67$&7& VOKMtA J^8st« %////////', wwz 'mm 'WW/, _h iii tmw mam w^wi iii u * mm iL,,„, mW>.' s *~m^M. 'mm. W£M i HIP WiM mm m, WLfak 11 V/mi//V/. V77%77??i "WHITE. White to play and mate in four moves. No. 620. By C. Bosch. i r w- v/M. mWk wm m m AMjp— P L5 r lil White to play and mate in four moves. o No. 621. By ^. Blumenthal. mm mm mw _JHI B WM hh III SB <# *1if k Wm J8ji mk^Jm JUJf wlmk wmk UPl „■ flLJa WHITE. White to play and mate in three moves. No. 622. By H. Lehner. ...Hi I! Pil til imm IS i Is M M ■ at. White to play and mate in four moves. No. 623. By Ignace Calvi. f.l im ^m * wB, Is. mm ■^11 B til m W^m. Ww/A mi mm mm & \ «&■ 111 ■ -'^,„„„„0^''.„„„„„ W&W WMfr. W%M>, WW/ ^ 'mm #1111 wwfr. "Wliite to play and mate in four moves. No. 624. By Ignace Calvi. BLACK. 1 mm Q WmZ^Jf^/A „liil i mm ° Hm "1 ■ ■//////■///■ ^^^ . if im 111 WHITE. White to play and mate in four moves. No. 625. By Lamouroux. ^ nit i H m HIP HiP IPailP §HP ^p w.: §h| fill ^ B CSi ili i I „ H_ML H '"Y///ZMi ^,^Sv?i waaam White to play and mate in four moves. No. 626. By Lamouroux. IB mm ^^ |B , wmCwB '§ * ill ill * ill ita §™ hhb ii« i§p ISi IMl^ ?:'' ^ „ 'w/m.^. , warn HI ^^«—J^^'- mW ^^ ! '': / H ° isi i i mm White to play and mate in four moves. 40 No. 627. By Lamouroux. BLACK. f« Hi wm r . Mm.. "mum %r« '' 4M. h WM fa M^mW, WHITE. White to play and mate in five moves. No. 628. By Schrufer. Wml. ip v mi mm Dil ■ WiM. WfM W mm. WM, i I ife^i — mm, . ■ B L VWs/sy/yW. &ssy0yy/w. 'ffi/kb'001' /\ fMt mm mw, mm. $ V/////////ji H IHi wmz. White to play and mate in three moves. No. 629. By L'Anonyme de Lille ( TH. HEELIN. ) BLACK. *m m mm WW ',.,../;..,.'; ! ■ I BB~A\~W WmJ—LWm. m m mr I mm WHITE. White to play and mate in six moves. No. 630. By L'Anonyme de Lille •'•99 %f% m b H7///M-/-. P- : aPV JM Wk IH >■•■■ x im x Pj^ BSE?™ «# !.®; #R# Jul . #^. , ISI . '/)■//////& W//////A mm White to play and mate in sis, moves. No. 638. By A.. F. op Florence. W" JjLJai 4M mm, ■>. /TV///////// A,, .s..Y. m, mm. m WKk x lip 1 i iw ill ; W %% V////S////JI V/777777/A White to play and mate in seven moves. No. 639. By a Shagird, OR NATIVE CHESS PLAYER OF INDIA. BLACK. fo&l fm& fmm. SggM Hi W JF m wm tM%Z%i 'iw/z/z/zZ. HHP ^ m WfJ H mm Hi White to play and mate in three moves. No. 640. By Moonshee Waeis Ali, or India. db i M : £1 1^1 ^H 1 VMM W ,: WzZm Wz^tfz. WzzWz, m 4M, y , mm wM pj mi '////zY/////. / /'//////z//. 'zt—^TTTsz. v//'zW'z '/•///Z/7777A 1 \ i III ■zW W m Either party playing first is to give checkmate in ten moves, the last three of which are to be three successive checks with three Pawns. No. 641. By Theodore Moeeis Brown. JF-wi- WM%- '%%%>- m / mm, m Mk '§§■ ■ " * m. .„ HI 39 Hif 111 111 iuf J f«i| llil llil -Jm£ wMk mm / /////M//, WHITE. White to play and mate in three moves. No. 642. By T. M. Brown. '//////////// Wffiffift y////// / //// a ytffiffiffi Will WM I %■////////. $%Wl_ . HI iff warn , A mtm A ^„ „ #£z5>7 4//////M. w. mm, mm, White to play and mate in four moves. 41 No. 643. By T. M. Brown. HHH *s. Iflll 'WmW it; fw wl-S&k m S 3 ':<■■ m> A m (m i m.^'wM,, wm mm mm* w/m. HHf "^ i'^J mm |a|| White to play and mate in six moves. No. 644. By T. M. Brown. WM^ ill JM iMl mm. mm. lili Kf 1 1 K HI 111 m, H ,„ mmm m n "pi % p m-////m. fy////////}. vi/MMyj, sfeesa* White to play and mate in three moves. No. 645. By T. M. Brown. 'WZM '/M '//, |j|P |f§ip ~"p W////>. r,„„„„„™^*m Wi%m - wk P. WmM /% W, WW W - w Jif[ warn, *. r msm. ,™ mm.'" WM. = : \^1mM.1u^wm w, Hi jb,j wiw, HH I WHITE. White to play and mate in five moves. No. 646. By T. M. Brown. BLACK. JJkJkJ^m~jmm w. mm. mm. mm wjm mk ill HI ftp x y/y//. .....w WB. WLA WM. wm wm. mm w»i i Smrmr White to play and mate in five moves. No. 647. By T. M. Brown. '■?'>w//7/. .. y 'v///mf: /j m wmwm m WBBk m j wm * Ki ■ "'-; * mm mm. §#i J HUC il IBJ White to play and mate in five moves. No. 648. By T. M. Brown. iH WHm y '//////////a #,.* &i wm,^;\ »| als^ '£:////'///,, 'M ■-"•3 •< WW - 'J faXa? ^ I „ffii * wk i H W//////?//. W/////"" ■ Wm~wm mm White to play and mate in three moves. No. 649. By T. M. Bkown. %; : » W.,„, r i« iRH HHI js&jl; mm wqm mi v?S?h&. ifewi #m///,A WHITE. White to play and mate in two moves. No. 651. By T. M. Brown. «§§! W% W ■ m k p ^ & JH lii IP Hi I White to mate in eight moves by discovered check with the King's Bishop (a8) , without moving the Queen's Knight's Pawn No. 652. By T. M. Brown. h! '■y^y/vm '•%%y'W / ', vt'MW/, ill &*■ II 1111 iHl H H "^ ISI KM 111 iM Hil 4. £ *-~«* *2i§, ^/m/Ay/.,,,, tv^^-A w, p m mm. ill • MrMI =i <=&* im nm §b "White compels Black to mate him in four moves. No. 653. By Charles A. Gilberg. wm, Wm wm, M. §1 1 i 1 ^ 'uwA 4^m w, 4m f@1 i A in ms „ W///M. IM.I1 ^ i^ mm Wm, Wm^mmk White to play and mate in four moves. No. 654. By C. A.. GlLBERG. mm wm mm mm JBiJk^JR— mm mm. mm.,. I i mJULM i ._« fSI 8 H ISI u ! 'lip zm%&, »//////%, "White to play and mate in four moves. No. 655. By C. A. Gilberg. M. m ■ v A ^ '^^A...^....WMiZ^.'W^...^...W^.. Hra. jd^^B^jBj* mm ■? tui "White to play and mate in four moves. No. 656. By C. A. Gilberg. ...m mmk°L±\ m wok mm mri wmhm vfewM m^m #iH '- , ^S A If HP wm^^mm^EL WHITE. White to play and mate in three moves. No. 657. By C. A. Gilberg. BLACK. up hp mp w/m. W* ^p iS i""""™' %////////,. ..mam WZM& ■ i 111 ill H. AYhite to play and mate in three moves. No. 658. By C. A. Gilberg. ■ J >ni >r ■iJUi HI ifei y=y ^ Hi *l pi A lilt i * flj Hi S I ■H Hi ; WM- M\ mm// m "White to play and mate in three moves. 42 No. 659. By Samuel Loyd. W////M %. mm I, ..§fc ifl ii iAj J §n^ n iwjs ^», §3 i m mJ^wJ mm White to play and mate in four moves. No. 660. By S. Loyd. "...^LpJ"".. ^mJ m 'm^ m 'm T 1 111 i m^Jmm i dBi^SH^ ilHB ■ H ill & ill White to play and mate in five moves. No. 661. By S. Loyd. p£^l ill wi ■ IIP ■ &■* wow m wm^^^'mm' A^.JxmM,.. mjt wm m w - «■- • Hi White to play and mate in three moves No. 662. By S. Loyd. m J* m J&~J!R* m ^ . mm S 41 4 4tL F^f ^^ 1I1P r-s-*, IP »_^ *— "- 11 a III ■ijuhl ■ i ^ «*■_« 1 H %l My///////. White to play and mate in four moves. ( No. 663. By S. Loyd. 1" %, mmoam B.^WAw. :•: mm wwm mm mm mm 'mm * r fa m wmk 1 " la HH S IHI White to play and mate in three moves. No. 664. By S. Loyd. 'A Ww i mm i fSf mm A US! ^^ pip » iip iip - mm * Hi M . mm » wm, mm. mm, White to play and mate in four moves. No. 665. By S. Loyd. ^^ ^^ IMi g :. m mm m mm... 4 "mr' mm. . laS «1. ^m WBl. mm mm.. i wm, III White to play and mate in four moves. No. 666. By S, Loyd. WH ill ■«r- I "J8 i till ^p PIP i_*4 VST wjm: 1 nil White to play and mate in four moves. No. 667. By S. Loyd. m mm m i * « ill fc m1^ wm.,f^ T mm, «1 §■ lUf fli ■'..'///////A m mm White to play and mate in three moves. No. 668. By S. Loyd. ifii j ■ ■ r&i' 111 ^^ » mm P w. wW, Hi ■ fli *■ Ymm w// ^ m ^ss^. ^m w jm m i H H ^i White to play and mate in three moves. No. 669. By E. B. Cook. PUP ■ mm. ^llli i Y///////A ©a® ^ Wl Hi ^^ H dllL .... ,,,. H III <=* IB fill Mi § ill u I lip A HH HH? ww ; White to play and mate in three moves. No. 670. By E. B. Cook. SI ^ «Bujm^m tii: ;^ $/////////.; 191 6 i Hi Pi m. ifc. ^^g _„m ,.^» W%@% WiM ww%> m*m m '■mm. w&>, mm w mm WHITE. White to play and mate in three moves. No. 671. By E. B. Cook. Jill «J=y mn nm m fll-M-iyi nn iilli* v/i'/ 7 ', yj%>, mi ^-> mm mm o VMM. A '9^%- ° W< pi WfflM//. ■,,,,..,.,., "^^y,,,////// WHITE. White to play and mate in four moves. No. 672. By E. B. Cook. H ^ illi HP iMS ^ wizy/yi »M, % iHH iHH Warn ill III HI PfPP a PW; A W"'% 9 11 '•77— /, ' ~ TT - s^S^L '/.TTTA.'^— ^//mM JlW ppir M 'KM jus JMiiijj^ ^ ~ \mkm- m mm. w, wm White to play and dra-sv. No. 673. By G. N. Cheney. H \M m p HHI Mill nm ^P #JSl HHl "Hi n ftlJJ « es * WW?A m. mm WHITE. "White to play and mate in three moves. No. 674. By G. N. Cheney. WKk 'wm, i is W// W wt ■ ■ p ■ _J(SlJ« JB ^-W - ii iiiii§ inm w*sm, #p^ n« ..M„ jrm jMm White to play and mate in three moves. 43 No. 675. By G. N. Cheney. WM^/ A «■» ; ,.„J11 .^Mj^mm y/soyy, '/////■ "a WmSLWrn ¥9 Wm PI ■ WHITE. "White to play and mate in three moves. No. 676. By G. N. Cheney. wm wm. i S Si i II S (■ JSSSP Pi wAA:m,...jm,... w .m. ^m"™" §1 111 I ..HH IM1„ White to play and mate in four moves. No. 677. By J. A. Potter. BLACK. wm P \ i mm m ^wJ^wmFfwmt mm warn tM?A ma WWk m^%m^:. 111. White to play and mate in three moves. No. 678. By J. A. Potter. % i mi p 111 mm. ®m.k i M «i,= I fi m*jBm w Va*t MM ^ m wx :m m m mm I © White to play and mate in four moves. No. 679. By J. A. Potter. r- III OP i i f ""P^P P "White to play and mate in four moves. No. 680. By J. A, Potter. WMM. ■Mm y/////// J? / '^\ /m mm^h^Sm ,. ., wm?A w¥ jJ^k _ P^f 4 Ml HP i* / /mmZM r mttXMi,. '#'//////%'/., , / if7^w». -fs as". White to play and mate in five moves. No. 681. By G-. E. Carpenter. *m 1 m,Jm* mm ifiJ if 111 ■i WHITE. White to play and mate in four moves. No. 682. By G. E. Carpenter. ...Jl nni mm ill WM mm % m Jxm m mwF M m \ ki- rn 9/, A ..I^,J « White to play and mate in three moves. No. 683. By G. E. Carpenter. mm Mm iP ft » WW wW> ** WwW, Ww ill ■*■ A i SI H Hi * is H m WW> Wffit HHP UPP : PI m ww/t. fill Q WW mm " wmj±jmk. i mm, % vwwz ■ 154 4m, White to play and mate in four moves. No. 684. By G. E. Carpenter. L~t"J .ill ii-; Wwm m.. ■ iP ■——- j lli HHI Y//////////. 't W&M Hi H ■ r ill ill wW^..,. Cj | PLJPL H ■§ i WHITE. White to play and mate in three moves. No. 685. By G. E. Carpenter. JH mm mm m tmL V/, '///, ■WM y 1&T mm m mm White to play and mate in two moves. Wo. 686. By G-. E. Carpenter. 'WW?, i§ HHl "wmF'm m WW 4% ,,$/////////. m wm y mm. wsm. I mm mm § 4M, i M i White to play and mate in three moves No. 687. By Jacob Elson. mm mm wm iH fiJ u ¥M ■ If I WB HP k iP IP ■^^//i Swhyrf. J///////& MtwW. ■:^& mmmr ^^ ^ ^^. ^^ »§^ - warn m jk $M §M m White to play and mate in five moves. No. 688. By Jacob Elson. wm„ mm mm 7 K^^\^ M w,J M €k HI. Pi % H ■ULH / stt5777& W/////AY/. _^"U mm White to play and mate in four moves. No. 689. By Jacob Elson. m if 111 § iH „Wi^-wSi & Wm. HI mm, i is il iH.. I JKm r _JSSa r,^ [ j^T^l' P' IIP '■mm WHITE. White to play and mate in four moves. No. 690. By. Jacob Elson. H W ^ ... ■«W^:-wJ m -- WM * i:4^ii ^ ^WJI JS^ ^ ^^^ i ■ HP Jill IHi ^H P ;' Iflfl'^'^P HBH II baa siHI ess iHp HP Hi JillllP ^^ % 111 III fill S%3 W//7//V/A White to play and mate in four moves. 44 No. 691. By Jacob Elson. I HIP WSF '"if H m " I ^» illf "J i ■ 1 ml * HH hi' 1b"W"W " isy nm inn iHH White to play and mate in three moves. No. 693. By Jacob Elson. PH lip wfflk §u is nm ill "^ mm =■ vMa Warn mm ■ HP ■ i Hi m mmF m& ^ WZ/.. tMxs/Zi. IJJi H ip ■v. JI vM^A White to play and mate in three moves. No. 693. By F. W. Maetindale. mm jjj-.^P lip ill mm mm. mm **,,.:,,:, m Hi 41 # ; t.^.mm mm wt 4 HlH «L -™ 1 H mm White to play and mate in three moves. No. 694. By F. W. Maetindale. mm WW mm....... s > 1! iml IK «j . Ml _«„_!&: iBAJll § s m w. "i. m ■i M 2^5?i wm^m 1 il ■ ■ III HP I „,,l!l„„iii n H #1111 White to play and mate in four moves. No. 696. By F. W. Martindale. 4m ™ vm lit b 1 *.J "■■■■ii Si SHH< White to play and mate in three moves. No. 697. By F. W. Martindale. IP HP wk„ iSlf I §||p g White to play and compel Black to mate liim in four moves. No. 698. By Joseph Wainwright. mm J » i mm Wm wm., ,,wMk. i'>&3 X S l8p X p pi ■ay////.''//. HHP x HIP US MAM ■ mm ■T^ i m : mi WMte to play and mate in four moves. No. 699. By W. A. Shinkman. B*il« WW) 'tMm> w: m mm, wm A 'WW WmSM MfLM m m iESy WW. V. ■ 4 I ^J^' ^y* WW). §j|Sf mm, ww, * ww, IS m m WW. w^'m^z *to*L--Jf™. 'ww%, mm ip IF $Mr4. WW, WW, WW, WHITE. "White to play and mate in two moves. No. 700. By W, A. Shinkman. W wm mm wfomm mm H mm, i„»i 'WW w,^^wW,,.^...^^B,..^..^w%. .W///W, m I €qJ <■- • w, HI ■ 89! ////v//////. WHITE. White to play and mate in three moves. No. 701. By W. A. Shinkman. A j^W.^jM.ihB » 1 ; jp*1 A*. HIP WM 'WM % ■ JM& ?dc3l mm iffifl . ii J3 " B I ,,JB Bi PIP ^# HP Hi H ■ i ■ Hi .. ms .' 181 WHITE. . White to play and mate in three moves No. 702. By W. A. Shinkman. % f % iiiP i^ ySsStt lllllP ^^ 1 HHy HlH ^%54%%5 "--—ail ™ <%v, .■■■ // i A ^~ A m P W i%^ BL-JBLaa^ WHITE. White to play and mate in two moves. No. 703. By W. A. Shineman. x/ wm. y mm r wm. I mm. 4fm r wit fit ^S ww. mm, JsSm mm, . w- mm w iii..AJlB * mt i HP 5 Pi iH 'WtB, Wmk*L IS w. White to play and mate in three moves. No. 704. By Denis Julien. ill Awmf-^mah 1 W^~\TWm WW? %WM t WW/ W; . W//////A 1 «#H i . ,._ ^^ HM ■ PI w ■ IB "White to play and mate in five moves. No. 705. By Joseph N. Babson. BLACK. ...m PPf fa HI pgg m m mkSLmk —m m. wm w in WJF- III m. H .. m 'ml mm, mm mm White to play and mate in three moves. No. 706. By Joseph N. Babson. 'mM mm WW, l "'W /M m- M wM'""""w * mm). wLM. mm „„„ itt_™. !M8 «j™ ■////; ■////#■; mm i mm mm mm k i m ■ m m * »e@ mmi. fmw. 4&^m m mm m §mi mm ■ wm m White to play and mate, sui-mate, stalemate, and sui-8talemate ; each in two moves. 45 No. 707. By B. M. Neill. m 1 I m mi H 4 ■ if PIP Pi^ P- ill ill III, if fi ■ ■ i . i^ a ^p p^p mm \~mk lu iSi ■ HP J «n White to play and mate in three moves. No. 708. By B. M. Neill. ■LJ-l ii in in ifil ggH 'towZwzfr %%%VZM>. fflfi™" m. H WHITE. White to play and mate in three moves No. 709. By V. N. Pobtilla. MP.tlr.IH WM ^ ^p lJi_M m&^ . e J m JBm wm mm. iSi ^wmf^wr^^ i til White to play and mate in three moves. No. 710. By V. N. Portilla. BLACK. ** IP HIP Prl 'Wk HI P ™- «LJ"L 'Zy//////A, £t^4M Wm> eQ~\ maim a®ai ■; ■ -ate:- ; '*%/>,' ■ wwk. ^^ ^jS„ HP White to play and mate in four moves. No. 711. By S. Tybbell, of Adelaide, (SO. AUSTRALIA.) BLACK. m i iBI Ill i mf'.'Wwf^ mm * iO pfirt b ■ 1 ^ ifc . 4k mmr m m mm. n ll„,„„,JliJ 'mm m White to play and mate in two moves. No. 712. By S. Tybbell. m -mm ■ vffi 4 Htt" ■'"" is %. %m^x. A m HI i i H H ta White to play and mate in three moves. No. 713. By S. Tyerell. - ij. wm. III +A n ^» til HI 4 fill UMI Hii H« Wwm §81 ,w//mWl White to play and mate in five moves. No. 714. By H. Chaelick, of Adelaide. ™mw^^sBh^!F> ■//.■■ -/■■/' W///// y ///, oa. y/y/xs:////. is m.^m Wm. Hi ^ 'W?:* i K- ^ J/^7777^/,, '{/:■ m W » WW, WKk White to play and mate in three moves. No. 715. By H. Charlick. ■mm. mm, .ma. mm ^ mm wm HP 6 lH P White to play and mate in four moves. No. 716. By W. Crane, Jr., of Sydney. ■ HP HP i fa fel H i . \<^jfj g Jjil H * HLJHJ I IP^f 1^ PW ,t. ^ W3 WW k< J ^H $JH1 WHITE. White to play and mate in two moves. No. 717. By J. Willis, of Bogan. / %'//////A, mmmw, 'Wmm,, . , wwo'///,,. m mm : - hi * if i Hi, i H ^^^■^aFP^r- SP Hi IMP ^# MP H H Pi White to play and mate in three moves. No. 718. By E. B. James, of Molong. m wmp mSM mm B mm miM 1 m mm up up mf up i m 4m wm m w, mm White to play and mate in three moves. No. 719. By D. W. Clark, op Barnaul, ( SIBERIA.) W/4P///'/. VW////, wm mm M m^m Warn 4mi fUJ www. Jam www, www. , iu mm. m I ill* 181 mmf^mm7Tmmf m m m a wm,^ www, mm WHITE. ' "White to play and three ia-Jetrr moves. No. 720. By D. W. Clark. White to play and mate in four moves. No. 781. Ghuznee. By Eev. H. Bolton. *£$ i iUm ■ mm ^ "MW Wma W//////A wmm |i «m mm m Mm mW mm.. WMuaiwr*^™' / / WMMXy '„,,,,/,,,,, ^^^' J^////0™m White to play and mate in eleven moves. No. 722. By F. Hoffmann. %- ''MW&- 'm ■P mtT WK fm^Wm WU* lb m White to play and mate in three moves. 46 No. 723. By Alphons Beck. i nm mm, ^ 4 9K HP « -J—jjm ma WB i 01 1111 I w mm, wA w/////////.„ '//////.■//7s. , SI 'M*K# ■ White to play and mate in three moves. No. 724. By A. Bayersdorfer. m 1 w/m, « H il.fi.Mli 88 ^^ «# a^n Wfml r>. Wm "White to play and mate in four moves. No. 725. By J. Berger. ■mm 1111 gg WmJ^jtmf^ mm mm mm j^v p wm m£//rfi — wm..... fm WmJkJm mi. wm^-'Mz White to play and mate in three moves. No. 726. By J. Berger. ■ ■mU/HI i mm mm. «_. d ilit myy/Ma III i i ri •L-J""- go J.®w mm. WHITE. White to play and mate in four moves. No. 727. By F. v. Geyersstam. HI ■r^fli w % WtMfr W, vMwk k~\ §§§lfp A m §11 » nn mm A HP i IIP ^P S ■ * §■ ill " MR ■ ■ II P* 1 0%%%%% wzfflyZ't $9i=^v/. ;^aBHl^ ^si IfeM H3t "^ Wwk ...mam White to play and mate in three moves. No. 7S8. By C. Bayer. I PI HP IP v™!!"* _jskJk is, — WHITE. White to play and mate in three moves No. 729. By H. F. L. Meyer. ■ ■ ■ ■ ■ fill w. B^j§ ju 111 H B H 1 w w H^l 11 111 fci 181 4 ill a fHf -ML is ■mw ww. if White to play and mate in three moves. No. 730. By P. Klett. HHI 'WZW m mm W//4Z i w y/ ,,,,,,jwm>, *mwi w. 4mMa wmi Wm A mm i mm m * ww, * eg I m^ H €sl >JI WW, i AMI .;«./. JPL Wm^'mm. mm. i Wm JWm, y 3Ww> w, mm, White to play and. mate in four moves. No. 731. By P. Klett. w mm m bI i mm fnn wm, t////Zt7'. mi, 4M ilfli wi' ^hfwmfm 151 HP JlSil *^* JHH ^H «? IIP 111/^ jm llli ^^ Jill White to play and mate in five moves. No. 733. By J. v. Szirmay. --W - ir"' 11 *^L, ^W^ iP pp iw ^# ^ ' fit fj pi mm .». ias8 if %y////A y . vm fe^S ? White to play and mate ia four moves. No. 734. By J. A. Miles. ^wxT^imtf"*** m. je ! ill "11 i 111 <' White to play and mate in three moves. No. 735. The Topaz. By G. Chocholous and J. Dobrusky. -%- m m w, ww, WW, 4, W, wW6. ww, ww. Wi... r/ IfeU WW/,., m ww, i" A wW ^kii A /j, 'wm"" mm, mm, mk ™ m i |g| l«* WHITE. "White to play and mate in three moves. No. 736. The Avalanche. By A. Townsend. '//-■;,■ mm ■ wm & ^fmJ^mm^MmfT'm i\ i mm. W . ',. w, . 3 gg : -;- m ww, = JHH ww HI j wm I HI iflp -n- 11111 iiil m i v». '////,. White to play and compel Black to mate him wa a Book in eighteen moves. SOLUTIONS X signifies takes + „ check K ,, King moves Cov. „ Covers Where Black's moves are compulsory they are in most cases omitted. No. 1. WHITE. BLACI 1 E. b7 + 2 P. h5 + 3 E. b6 + 4 E. a6 + 5 P. XP. + 6 P. b6 + 7 E. a8 + 8 P. c6 + 9 P. b7 + 10 Kt b5 + 11 Kt c7 + 12 B. c5 mate. No. s. 1 B. g7 + 2 E. h8 + 3 E.. xQ. + 4 E. f6 + 5 E. d6 + 6 E. d4 + 7 E. f4 + 8 E. f2 + 9 B. fl + 10 E. f4 + 11 E. d4 + 12 E v d6 + 13 E. f 6 + 14 E. f 8 + 15 E. h8 mate. No. 8. 1 E. a4 + Q. a5 2 Kt. d8 + E. X B. 3 E. a7 + Q. X E. 4 P. mates. No, 4. 1 E. c8 + 2 E. 3 B. a6± o6# • 4 E. a!8vidate. 47 No. 5. ■WHITE. BLACK. IE. h5 + 2 R. g3 + 3 Q. g7 + 4 P. f7 + 5 P. xE. mate. No. 6. 1 E. g4 2 E. h4 3 E. h5 4 Q. f7 + 5 Q. g6 6 P. h7 7 K. g6 8 K. h6 9 P. g6 P. g7 mate. No. 7. 1 B.xKt.+ E. xB. 2 Q.xP. + K. x Q. 3 E. al + 4 E. a8 + K. xE. 5 Kt. b6 + 6 E. d8 + E. c8 7 E.xE, + 8 E. a8 mate. No. 8. 1 Kt.xf5 + P. x Kt. (A) 2 B. e5 K 3 Kt. c5 mate. (A)l. E. x Kt. 2 B. e5 + E. X B. 3 P. X E. mate. No. 9. 1 Kt. f5 + b7 b6 2 P. X P + K. a8 3 P. queens + E. xQ. 4 E.xE. + 5 Kt. d6 mate. No. 10. WHITE. BLACK. 1 Q. h8 + E. X Q. 2 Kt. f6 + K. g7 3 Ktxd7 + Q. cov. 4 B. XQ. + 5 E. f8 + 6 E. xh8mate. No. 11. 1 Kt. c6 + B. X Kt. 2 E. d8 + K 3 Q. xB. + K. X Q. 4 B. e4 + K. c5 5 Kt. d3 + Kt x Kt. 6 E. d5 + K. c6 7 E.xKt+ K. c5 8 P b4 + K. c4 9 B. d5 mate. No. 1 Q. h7 + 2 Kt. g6 + 3 B. mates. 12. Kt x Q. No. 13. IE. f 8 + K. h7 2 Q. g6 + B. X a 3 P. XB. + K. X P. 4 B. d3 mate. No. 14. 1 Q. d6 + K. a8 2 Q. c6 B. x Q. (A) 3 E. d8 + Q. c8 4 E. xQ. mate. (A) 2. Q. c8 3 E. d8 anything. 4 Mates. No. 15. 1 Ktxc7 + B. x Kt. 2 Q. x B. + K. x Q. 3 B. e4 + K. b8 4 Kt. a6 mate. No. 16. 1 Kt. e7 + K 2 Q. g8 + E. x Q. 3 Kt xB. + P. xKt. 4 E. h.4 mate. No. 17. 1 a gs + K. xQ. 2 Kt. e7 + K - 3 Kt. f7 + E. x Kt 4 E. X E. + E. f8 5 E. X E. mate. No. 18. •WHITE. BLACK. 1 2 3 4 5 3 4 Q. d7 Kt. g5 Q. h3 Q.X h6 + E. b.7 mate. (A) 2. Q. f5 Q,. h3 mate. E. g8 b8 f8 (A) P. h6 P. xQ. P. h6 P. XKt No. 19. 1 2 Q. f8 + E. xQ. + Q. x Q. K 3 B. h.6 + K. X B. 4 E. g8 anything. 5 E. h5 + B. X E. 6 P. g5 mate. No. 20. 1 Q. e5 + K. a8 2 Kt. c7 + 3 KtxE. + 4 Kt. c7 + 5 Kt. a6 + 6 Q. t8+ E. X Q.. 7 Kt. c7 mate. No. 21. 1 Q. a6 E. b8 2 f3 d4 Q. c8 3 d4 b5 E. b7 4 bl c3 a b8 5 c3 e4 Q. c8 6 e4 g5 Q. b8 7 g5 e6 Q. c8 8 e6 d8 Q. b8 9 P. xE. + Q. X P. 10 Q. xQ. mate. 1 2 3 4 5 6 7 8 9 10 No. 22. 1 Q. e8 + 2 B. h6 + 3 Q. f8 + 4 E. f5 + 5 Q,. g7 mate. No. Kt. f5 KtxB. Ktxh5 + Kt. f6 + Kt X Q. + Q. f6 E. gl + E. X Kt + Q. f3 + Q. g4 mate. K. X B. P. X E. 23. Q. P. g8 h5 K. h7 K. h6 K — K. Kt. S 4 g2 h3 No. 24. WHITE. BLACK. 1 Q. g4 + B. X Q. 2 E Xh6 + P. x E. 3 B. f7 mate. No. 25. 1 b3 xc5 + K. c4 + 2 B. b3 + E. X B. + 3 P. xE. + Q. x P. + 4 Q. xQ. mate. No. 26. 1 Q. f6 + K. h.6 2 Q. h4 + K 3 Q. d4 + K. h.6 4 Q. f4 + K 5 Q. e5 + K. h6 (A) 6 E. h.5 + P. x E. 7 Q. f6 mate. (A) 5 K. f8 6 Q xE. + K. g7 7 Q. f8 mate. No. 27. 1 0. d6 + P. X Q. 2 E. f4 P. d5 3 Kt. e4 + P. x Kt. 4 B. e3 P. x P. 5 Kt. f2 P. x P. mate. No. 28. 1 E. f8 + E. x E. 2 E. X E + K 3 B. g8 + K 4 B. f7 + K 5 B. g6 mate. No. 29. 1 Q. X h7 + K. x Q. 2 E. hi + K. g8 3 E. h8 + K. f7 4 E. h7 + K. g8 5 E. g7 + K. h.8 6 Kt X g6 mate. No. 30. IE. e8 + Kt. c8 2 E.xKt + K. x E. 3 E. a8 + Kt. b8 4 P. f6 + K. d8 5 E. X Kt mate. No. 31. IE g8 + 2 B. g5 + 3 Kt. gl + 4 B. e3 + 5 B. f2 + No. 82. WHITE. BLACK. IB. e8 + 2 E. x h.6 + 3 Q. e7 + K. g8 4 Q f7 + 5 Q. f8 + 6 B. g6 4- 7 Q. g8 mate. No. 33. 1 Q. X Kt + Q. x Q. 2 b7 x g7 + K. h8 3 E. g8 + B. x E. 4 B. d4 4- K. b.7 5 E. g7 + K. h8 6 E. a7 -(- Q. cov. 7 B. x Q. mate. No. 34. 1 P. f6 P. g3 2 Q. h6 Q. g8 3 K. g5 P. b3 4 B. b2 P. a4 5 B e5 P. a3 6 B. d6 P. b2 7 B. 18 P. queens. 8 B. g7 + 9 Mates. No. 35. 1 2 3 4 5 b4 E.X E. c6 e5 c6 4- c7 + d7 + e7 + c6 mate. K K. x E. K K No. 36. 1 2 3 4 5 B. Q. E. B. P.x d4 + a5 + a3 + c5 + P. mate. K K. x Q. K. b4 K. c4 No. 37. 1 Q. h3 + Kt. h6 (A) 2 Kt. f7 4- K. g7 3 Kt. h5 + K. g8 4 KtxKt+ K. h8 5 P. mates. (A) IK. g7 2 Kt. e8 + E. X Kt. 3 Q. h.7 + K. f6 4 Q. h.8 + K. g5 5 P. mates. K. K h4 g 4 XB. No. 38. Kt. 1 Kt. dl + K. c4 K. h4 2 E. e4 + Kt. cov. Kt. XB. mate. 3 P. e3 + 9 10 11 WHITE. P. X Kt + P.xP. + P.XB. + R. b4 + R. a5 + P. X R. mate. BLACK. No. 39. R. P. R. R. B. P. P. P. P. Kt+ P. mates b8 + g6 + h5 + h8 + c8 + e3 + d4 + c5 + b6 + K. X R. No. 40. 1 R. 2 Q. h7 + B. x R. g8 + B. x Q. P. g 6 B P. mates. No. 41. R. K. Q. R. a7 + f7 f6 + a8 -f K B. al B. x Q. B. GOV. P. mates. Q. Q. Q. R. No. 42. White mates. f6 + f7 + e7 + + B. cov. P. mates. Compels Black to mate. IB. h6 + 2 R. a8 + 3 Kt. d2 4 K. g2 5 Kt, ft 6 Kt. h2 7 K. hi 8 P. b7 8. P. mates. No. 43. Kt. f7 4- Q. f8 + R. X h6 + Q.X P. + Q. f8 + 6 R. h3 4- 7 P. mates. B. x Kt. B. cov. P. xR. B. cov. B. cov. Q. xR. WHITE. BLACK. No. b8 + c8 + b6 -4- d2 + Q.xKt + Kt. c6 + R. a4 + P. mates. R. Kt. R. B. 44. Kt xB. R. X Q. B. x Kt. B. xR. No. 45. d8 f7 + Ktxh6 + e5 ■ f7 + KtxB. + Kt. g5 + R.xKt + Kt. f7 + Kt. e5 + Kt. g6 mate. K. g S K. h8 B. x Kt. K K. h8 Ktx R. No. 46. 1 2 3 4 5 6 7 8 9 10 11 3 4 5 6 7 8 9 10 11 12 13 14 15 16 R. B. Q Q. Q. R. Q. Q. R. Q. a4 + e5 + c4 + c7 + d7 + f4 + e8 + h8 + f6 + h6 + P. mates. No. 47. Q. Q. R. R. Q. Q. Q. Q. B. Q. B. B. Q. R. P.X e7 + e6 + d7 + a7 + d6 + c6 + c7 + d7 + h6 + f7 + g7 + e5 + f6 + h7 + P. + K. c8 P. X Kt mate KtX P. No. 48. 1 Kt. g2 2 Kt. el Kt. g3 Q. dl Q. Kt. 3 4 5 6 7 Kt. cl 8 R. f8 -f al e2 "WHITE. BLACK. 9 f6 f7 + 10 E. h8 + 11 B. h4 + 12 B. f5 + 13 cl d3 + 14 Q. b2 15 K. f2 16 E. a7 17 E. a6 18 K. f3 19 B. g4 20 K, g2 dis. + 21 P. c4 dis. -j- 22 Q. cl + 23 B. f3 + 24 Q. g5 + 25 B. g4 + 26 B. h5 + 27 E. h6 + 28 Q. d8 + 29 P. c5 30 Q. a8 + 31 B. d8 + 32 P. mates. No. 49. 1 Kt h-7 + 2 e4 f6 + 3 h7 f8 anything. 4 Q. or Kt. mates. No. 60. 1 Q. d4 + P. e5 2 Q. X P. + K. x Q. 3 B. e7 P. b5 + 4 K. e5 Q. gl + 5 P. d4 + Q. X P. + 6 P. X Q. mate. No. 61. 1 P. f7 P. e6 2 P. X Kt. aQ+ B. d8 3 a. XP. + K. b8 4 Q. d6 + B. cov. 5 Q. xB. + 6 a c8 mate. No. 5S. 1 Q. c5 + K. a5 2 Q. b4 + K. b6 3 Q. d4 + K. a5 4 B. b4 + K. a4 5 B. el dis. + K. b3 6 a b4 + K. c2 7 a bl mate. No. 53. 1 Kt. d6 + K- 2 E. h8 + B. g8 3 E.X B. + K. K E. WHITE. 1 2 3 4 5 6 7 8 9 10 1 2 3 4 5 6 7 8 9 10 11 1 2 3 4 5 6 7 8 9 10 11 12 13 14 15 16 17 18 19 Q. P. Kt. Kt. Q. c4 + g6 + f7 + h6 + g8 + Kt. mates. (A)< Kt. f7 + Q. h4 + Q. h8 + P. mates. (B)4 Kt. b.6 -4- Q. g8 + Kt. f7 + P. mates. No. Q. X f6 + Q. g7 + Kt. d5 + Kt. f6 + KtxQ. + Kt. f6 + KtxE. + Kt. f6 + Kt. d5 + Kt. e7 mate. BLACK. K. h7 (A) K. h8 K. g 8 K. h8 E. x Q. K. h8 K. b7 (B) K. g8 K. x Kt. K. g8 K. h8 e. x a. K. h7 54. Kt. Kt. Kt. E. Kt. No. e6 + c7 + a6 + d3 + b4 + 55. Q. x E. KtxQ. + Kt. Kt. Kt. Kt. P. b4 + a6 + c7 + e6 + c4 mate. No. 56. E. x P. + KtxP. + B. f5 + Q. h.6 + B. h7 + B. x P. + B. hi + B. x E. + B. b.7 + B. x E. + B. h7 + B. xKt + B. h.7 + B. Q. Q. Q. B. d3 + H7 + h8 + d8 + X c3 Q.XP.+ K K. x E. P. X Kt. K K-^- K K K K K K K K K K K K K Q.XB. (A) K- BLACK. 20 Q. c6 mate. (A) 18 B. 19 Q. xP. + K- 20 Q. f5 mate. a4 1 2 3 4 5 6 7 8 9 10 11 12 13 14 15 16 17 Q. Q. Q. Q. Q. Q. Q. Q. Q. B. Q. Q. Q. Q. Q. Q. Q. i2 a No. 57. d7 + d6 + g6 + g4 + d4 + d2 + el + gl + hi + el + h4 + f2 + d4 + g4 + g6 + d6 + d7 mate. (A) UK. f5 g4 + &c. No. 68. K. e3 (A) P. a4 + K. X P. Kt. c3 + K. h4 Kt. a6 + Q. x Kt. Kt. d5 + K. a4 Kt. c3 + &c. Drawing by perpetual cl:eck. If Black play 4 K. b5 then Kt. c7 + winning the Q,. and the game. No. E c7 + Kt. b6 + E. e7 + Either P. + 69. K. d5 K. e5 Kt. cov. e2 X P. Other P. mates. No. 60. 1 e3 2 B. 3 R. d5 + c3 + el 4 E. al + 5 P. mates. Q. E. Q. Q. Q. Q. No. a6 + b7 + c6 + c7 + d7 + e7 + K. a5 Q, cov. Q must take B. at c3 Q. X B. 61. BLACK.. 7 Q. 8 Q. 9 E. 10 E. 11 E. 12 Q. 13 E. 14 E. 15 Q. 16 Q. 17 E. 18 E. " 19 Q. 20 Q. 21 E. 22 Q. 23 Q. 24 Q. 25 E. 26 Q. 27 P. X 16 E. 17 Q. 18 Q. 19 E. 20 Q. K. K. f7 + f 6 + g7 + g5 + h5 + g6 + f 2 + f7 + d6 + e6 + f8 + c8 + c6 + a8 + c6 + b7 + b3 + b6 + c4 + 63 + P. mate. A. 15 K. c7 + b6 + c6 + c8 + a8 + d8 e8 or (A) K. a4 c8 &c. as before. No. 62. Q. c8 + K. d7 P. b6 B. c5 K. x Q. P. mates. P. a6 P. a5 a. X Q. + Kt. Kt. Q. Kt. Q. P. + No e3 + c4 dis f7 + d2 + e7 + c4 + mate. 63. P. X Kt. Q. Q. B. E, P. P. No. d8 + a8 + c6 + + a8 + a5 + X Kt. mate. 64. K. x Q. Kt xP Kt. b8 + G. xE. + B. xKt. B. d2 + Kt. c3 + P. mates. (A) K. c3 + B. X Q. + P. mates. No. 65. K K. x Q. K. b4. (A)(B) 3 Q. + Q. cov. + BLACK. WHITE. WHITE. BLACK. (B)J ! B. xP. + 4 B. h5 4 K. c3 + B. cov. 5 B. + 5 B. xB. + 6 P. mates. 6 P. mates. (B) 2. K. g8 3 E. + K. f7 No. 66. . 4 B. h5 + 5 E. a8 1 Q.X d6 + K. e4 6 B. + 2 P. b3 + Kt X P. 7 P. mates. 3 E. cl + Kt X E. (0) 2. K. g7 4 E. e4 + Kt X E. 3 E. e8 5 Q. d3 + Kt x Q. 4 B. h5 6 P. X Kt + K. d5 5 E. a8 7 P. x Kt mate. 6 B. + 7 P. mates. No. 67. (D) 3. K. g« IP. g6 + 4 E. + K. g7 2 P. g7 + 5 E. a8 3 Kt. g5 + P. X Kt 6 B. + 4 E. a4 + P. X E. 7 P. mates. 5 B. g6 + 6 P. g4 anything No. 70. 7 P. g5 mate. 1 Kt. f3 + Q. 2 Q. d5 + K X Kt. No. 68. 3 Q. d6 + K 4 Q. e6 + K- 1 B. x P. + P. XB. 5 Kt. g6 4- E. X Kt. 2 Q. g8 + K. X Q. 6 Q. e7 + 3 E. e8 + K. g7- 7 Q. e8 + 4 Kt. f5 + K. g6- 8 P. x E. mate. 5 Kt. f4 4- B. X Kt. 6 P. h5 + K- No. 71. 7 P. h4. mate. 1 B. x g7 + No. 69. 2 Q. x h7 + 3 Kt. f6 + 1 E. e2 K. f7 (A) 4 E. gl + K. f5(A) 2 K. g5 K. f8 (B)(0) 5 Kt. h4 + 3 B. Ii5 K. g7 ()->' 6 Kt. h5 + 4 E, e8 7 B. d3 + 5 B. + 8 P. mates. 6 P. mates. (A) 4. Kt. g2 (A)l. 2 K. g5 3 B. h5 K. K. K. g7 f8 g» g7 (E)(F) 5 E. xKt + 6 Kt. h4 + 7 Kt. h5 + 4 E. + K. 8 P. mates. 5 E. a8 6 B. + No. 72. 7 P. mates. 1 Q. e8 + (E)2 K. f7 2 K. f6 3 B. h5 + K. f8 G) 3 Kt. iS 4 E. + 4 Kt. g5 5 E. a8 5 Q. e6 + 6 B. + 6 K. f7 7 P. mates. 7 Q. e8 + (G)3 K. g8 8 P. mates. 4 E. + K. g7 5 E. a8 No. 73. 6 B. + 1 E. + 7 P. mates. 2 Q. x E. + (F)2. K. h7 3 B. e4 3 E. e8 4 E. e7 ■WHITB. BLACK. 5 Kt . c4 K. x Kt. (A) 6 E. e5 7 E. h5 8 E. d5 9 P. X P. mate. (A) 5. P. a5 6 B. b7 7 E. e5 8 E. d5 9 P. X P. mate. No. 74. First Solution. 1 Q. a8 2 E. + 3 Q. d8 4 Q. f 8 + 5 Q. f4 + 6 K. h3 7 Q. d6 + 8 Q. e5 + 9 P. mates. Second Solution. 1 R. e8 2 Q. b7 + 3 Q. e7 + 4 E. f8 + 5 E. f7 6 K. f3 7 E. f 6 + 8 E. f5 + 9 P. h5 + 10 P. g4 mate. Third Solution. 1 E. e8 2 Q. d7 + 3 Q. e7 + 4 E. f8 + 5 E. f7 6 K. f3 7 E. f6 + 8 E. f5 + 9 E. h5 10 P. g4 + 11 P. h5 mate. No. 76. 1 Q. a7 + 2 Q. a8 3 Q. b7 4 B. c3 + 5 Q. c8 6 P. d3 P. X B. 7 P. XP. 8 Q. c4 9 Q. c6 io P.: 34 35 36 37 38 1 2 3 4 5 6 7 8 9 10 11 12 1 2 3 4 5 6 7 8 9 10 11 12 13 14 E. P. E. Q. Q. e6 + b6 + d3 + c6 + c7 + P. mates. No. 92. Q. c8 + P. e6 + Q. f8 + Q. X E. + a. f5 4- d5 + b3 -r c4 + d5 + f5 + f7 + f8 mate. Q. a. Q. Q. Q. Q. Q. E. K. No. 93. To mate with K. at e8. cl d3 d3 eo eo f7 el g2 d2 el g2 e3 17 go el f2 e3 f5 g5 e4 -f f2 g2 e4 f2 + f5 e3 e3 f 1 dis. mate. To mate with E. at c8. The Play must be made on the other side of the board. 1 2 3 4 5 6 7 8 9 10 11 12 13 14 Kt. B. d5 + Kt. f7 + KtxP. + Kt. f7 + Kt. d8"+ No. 94. e5 + K. P. Kt. Kt. Kt. E. e8 queens + f7 + g8 B. cov. e5 + g6 + hi + E. X B. + Q. X P. + Q. x B. mate. P. X Kt. Q. B. cov. P. X E. B. cov. No. 95. a. c5 + f8 + WHITE. liL A.0K. 3 B. c3 dis. + 1 Q. d6 + 5 B. e2 + 6 E. XP. + 7 Q. a6 + 8 U. b5 + 9 E. b6 + 10 E. a7 + K. d8 11 Q. d5 + K. e8 (A) 12 Q. f7 + 13 a c7 + 14 B. h5 + 15 a. e7 + 16 Q. £7 + 17 K. e7 dis ■+ !8 Q. f8 + 19 B. g6 + Kt. COV8. 20 P. xKt. mate. (A) K. c'8 12 Q. c6 + 13 a. o7 + &.C as before. No. 96. 1 E. c5 + E. x E. 2 E. b8 dis. + E. X B. + 3 P. XE. P. c5 4 Q. b4 + P. X Q. ■+■ 5 P. X P. mate. No. 97. 1 E. 2 E. 3 P. 4 P. 5 B. 6 E. 7 E. 8 Q. 9 Q. 10 Q. 11 B. 12 B. 13 B. 14 B. 15 B. 16 B. 17 B. 18 B. 19 B. 20 B. 21 Q. 22 a 23 Q. 24 Q. 25 Q. 26 a 27 d7 28 Q. 29 Q. 30 B. 31 Q. al + xKt.+ b5 + b4 + c3 + b2 + a2 + dl + X a4 + b3 + b2 + X e5 + b2 + Xf6 + b2 + Xg7 + b2 + Xh8 + b2 + d4 + b2 + c2 + d2 + dl + gl + hl + f6 + h4 + h3 + e3 + g3 + K. b-3 K. f4 K. e6 WHITE. B LACK. 32 Q. g4 + K. e5 33 Q. g5 +. 34 Q. d5 + 35 a. d7 + 36 a e8 + 37 a f7 + 38 Q. f8 + 39 P. mates. No. 98. 1 P. be 2 Kt . e7 3 K. c4 4 K. d5 5 K. d6 6 K. d7 7 Kt c6 8 K. c7 9 ?, + 10 P. queens + 11 Q. mates. * No. 99. 1 Q. eo -4- 2 f5 e7 + 3 Q. f6 + 4 Q. h8 + 5 Q. c8 + 6 Q. c6 + 7 Q. f6 + 8 Q. f4 + 9 Q. e3 + 10 Q. xP. + 11 a c6 + 12 P. mates. No. 100. 1 Q. a8 + 2 P. d5 3 B. el 4 Q. b8 + 5 E. Xd6 + 6 K. a3 7 B. b2 8 Kt d7 9 Kt xP. 10 Q. a7 + 11 E, b4 + P. X E. mate No. 101. 1 Q. g6 2 P. g3 3 Q. g5 + 4 E. c8 + 5 P. g 4 6 Q. e5 7 Q. h8 8 B. d2 9 B. cl 10 B. a3 WHITE. r ^.BLACK. 11 Q. g8 + 12 K. d5 13 E. d8 14 K. e4 15 E. f8 + 16 Q. g7 + 17 B. el P. mates No. 102. With B. at al. 1 Kt f7 2 Kt. c7 3 Kt. g5 4 K. f6 5 c7 e6 6 K. f7 dis. mate WitliB. at hi. 1 Kt. d7 2 Kt. c7 K. c7 3 Kt. b5 K. d8 4 Kt. 15 K. e8 5 K. e7 6 Kt. e8 7 Kt. d6 + 8 Kt. b7 9 K. d6 10 K d7 11 K. c6 12 Kt. d6 13 K. c7 dis. mate. No. 103. With P. at c3. 1 Kt. e3 2 E. bl 3 K. e2 4 E. b5 K. c3 5 K. dl 6 K. d2 7 E. g5 8 E. f5 9 P. f3 + 10 P. c3 mate. With P. at f3. 1 to 6 as before. 7 E. i5 8 P. c3 4- 9 'P. f3 mate No. 104. 1 Q. d3 + 2 E. c2 + 3 Q. b3 + 4 Q. c3 + 5 E. cl + 6 Q. e2 + 7 E. al + WH ITE. BLACK. "WHITE. BLACK. 8 E. a4 + 32 E. a3 + E. x E 9 Q. c4 + 33 Q. a6 + 10 E. a6 + 34 P. X E. mate 11 B. c8 + 12 E. b6 + K. a7 No. 106. 13 Q. c7 + 1 B. dl 14 Q. c6 + 2 E. g8 15 E. b7 + 3 E. g 5 16 E. b5 + 4 K. e7 17 Q. c7 + 5 E. h5 + 18 B. b7 + 6 P.+ 19 B. g2 + 7 P.+ 20 Q. c6 + 8 P. mates. 21 E. b7 + 22 E. c7 + No. 107. 23 E. c8 + 24 Q. a8 + 1 K. h4 * 25 E. c6 + 2 Kt. fo 26 Q. a6 + 3 Kt. e7 K. h8 27 E. c4 + 4 E. f8 + 28 Q. a4 + 5 E. f7 + 29 E. c2 + 6 P.+ 30 Q. b3 + 7 P.+ 31 Q. c3 + 8 P. mates. 32 E. b2 + 33 E. f2 + No. 108. 34 Q. c2 + 1 B. do 35 Q. dl + Q. mates. 2 K. 3 Kt. o7 c4 No. 105. 4 Kt. eo 1 Q. ho + 5 Kt. d7 2 Q. g4 + 6 P.+ 3 Q. f3 + 7 P. mates. 4 E. g2 + 5 E. d2 + No. 109. 6 Kt. e2 + 1 K. f6 K. g8 7 Kt. cl + 2 B. c5 8 E. g2 + 3 B. f8 9 E. e2 + 4 B. g7 h4 10 E. el + 5 P. 11 Q. 12 + 6 P. h5 12 E. hi + 7 P. he 13 E. b4 + 8 P. g3 g 4 g5 g6 + h7 mate 14 Q. f4 + ' 9 P. 15 E. h6 + 10 P. 16 Q. f6 + 11 P. 17 Q. h8 + 12 P. 18 E. f6 + 19 Q. f8 + No. 110. 20 E. d6 + 21 Q. d8 + 1 Kt. e6 22 E. b6 + 2 B. f7 23 Q. c7 + 3 B. e8 K. h8 24 Q. c6 + 4 P. g6 25 E. b7 + 5 P. g5 26 E. b3 + 6 P. g 4 27 Q. c7 + K. a8 7 P. g3 28 Q. c8 + 8 K. hi 29 Q. b8 + 9 K. h2 30 Q. t>7 + 10 K. h3 31 Q. b6 + 11 B. f7 + WHITE BLACK. WHITE. BLACK. 12 P. + ,'A) 1 K. h7 13 P. + 2 P. g 4 * K. g« 14 P. + 3 K e7 K. g7 15 P. mates. 4 P. f4 &c. as befo: (B) 4 K. '6. h6 No. Ill . 5 K f7 With P . at e4. 6 7 P. P. g5 f6 1 E. eo 8 P. f7 2 E. e7 9 P. e4 &c. as befoi ■e. 3 Kt . d4 (C) 4 K. h7 4 Kt . e6 5 K. f7 K. e6 5 K. g6 6 P. e4 K. h7 6 K. ho 7 P. g5 &c. as before. 7 K. h6 (D) 5 K. h7 8 E. e8 + 6 P. go K. 8' 7 9 Kt g7 7 P. e4 K. gH 10 P. g 5 + 8 P. f6 K. h7 11 P. g6 + 9 K. f7 &c. 12 P. e5 mate. 6 K. (E) 5 K. f7 h8 WithP . at g4. 7 8 P. P. g5 n4 1 K. e6 + K. e8 9 P. e5 2 E. d5 10 P. e6 3 Kt d4 K. g8 11 P. e7 4 E. h5 K. g7 12 K. g'G fe 5 Kt c6 K. g6 or g8 13 P. 6 Kt e7 + 14 K. he 7 E. hi 15 P. g6 ho 8 E. h7 10 K. 9 K. d6 .17 K. g5 h6 10 E. hi 18 K. 11 E. h8 +■ 19 P. f7 + 12 E. g8 20 P. g7 mate. 13 P. e5 + (F) If Black play K to h8 14 P. e6 + h.7 White plays K. to e 7 &c. 15 P. go mate. No. 113. No. US 1 E. b6 + K. d8 (A) 2 a. e8 + K. x Q. 1 K. to K. g8(A) 3 E. Xg6 K. X e7 + K. X f8 2 K. e7 K. g7 4 B. B. 3 P. g 4 K. g6 5 E. e6 + K. — 4 P. f4 K. 87(B)(0) 6 E. e8 mate. 5 P. 15 K. h6 (D) (E) (A) 1 K. X f7 + K. f8 6 K. f6 K. h7 2 B. f'7 7 P. g& K. g» 3 B. X g6 + K. g8 e8 + Kt. cov. 8 K. e7 K. g7(F) 4 a 9 B. e4 K. g8 5 Q. X Kt. mate. 10 p. 16 K. h7 11 K. f7 No. 114. 12 P. eo 1 Kt b3 + 13 P. eG 2 Kt c5 + 14 K. g 6 3 B. c3 B. e5 + 15 P. e7 4 E. cov. + B. x B. 16 K. h5 K. h7 5 P. b4 + B. X P. 17 P. + K. g8 6 P. X B. mate. 18 K. g5 19 K. he No. 115. 20 P. + 1 a c7 B. x Kt. 21 P. i nates. 2 B. ti3 + P. cov. P.XP.f^6 d7 + 5 c8 b6 + 6 E. a8 + Kt. x E. 7 Kt. c8 mate. No. 119. 1 Q. a4 + 2 Kt. d7 + 3 Kt. e5 + 4 E. b.5 + .5 E. c5 + 6 E. c6 + 7 E. d6 + 8 R. d7 + 9 E. e7 + + 10 Kt. g6 + Q. x Kt. 11 Q. e8 + Q. X Q. + 12 E. x Q. + + K. x E. 13 P. f7 mate. No. 120. 1 R. e8 + E. cov. dis. + 2 E. f 3 dis X Kt. e5 3 E. X E. + Q. x E. 4 B.x Kt + Q. cov. 5 E. f8 + B. cov. 6 E.x B. mate. No. 121. WHITE. BLACK. 1 Kt. d7 + K. e6 2 Q. b5 + K. x Q. 3 Kt. d4 -f K. x E. 4 Kt. CO + K. a3 5 Kt. b5 + K. b2 (i Kt. d3 + K. x B. 7 Kt. a3 mate. No. 122. 1 Kt. a2 + Kt x Kt, o B. a3 B. c2 3 B.x Kt. dis -1- B. cov. 4 B.x B. K. dl 5 B.x b2 E. xE. 6 B. e4 dis. + E. cov. 7 E.x E. mate. 9 10 11 12 3 4 5 fi 7 8 9 10 11 12 13 14 No. 123. IP. b7 + 2 B. f4 + K. 1.8 E. cov. K. B. K. K. K. o-fi eo L7 g8 f8 B. xh2 B. e5 K. e8 K. d8 B. X E. mate. P. P. P. P. P. P. P. P. ho h.4 g« + h.3 h2 g2 P. queens. No. 124. Q. f5 + c5 x d7 P. g4 mate. (A) 1 K. Q. Xd7+ K. Q. e6 mate. No. 126. K. x Q. (A) 1 Kt. e4 K. e6 g 5 f3 d4 id K. f4 e4 g6 g7 f6 f5 e.j i'3 d4 fo + g7 U g 6 f6 f4 fo g4 + g3 mate. K. K. K. K. K. K. K. K. K. K. K. K. K. e7 f6 fo eo 15 g4 h4 g 4 hi prl h.3 h2 hi h2 hi No. 126. IE. d3 + 2 Kt. e6 3 Ktxb2 + K. X E. (A) E. x Kt. K. c2 No. 149. This Problem and " The Hero " (No. 141) , as well as Ghuznee, appear never to have been correctly solved. They were published by Lewis, in 1844, without Solutions, and copied into Le Palamede, and afterwards by Alexandre in his " Beauties of Chess," but the Solutions of the three Problems in both these works are incorrect. 1 2 3 4 5 6 7 8 9 10 11 12 13 14 15 "WHITE. E. xP. + E.xKt + E. g7 + E. X d7 + E. g7 + E. X Kt + E. g7 + E. c7 + Q. xE. + E. g7 + E. g4 + Q. b7 + a. X B. + Q. XE. + Kt. f8 mate. BLACK. Q. d8 B. cov. E. cov. Q.XQ. 1 E. 2 B. Kt. Kt. Kt. KtX Kt. 8 KtX 9 B. 10 Kt. 11 KtX 12 Kt. 13 P. 14 Kt. 15 P. x 16 E. 17 Kt. No. 150. a7 + a4 + e6 + c7 + a6 + b6 + c7 + B. + d7 + c7 + Kt + e8 + g7 + f8 B. (a Kt.) + h7 + g6 mate. P. XE. Q. c3 d3 c5 + Q. X P. + Kt. d3 + P. b3 + P. bl + Kt. mates. No. + x P. a6 161. P. K K K. x Q. K. x a4 K. a5 K No. 152. E.xKt + Q. Q. K. Q. E. c3 + c7 + f5 + f4 g5 + K. K. K. K. E. K. X E. e7 f6 g 6 g7 h7 WHITE. BLACK. 7 Q. h2 E. x E. 8 B.: < E. + K 9 Q. h.6 mate. No. 153. 1 Q. al 2 B. e4 3 E. d2 4 Q. dl 5 E. a2 6 B. bl 7 P. b4 8 a. d7 + 9 P. b6 10 K. b5 11 K. c4 12 K. b3 13 E. c2 14 K. a2 15 K. al 16 Q. c8 + 17 E. a2 + 18 E. a6 + 19 Q. c6 + 20 E. b6 +' 21 Q. c5 + 22 E. b4 + 23 E. e4 + 24 Q. b5 + 25 a b2 + P. x Q. mate No. 154. 1 Kt. d5 Kt x B. 2 Ktxb6 + K 3 Q. a7 + K 4 Kt. a8 anything. 5 Mates. (A) 1 P. x Kt. 2 B. d6 dis + Kt. b8 3 B. f5 + E. d7 4 Q. x Kt. mate. No. 155. 1 P. b7 + B. x P. 2 E. d7 KtxKt. 3 R. xB. Kt. d8 4 E. bl dis + Kt. b7 5 E. X Kt. anything. 6 E. bl dis mate. No. 156. 1 Kt. d6 + K. d3 2 P. a3 E. d8 3 P. g8 queens E. x Q. (A) 4 Kt. cl + K 5 K d2 P. c4 6 a3 X b4 anything. 7 Mates. WHITE. BLACK. (A) 3 E. x Kt. 4 Kt. cl + K. d4 5 Kt. e2 + K. d3 6 Q. b3 mate. No. 157. 1 Q. e5 + K.xE. (A) 2 Kt. d6 + K 3 Kt. d3 + K 4 Kt. e4 B. x a2 5 Q. d4 + B. cov. 6 Q. xB. + K 7 Q. b4 + K 8 Kt. c6 mate. (A) If lK.xQ. 2 Kt. d3 + K 3 E. d4 mate No. 168. 1 KtxB. + K. Ii7 2 Kt. f8 4- K. g8 3 Kt. c!7 + K. b7 4 E.x B. + K.xE. 5 KtxE. P. xE. 6 Ktxb3 P. h4 7 Kt. d4 P. h.3 8 Kt. c6 P. h2 9 Kt. eo P. queens. 10 Kt x f 7 mate. No. 169. 1 KtxE. + E. x Kt. 2 B. a7 + 3 Q. f3 + 4 K. b6 5 K. b5 + 6 K. c5 7 K. c4 + 8 K. d4 9 K. e4 + 10 B. e3 P. X B. 11 K.XP. 12 Q. dl mate No. 160. 1 Q. e7 + K. d5 2 Q. d8 -4- K. c5 3 0. a5 + P. b5 4 E. c4 + K. d5 5 E. c5 + K.xE. 6 Q. b4 + K. d5 7 Kt. e7 + K. e5 8 Q. X f4 + K. XQ. 9 Kt. g6 mate. No. 161. 1 Kt. e7 + K. e4 2 Kt. f5 P. X P. 3 K. h4 E. e6 4 Q. g2 + K. X Kt. 5 Q. g4 mate. No. 16S WHITE. BLACK. 1 Kt. c6 dis. + K. b5 2 E. a6 E. XE. 3 Q. e2 + K. xKt. 4 Q. xB. + E. d7 5 Q. e4 + K. b5 6 Q. c4 + K. c6 7 Q. xE. + K. d5 8 Q. c4 + K. c6 9 Q. e4 + K. b5 10 Q. a4 mate No. 163 1 Q. g6 B. XB. 2 Q. xP. + B. cov. 3 Q. f8 + B. cov. 4 Kt. f7 + K. h7 5 Kt. go + K. h8 6 Q. h6 + B. cov. 7 Q. xB. mate. No. 164. I Q. a4 + K. XKt 2 P. c5 + K. e6 3 Q. g 4 + P. f5 4 g5xP. + K. f7 5 P. XP. + K. g8 6 Q. e6 + E. x Q. 7 P. f7 + K. h8 8 Kt. f6 E. X Kt. 9 P.xE.(aQ)+ E. cov. 10 Q. x E. mate. No. 166 . 1 Q. d6 + Q. eo 2 Q. b4 + Q. e4 3 Q. b8 + Q. eo 4 E. c7 Q. d6 5 Q. b2 B. g7 6 Q. XB. Q. c5 7 Q. b2 Q. d5 8 E. c4 + Q. XE. 9 Q. b8 + Q. c7 10 Q. X Q. mate No. 166 . 1 Q. f3 + K. a7 2 Kt. c6 + 3 Kt. d8 + 4 P. XP. + K. XP. 5 Q. c6 + 6 Q. c5 + 1 Q. d5 + 8 Q. d4 + 9 Q. e4 + 10 Q. e3 + 11 Q. f3 + 12 Q. 12 + 13 Q. xP. + "WHITE. BLACK. 14 15 16 17 18 19 20 21 22 23 24 23 24 1 2 3 4 5 6 7 8 9 10 11 12 13 14 1 2 3 4 5 6 7 8 9 10 11 12 13 14 15 16 Q. Q. Q. Q. Q. Q. Q. Q. Q. Q. Q. Q. Q. f2 + f3 + e3 + e4 + d4 + d5 + c5 + Xc8 Xa6 -f c6 + X B. mate. (A) 22. c6 + mates. P. B. E. B. f6 eov. cov. COT. (A) No. e4 + f6 + h6 + g3 + KtxB. + E. f6 + E. Xb6 + E. f6 + E. c6 + Kt. E. E. Kt. 167. K. f5 g3 + e2 + g6 + E. X c5 + P. mates. Kt. Kt. Q. No. Q. Xd6 KtxQ. Kt. c7 Kt. c5 P. e4 P. XE. + mate 168. Q. d5 B. x Q. B. x Kt. Kt. d5 E. xP. No. 169. KtXa6 + KtxE. + Kt. a6 + Kt. c5 + Q. a7 + KtxKt + B. x f4 + B. £5 + B. g5 + K. X Q. Kt. Kt. P. B. Kt. Kt. Kt. d6 + d7 + h7 + e6 + f7 + e5 + g6 mate. E. x P. No. 170. P. queens + K — P.Q. h8 + K— Q. g7 + K— WHITE. BLACK. 4 Q. e 5 + K 5 P.Q. g8 + K 6 Q. h8 4- B. cov. 7 P.Q. X B. + K 8 Q. g7 + Kt. cov. 9 Q. xKt. + K 10 Q. e4 + K 11 Q. e5 + Q. cov. 12 P.Q. x Q. + K 13 P.Q. f8 + K. g6 14 P.Q. e8 + K. h7 15 P.Q. f7 + K 16 Q. is + K 17 P.Q XE. + K 18 P.Q. e5 + K 19 P.Q. e4 + K 20 Q. g7 + K 21 P.Q. f5 + B. cov. 22 P.Q. 1l3 + B. cov. 23 P.Q.xh2 P.xP.Q 24 P. g4 mate No. 171. ] Kt. £4 P.xKt. 2 R. gl K. e8 3 E. g8 + B. cov. 4 Q. d6 K. f7 5 E. g7 + B. X E. (A 6 Q. e7 4- 7 Mates. (A) 5 K. e8 6 E. e7 + B. X E. 7 Q. x B. mate. No. 173. 1 KtXg7+ K. f6 2 E. f8 + K. g5 3 Q. f4 + K. x Q. 4 E. X B. + Kt X E. 5 Kt. e6 mate. No. 173. 1 Q. d3 + K. g2 2 Q. g3 + K. fl 3 Q. gl + K. e2 4 Q. dl + K. e3 5 Q. d3 + K. x Q. 6 P. X Kt (aE.) anything. White is stalemated. No. 174. 1 Kt. a6 + K 2 Kt. d6 + K. x Kt. 3 Q. xKt+ P XQ. 4 E. h4 anything. 5 B. mates. No. 175. 1 B. xP. dis. + Ktx Q. 2 B. e8 dis. + Q. X E. WHITE. P. BLACK. g5 + K. g6 4 P. f8 (a Kt.) + + and mate. No. 176. 1 Kt. f6 + KtxKt. 2 KtxKt+ K. Ii8 3 Q. h4 E. g8 4 B. f8 E. g7 5 Q. h6 E. X B. 6 Ktxh7 P. f5 7 Kt. f6 + E. h7 8 Q. x E. mate. No. 177. 1 P. f6 Q. gl + 2 E. xQ. P. xQ. 3 P. f7 + E. cov. 4 P. f8 (a Q.) mate. No. 178. 1 E.xKt. E. X E. (A) 2 Q. f5 K. g8 3 B. b3 + a7 f7 4 Q. xE. + E. X Q. 5 E. e8 mate. (A) Kt. e7 2 E. xKt. g8 X f8 3 Q. f5 E. X Q. 4 E. + E. cov. 5 E. x E. mate. No. 179. 1 E.X f7 B. e6 (A)(B) 2 B. X B. Q. X Kt. 3 E. X E. + K. L.7 4 E. h8 + K. x E. 5 Q,. e8 + K. h7 6 B. g8 + K. h8 7 B. f7 + 8 Q. mates. (A) E. x E. 2 Q. XE. + K. Ii8 3 (J. g6 &c. (B) Q. e8 2 E. e7 + Q. f7 3 Q. xQ. + E. X Q. 4 E. mates. No. 180. 1 B. h6 K. f7 2 Q. g7 + K. e6 3 0. g8 + K. d6 (A) 4 P. + K. X P. 5 P. + Q. X P. * 6 B. f 8 + and wins the Q, *K. X P. 6 B. d2 + and wins the Q. WHITE. BLACK. (A)3K. e7 4 B. g5 + K. d6 5 Q. f8 + K. anywhere. 6 Q. orB. mates. No. 181. 1 Q. g5 + K. X Kt. 2 Q. XB. + K. XQ. 3 B. xb3 mate. No. 182. 1 Q. h5 + K. e6 (A) 2 Q. e5 + -K. X Q. 3 P. XE. + K. e6 4 P. xE. (aKt.)mate. (A) 1 Kt. cov. 2 Q. xKt+ K. e6 3 P. x R. mate. No. 183. Kt. 1 Q. d6 Kt. e6 2 E. X c7 Q. e5 3 E. c8 + K. X E. 4 Kt X a7 mate. No. 184. 1 Kt. e6 + Q. X Kt. 2 E. X Kt. Q. X E. 3 Q. a5 + K. d4 4 Q. c3 mate. No. 185. 1 Q. b3 B. d5 2 Q, dl P. f4 3 Q. h5 Kt. d7 4 Q. e5 + Kt.xQ. 5 Kt. c5 mate. No. 186. 1 P. g4 + B. X P. 2 bl c3 B. X Kt. 3 B. d5 K. e5 4 Q. f 6 + K. X B. 5 Kt. mates. No. 187. 1 Q. g5 E. f8 2 E. c5 E. f5 3 Q. f4 + E. or B. x Q. 4 E, or Kt. mates. No. 188. 1 B. d4 2 B. e3 3 Kt. c4 4 B. a4 5 B. mates. cl el E. x B. P. X Kt. anything. No. 221. WHITE. BLACK. 1 E. cl Q.xKt+(A)(B) 2 K. dl anything. 3 E. or B. mates. (A) I E. X E. 2 K. dl &c. (B) 1 E. g8 2 K. bl &o. No. 222. 1 Kt. d2 B. X Kt. + 2 Kt. e4 B. moves. 3 E. mates. 1 Q. 2 B. d4 3 Mates. No. 223. g5 Q. xKt. + anything. No. 224. 1 Q. a2 . E. XKt. 1 P. c4 anything. 3 Mates. No. 225. 1 Q. b4 E. b2 2 B. b3 anything. 3 Q. mates. No. 226. 1 Q. al+ P. d4 (A) 2 Q. b2 either E. + 3 ft. xE. mate. (A) c2 b2 2 Q. X E.+ E. x Q. or anything. 3 Kt. mates. 1 Q. No. 227. b2 No. 228. 1 E. d6 either E. xE. 2 B. d5 or e6 anything. 3 Mates. No. 229. 1 Kt. c5 " P. X Kt. 2 E. e5 anything. 3 E.orB. mates. No. 230. 1 B. d5 Q. gl 2 B. g3 Q. a7 3 B. f2 anything. 4 Mates. No. 231. 1 Q. f8 Q. cl + 2 B. c2 dis. + Q. X E. 3 Q. f5 + Kt X ft. 4 B. mates. No. 232. WHITE. BLACK. 1 Kt. f5 Kt X e5 dis. + 2 P. c4 + Kt X d dis. + 3 Kt. e3 + KtxKt.dis.+ 4 P. c4 + Kt X P. dis. + 5 Q. e4 mate. (A) (A) 4 P. X P. en pass. dis. + 5 Q. d3 mate. No. 233. 1 E. x P. + P. X E. 2 E. c6 Q. X E. (A) 3 Kt. g8 anything. 4 P. or Kt. mates. (A) 2 B. g6 3 Kt. f 7 + If B. x Kt. 4 P. mates. If Kt.xKt. 4 E. X B. mate. No. 234. 1 Kt. c5 + K. d6 2 B. e7 -f K. x E. 3 E. g5 anything. 4 f 5 moves, mate. 235. 1 Kt. d2 + K. e5 2 Kt. c6 + K. d6 3 R. dl 4 Mates. 236. 1 E. g6 B.xKt+(A)(B) 2 K. d2 + E.xQ.* 3 E. e6 + 4 One of the Bishops mates. *IfK. f4ore5 E. + and B. mates. (A) IB. d4 + 2 K. d2 B. c3 + 3 K. e3 4 Mates. (B) IB. e3 + 2 B. cov. B. d4 + 3 K. d2 4 Mates. No. 237. 1 P. d4+ K. x P. (A) 2 Q. a7 + P. cov. 3 Q. al Q. X Kt. 4 Q. gl Q. X Q. * 5 Kt. mates. * 4 If any other move, 5 E. e4 mate. (A) 1 K. b6 2 Q. b7 + K 3 Q. Xb4 + K. X Q- 4 E. a3 anything. 5 P. mates. No. 238. WHITE. BLACK. 1 Kt. f5 + K. do 2 Kt. e4 E. X d4 + 3 K. e3 E. d3 + 4 K. f4 B. d6 -j- (A) 5 KtxB. mate. (A) 4 Q. X Kt.+ 5 P. xQ. mate. 1 Kt. 2 Kt. d3 + 3 P. x Kt+ 4 B. c2 No. 239. f5 + K. c5 K. d5 K. xE. anything. 5 Mates by discovery. No. 240. 1 E. c7 Kt.xE. 2 B. f3 + P. X B. 3 P. x Kt. E. x E. (A) 4 P. X E. (a Kt) anything. 5 P. mates. (A) 3 P. d6 4 P. c8 queens E. X Q. (B) 5 E. X P. mate. (B) 4 R. X E. 5 P. mates. No. 241. 1 2 3 4 5 Q. a2 R. f5 Q. d2 E. e6 Q. d5 + Kt X Q. KtxB. + Kt. b.5 mate. No. 242. 1 2 2 4 5 Kt. d4 a3 c4 K. g3 Kt. dl Q. x Kt. P. X Q. K. f4 anything. K. moves, mate. No. 243. 1 P. h4 P. X Kt. 2 Kt. f 1 Q. X B. 3 •■ E. c4 P. b5 4 B. X f3 + P. X E. + 5 B. b7 + P. X R. 6 P. g3 mate. No. 244. 1 B. d5 + K. X B. 2 Kt. c7 + K. e4 3 0. X f4 + K. X Q. 4 Kt. d5 + K. e4 5 Kt. g5 + K. X Kt. 6 P. e4 mate. No. 245. 1 E. e4 E. X B. 2 KtXf6 + K. e6 3 Kt. d4 + K. e7 4 B. xKt+ K. X B. 50 WHITE. BLACK. 5 E. e5 anything. 6 Mates. No. 246. 1 E. b8 either E. X Q. (A) 2 Kt. f5 + K. d8 3 B. b7 dis. mate. (A) 1 B. d7 2 B. X B. P. X Kt. 3 E. e8 mate. If 2E. X Q. orE. a8 2 Kt. f5 mate. No. 247. 1 B. ho B. f5 (A) 2 B. X f3 B. X Q. 3 B. d5 + K. d3 4 Kt. cl mate. (A) 1 P. e2 2 P. b3 + P. X P. 3 Q. x e2 + P. X Q. 4 B. X P. mate. No. 248. 1 Kt. e7 E. x Kt. (A) 2 K. f2 Kt. e5 3 K. e2 Q. X P. 4 KtxB. + E. xKt. 5 E. d5 mate. (A) 1 Kt. + orKt. orB.xKt. 2 K. e2 and mates in two more moves. If 1 B. a5 2 KtxP. + 3 E. mates. If 1 B. x B. or Ktx P. or B. a5 or E. e7 2 P. + 3 B. mates. No. 249. IB. cl + E. e3 2 E. x P. E. X E. 3 Q. h8 If Black take either Q. or Kt. he is mated at once. 3 anything else. 4 Kt. g6 + E. X Kt. 5 Q. mates. No. 250. 1 2 3 4 5 Kt. f5 Q. e5 B. b5 + Kt. e6 + B. mates. KtxKt. P. X Q. K. d4 K. XP. No. 251. 1 2 3 el B. B. d3 + e4 f 4 mate B. x Kt. B.x B. (A) WHITE. BLACK. If 2 E. X B. or E. X Kt. 3 Q. x d5 mate If 2 K. X B. 3 Q. f4 mate. (A) 1 E. X Kt. 2 B. f4 + anything. 3 Mates. No. 252. 1 Q. c3 E. f8 2 Q. h8 E. f7 3 Q. g7 anything. 4 Kt. or Q. mates. No. 253. 1 E. d4 + K. x E. (A) 2 P. g4 If he play P. X E. he cannot win in four ! 2 E. x P. 3 Kt. g3 E. x Kt. or anything. 4 Q. matea. (A) 1 K. fo 2 Q. f7 + K. g5 3 Q. f6 + K. X Kt. 4 E. X E. mate. No. 254. 1 Kt. d3 + P. X Kt. 2 B. g4 Kt X B. (A) 3 E. f5 + 4 Q. mates. (A) 2 Kt. e4 (B) 3 Q. f5 + K 4 Q. co mate. (B) 2 B. X E. (C) 3 Q. c3 + K 4 Q. mates. (C) 2 K. x E. (D) 3 Q. c7 mate. (D) 2 B. X Kt. 3 Q. c7 + 4 Q. e5 mate. No. 255. IE. e4 + K. x E. 2 E. e6 -4- K. fo 3 B. c8 anyihing. 4 Mates. No. 256. 1 B. d2 P. c5 (A) 2 B. g5 anything. 3 Mates. (A) 1 P. e6 (B) 2 B. b4 + anything. 3 Mates. (B) 1 K. x P 2 E.xP. P. e6dis.+(C) 3 B. b4 mate. WHITE. BLACK. ( C) 2 K 3 B. mates. No. 257. 1 Q. f8 Kt x Q- 2 P. g 6 P. fe 3 P. g7 anything. 4 Mates. No. 268. 1 Q. e2 anything. 2 Mates. No. 259. 1 E. g5 Q. e4 (A) 2 E. e5 Q.xE.orK- 3 Q.xhS + K 4 Q. mates (A) 1 Q. a6 2 Q. d5 + anything. 3 G. dl -4- anything. 4 Mates. No 1 E. e5 2 Kt. e4 3 Q. or Kt. mates. 260. P. xE. anything. No. 261. 1 Kt. e7 E. g8 2 Kt. a8* anything. 3 Q, or Kt. mates. *The object of this move is far from being obvious. No. 262. 1 2 3 3 H. Q. B. Q. b4 e4 + c6 mate. (A) c6 mate. P. X E. K. x Q. (A) K. x Kt. or K. co No. 263. 1 2 3 4 Kt Q. Kt. E. Xf4 a 5 d6 + co mate. P. X Kt. P. X Q. Kt x Kt. No. 264. 1 G. c4 B. g.3 (A) 2 B. b6 K -* . 3 B. c7 + anything. 4 Q. cl or Kt. f 1 mate. (*) 2 B. el 3 Q. cl + B.cov.orK.xP. 4 Q. x B. or Kt. c5 mate. (A) 1 K 2 Q. cl + 3 Kt. f3 + 4 Q. c4 mate. No. 265. •WHITE. BLACK. 1 K. C2 B. X E. + (A) 2 K. c3 B. X Q. (B) 3 B. mates. (A) 1 E. X E. 2 Q. b3 + K. x P. 3 Kt X P. mate. (B) 1 K. X E. 2 K. c3 di8. 4- 3 Q. mates. No. 1 Q. b6 2 Q. x c7 3 Kt e8 4 Mates. 266. E. c6 E. X Q. (A) (A) 2 P. d4 3 Q. X c6 4 Q. mates. No. 267. 1 Kt. fl + K. e2 2 Q. d6 If Black take Q. with either P. then Kt. e6, and mates next move. 2 E. d3 (A) 3 Kt. e6 E. f8 4 Q. x E. + B. x Q. 5 Kt. d4 mate. (A) 2 B. d3 3 Q. e5 + B. cov. 4 Q. x B. + E. cot. 5 Q. x E. mate. No. 268. dl E. hi (A) t B. 2 R. d4 + B. c4 3 Q. h3 anything. 4 Mates. (A) 1 E. x Q. 2 P. c3 dis. + B. cov. 3 E. d4 + Q. cov. 4 E. X Q. mate. No. 269. 1 Q. X Kt. a8 X Q. (A) 2 E. X a4 anything. 3 KtorE. mates. (A) 1 e8 X Q. 2 E. b6 3 Kt. mates. No. 270. 1 Q. Xa7 + P. b6 2 Q. g7 B. x Q. 3 K, g4 anything. 4 B. or Kt. mates. No. 271. 1 Kt x c2 P. e5 (A) 2 E. X P. + anything. 3 Q. or Kt. mates. WHITE. BLACK. (A) 1 P.xKt. (B) 2 Q. x Kt + K. X E. 3 Q. d5 mate. (B) 1 E. X Kt. 2 E. f4 + 3 Q. mates. No. 272. 1 E. x e4 P. c5 (A) 2 E. c4 3 Mates. anything. A 1 P. x E. dis. -f anything. 2 B. x E. 3 Mates. No. 273. 1 d5 e3 K. xE. (A) 2 Q. b4 + K. x E. or Q. 3 Kt. mates. A 1 E. x B. 2 Q. c3 3 Q. mates. No. 274. 1 B. c7 B. x B. (A) ' 2 Q. h8 anything. 3 Q. mates. A 1 P. f4 (B) 2 Q. e5 + K 3 Kt. mates. (B) 1 Q. f6 2 P. e4&c. Note. If White begins 1 P. b5 he will be defeated by 1 B. a5 No. 275. 1 B. e2 B. x B. 2 Q. c5 P.orKtxQ. (A) 3 Kt. d4 dis. + K 4 Kt. mates. (A) 2 Q. d8 4- 3 Kt. d2 dis. + K 4 Q. mates. No. 276. 1 Q. b6 B. orE.xQ. (A) 2 E. Xd7 dis. 4- K. g6 3 P. e5 anything. 4 E. or B. mates. (A) 1 P. f4 (B) 2 P. e5 4- K. f5 3 B. xP. + K. X P. 4 Q. d4 mate. (B) 1 B. X c3 2 E. X d7 dis. x 3Q. gl + 4 Mates. No. 277. 1 Kt. g6 P. X Kt + 2 K.xP. anything. 3 (J. mates. No. 278. WHITE. BLACK. 1 Q. f5 2 Q. bl 3 Mates. Kt. e5 anything. No. 279. 1 B. 16+ K. X B. 2 B. bl R. x B. 3 Q. h8 + K. g 5 4 Q. x i'6 mate. No. 280. 1 Q. b5 P. x Kt. (A) 2 B. h2 anything. 3 Q. or Kt. mates. (A) B. c4 2 Kt. f3 dis. + B. X Q. 3 Ktxd6mate. No. 281. 1 B. b.4 Q. g8 (A) 2 Q. X e5 -f Kt X Q. 3 B. el and although Black can gwe check with four different pieces, yet White mates next move. (A) 1 Kt c5 + 2 Kt x Kt-f- and mates in 2 moves. No. 282. 1 E. g3 P. K 5 2 Q.xKt. Kt X Q. (A) 3 E. b3 anything. 4 Kt. mates. (A) 2 B. X P. 3 Q. xB. anything. 4 Mates. No. 283. 1 Kt. c8 B. d4 (A) 2 Q. f7 E. h.7 3 Q. a2 Kt X Kt. 4 B. xP. + K. XB. 5 Q. c4 + K. X Q. 6 Kt. d6 mate. (A) 1 Kt. d4 2 Q.xB. E. h8 3 E. g7 E. x Kt. 4 Q. f7 E. c7 5 Q,. xE. anything. 6 Q. mates. No. 284. 1 B. h6 dis. + B. cov. 2 B. e8 Q x d5 3 B. xB. Q. dl + (A) 4 B. cov. dis. + Q. x E. 5 B. a2 a. e6 6 E.xKt+ E. X E. 7 B. x Q. mate. (A) 3 P. X B. 4 E. xP. + K. h7 ■WHITE. BLACK. 5 E. b7 + K. X B. 6 Kt. f5 + K. h5 7 P. mates. No. 285. 1 Q. fl + K. h2 2 E. g2 + K. h3 3 E. xKt, dis.+ Q. cov. 4 E. a4 dis.+ B. cov. 5 E. Xa3 B. X B. 6 Q.xB. + Q. cov. 7 Q. fl + Q. cov. 8 E. a4 Q. X Q. mate. No. 286. 1 B. f8 Kt. c4 (A) 2 P. e4 anything. 3 Mates. A 1 E. x P. + (B) 2 Kt. g5 + K 3 Q. mates. B. 1 a X P. (C) 2 Ktxf4 + E. x Kt. 3 Q,. d7 mate. C. 1 Kt. d6 (D) 2 Q.xKt + K. x Q. 3 KtxE. mate. DIE. f5 (E) 2 P. xE. + anything. 3 Q. f7 mate. E 1 K. d5 2 Q. e5 + K. c4 3 Q. c5 mate. No. 287. 1 Q. g6 K. xKt. (A) 2 Q. bl anything. 3 Q. b5 mate. (A) K. X B. (B) 2 Kt. b7 anything. 3 Q. mates. (B) 1 K. c4 or c6 or P c2 2 Q. d3 anything. 3 Q. or Kt. mates. No. 288. 1 B. xP. Kt.xB. (A) 2 Q. el B. d4 + * 3 P. X B. + anything. 4 Q,. mates. * 2 B. X P. or K. x Kt. 3 Q. g3 + K 4 Q. mates. (A) 1 Kt.xKt. (B) 2 Kt. f6 dis. + Kt. b5 3 Q. x Kt + anything. 4 Q. or Kt. mates. (B) 1 P. queens (C) 2 B. f6 + K. X Kt. 3 Q. d8 + anything. 4 Mates. WHITS. (C) B. f6 + Q. b6 + Q. eO mate. BLACK. 1 B. b8 K. xKt. K No. 289. B. el Q. d4 B. h4 + Mates. (A) P.xQ. B. h4 + Mates. (B) h4 + 8'5 + X Q. mate. (C) 1 h2 h4 + B. Q. Q. Q. B. P. e2 (A) f4 X Q. anything. Q. X f7 (B) Kt.xQ. anything. Q. e4 (C) Kt. x B. Q. e7 Mates. B. g2 Kt. x Q. anything. or g4 No. 290. 1 Q. h3 R. + (A) 2 Kt. cov. dis. + K. f7 dis. + 3 Kt. e5 mate. (A) 1 Q. or E. d7 (B) E. h7 Q. f8 or h8 or E. X Q. 2 c4 e5 + anything. 3 Q., Kt., or B. mates. (B) 1 K. d7 dis. + or B. x Kt. 2 f4 e5 dis. + K. c7 3 B. mates. No. 291. 1 B. d4 B. e4 or c6 (A) 2 B. e5 Q. X B. Q. xh5 + Q. X Q. Kt. X B. mate. (A) B. h3 B. f7 + K. d7 P. queens + K. eS Q. mates. No. E- P. E. B. cl h5 hi b2 5 Mates. 292. bl b3 P. a6 P. a5 anything. No. 293. Q. b6 Kt. d6 + Q. f2 B. mates. Q. a6 (A) B. x Kt. anything, WHITE. (A) 1 P. XE. Kt. g7 + Q. f6 mate. BLACK. E. X P. Q. a4 Ktx Kt. No. 294. (B) 1 B. f5 a8 d8 2 B. dl B. g7 (A) 3 B. a2 anything. 4 Mates. (A) 2 E x P. 3 B. e4 + Ktx B. 4 Q, x E. mate. (B) 2 f8 e8 (C) 3 P. x E. (a Kt.) anything. 4 Kt. mates. (C) 2 f8 g8 3 P. x E. queens + &c. If 1 f8 d8 2 P. queens. B. x Q. 3 Q. g8 &c. No. 296. Q. c7 + E. x Q. E. d6 Q. x B. P. d4 + K. x E. P. c5 mate. No. Kt. e4 + Q. e5 + E.xE. E. d7 mate. 296. K. d5 P. X Q. anything. No. 297. B. xQ. K. x E. or anything. B. or Kt mates. E. E. g7 a6 (A)l Q X d7 + Q. d3 mate. (B)2 Q. a4 mate. B. B. X B. cov. (B) K. c4 No. 298. 1 Q. 2 K. P. X Q. (A) anything. g6 c2 Mates. (A) 1 Kt. e6 E. c3 anything. Mates. No. 299. 1 E. 2 E. h2 cl Kt. h4 + E. c2 B. mates. Kt. g6 E. x E. Kt x Kt. anything. WHITE BLACK. No. 300. 1 Q. el B. e5 + (A) 2 K. d4 B. X E. + 3 Q. c3 B. orP.xQ. + 4 K. b3 anything. 5 Kt. mates. (A) 1 P. f2 o Kt. d7 + E. X Kt. 3 E. do + P. X E. 4 Q.Xb4 + K 5 Q. b6 mate. No. 301. 1 E. ol Kt. e4 (A) 2 Q. b3 P. X Q. 3 P. o4 mato. (A) 1 B. eo 2 E. dl + anything. 3 Q.orKt. mates. No. 302. 1 B. g6 B. g4 (A) 2 B. e8 Q. e7 3 Q. x c4 Kt. c2 4 Q. a4 + P. X Q. 5 Kt. mates. (A) 1 B. f5 2 KtxP. + K 3 Q. dl + Kt. c2 4 Q.xKt + B. X Q. 5 B. xB. mate. No. 303. K. b5 (A) K. x Q. 1 R. f4 2 Q. a4 + 3 Kt. mates. (A) 1 P. X E. 2 Kt. c5 + anything. 3 Q. or Kt. mates. No. 304. 1 Kt. f6 Kt X f6 (A) 2 E. d7 + Kt x R. 3 P. f6 anything. 4 B. or Kt. mates. (A) 1 E. d8 2 Kt. g4 ' E. dG 3 B. c6 + R. X B. 4 E. mates. No. 305. IE. d7 + K. xKt. 2 B. f 5 + K. X B. 3 E. e7 P- moves. 4 P. X P. mate. No. 306. 1 Q. h2 + B. cov. (A) 2 Q. h8 B. _e5 + 3 E. e3 mate. v v WHITE. liLAOK. (A) 1 Kt. COV. 2 Q. b2 anything. 3 E. c6 mate. No. 307. 1 B. g7 E. e5 2 Q. el E. X Kt. 3 ft. XP. mate. No 308. 1 E. g2 B. X E. 2 Q. a2 anything. 3 Q.orKt. mates. No. 309. 1 B. e3 E. e5 2 Kt. b6 R. x Q. 3 B. co dis. mate. No. 310. 1 E. f7 No. 311. 1 B. xP. B. c8 2 Kt. f3 + K. X P. 3 B. g7 anything. 4 Kt. mates. No. 312. 1 B. el E. f6 (A) 2 Kt. e7 + K. e5 3 E. e4 mate. (A) P. h3 2 B, g3 &c. If 1 E. f5 2 Kt. b8 dis. + If 1 Either Kt. moves 2 B. c3 &c. No. 313. 1 B. g7 E. x B. 2 E. f2 anything. 3 E. g5 or f4 mate. No. 314. 1 Q. d3 B. e8 (A) 2 Q. h7 anything. 3 Q.orKt. mates. (A) 2 Kt. c7 2 Kt. e6 + Q. or Kt. X Kt. 3 Q. mates. No. 315 1 B. b4 anything. 2 E.xb6 anything. 3 b7 mates. No. 316. 1 B. a2 B. X B. 2 Q. bl Q. X P. + 3 B. xQ. anything. 4 Q.orKt. mates. No. 317. ■WHITE. BLACK. 1 Q. d2 E. b4 2 B. e6 B. d6 3 E. g~ E. b.5 (A;(B) 4 Q. f4 + . IfE. XQ., - 5 P. mates. If B. X Q 5 E. g4 mate. (A) 3 B. f4 4 P. go + anything'. 5 Q. -mates. (B) 3 B. e7 4 P. g3 + K 5 B. i'7 mate. No. 318. 1 B. b7 + Rt X B. 2 E. a4 B. X E. (A)(B) 3 Q. a7 -f E. X Q. 4 E. c7 anything. 5 E. mates. (A) 2 P. X Q. 3 E. XE. + B. X K. 4 E. c7 5 Mates. (B) 2 b7 c5 3 E.xE. + B. X E. 4 Q. x o 5 5 Mates. No. 319. 1 Kt. f5 + K. g4 2 Kt. h6 + K. g5 3 Kt. e4 + E. X Kt. 4 Q. h4 + K. X Q. 5 B. b6 anything. 6 B. d8 mate. No. 320. 1 E. f4 + K. X E. 2 KtXd5 + K. XE. 3 Q. e2 P. X Q. 4 P. f4 mate. No. 321. 1 B. dl K. X E. at cl (A)(B) 2 Kt. c4 anything. 3 B. b3 dis. mate. (A) 1 K. X E. at el 2 Kt. e4 anything. 3 B. f3 dis. mate. (B) 1 B. X Kt. 2 B. b3 anything. 3 cl dl mate. No. 322. 1 E. b2 B., P., or Kt x E. 2 Q. a2 anything. 3 Q. mates. ■\VJIITE. BLACK. No. 323. 1 Q. g8 . P. X Kt. 2 E. c2 anything. 3 Q. mates. No. 324. 1 Q. a4 P. f4 2 Kt. e3 + P. X Kt. 3 Q. XP. + K. X Q. 4 B. mates. No. 325. 1 Q.xE. E. bl 2 E. b4 B. X K. 3 Q. d2 + Kt. or B. X Q. 4 Kt. mates. No. 326. 1 Kt. g5 K. g7 2 Kt. g8 B. X Kt 3 Q h8 + K. X Q. 4 B mates. No. 327. 1 B. e4 K. XB. 2 Kt. f3 dis. +K. X d3 (A) 3 Q. h7 mate. (A) 2 K. X f3 3 Q. x Kt. mate, &c. No. 328. 1 B. c7 B. X Q. 2 Kt. e3 anything. 3 Mates. No. 329. 1 B. e6 E. xKt. 2 Q. d7 + K. eo 3 B. d5 anything. 4 Mates. No. 330. 1 B. d4 K. p4 2 Q. h7 + K. X B. (A) 3 Q. d3 + B. X Q. or anything. 4 Kt. mates. (A) 2 K. X Kt. 3 Q. h4 4- 4 Q. g4 mate. No. 331. 1 Kt. b2 P. x Kt.(A) 2 Q. a7 4- K. e5 3 Kt. c4 + P. x Kt. 4 Q. mates. (A) IE. h6 + 2 P. X E. P. x Kt. 3 Q. f6 K. co 4 Q. b6 mate. ■WHITE. BLACK. No. 332. 1 Kt. f4 P. x Kt. (A) 2 B. e4 anything. 3 Mates. (A) 1 B. g2 2 Q. Xb6 + P. X Q. 3 Kt. e6 mate. No. 333. 1 E. e4 1 K. X E. 2 Kt. f 6 mate. 1 B.orKt.xE. 2 Kt. f4 + + mate. 1 E. X B. + 2 KtxE. dis + 1 P. X P. + 2 B. X P. mate. 1 Q. e6 2 E. d4 mate. I E. x P. + 2 Kt. cov. dis. mate. 1 Kt. f4 P. d6 E. e7 Q. e8 or Ktx P. 2 Kt. f6 mate. 1 P. x Kt. R. g6 &c. 2 E. e5 mate, No. 334. 1 Kt. f4 1 K. d6 2 Kt. e6 mate. IK. f6 2 Kt. e6 mate. 1 E. xKt. + 2 Kt. f3 mate. IB. c4 + 2 Kt. e2 mate. 1 P. + 2 Ktx P. dis. mate. IB. c4 + 2 Kt. e2 mate. I K.xKt. 2 Kt. e6 + + mate. .1 E. a7 or d7 or E. g2 or Kt. e2 2 Kt. f3 mate. No. 335. 1 Q. g4 1 K. c6 2 B. d7 mate. 1 K. c4 2 B. X P. mate. 1 K. e5 2 B.X P. dis. mate. 1 Kt. moves. 2 Q,. e4 mate. 1 P. moves. 2 B XB. mate. WHITE. BLACK. No 336. 1 P. h3 1 K. X E. or E. X e6 2 B. h2 mate. 1 B. X E. 2 P. e3 mate. 1 Q. X B. or Q. X dl &c. 2 E. f5 mate. &c. No. 337. 1 Kt. d6 E. X Kt. (A) 2 K. X B. anything. 3 Mates. (A) 1 B. X B. 2 Kt. e5 + K 3 P. d3 mate. No. 338. 1 E. c7 1 K. X P. 2 E. e4 mate. 1 E. X E. 2 Q. do mate. 1 Kt. e4 2 E. XE. mate. 1 Kt. d3 2 P. e3 mate. 1 Kt. anywhere 2 Q. gl mate. else. No. 339. 1 B. b7 K. x Kt. 2 Q. xP. + K. x Q. 3 E. c4 dis. mate. No. 840. 1 Q. dl P (A) (B) 2 P. d3 + 3 Mates. (A) 1 K. d5 2 Q. a4 K. X P. 3 Kt. d8 mate. (B) 1 K. d3 2 K.XP. + K. d4 3 Q. al mate. No. 341. 1 Kt. h2 E.. f4 2 Q. f7 Kt X Q. 3 E. g4 + K. x E. 4 Kt.Xf3mate. No. 342. 1 Q. h7 E. xE. (A)(B) 2 Q. a7 3 Mates. (A) 1 P. x Kt. 2 Q. + &c. (B) 1 K. XKt. 2 0. + &c. WH^TE. BLACK. 2 Q. (C) 1 Kt. e7 X Kt. 3 Q. mates. • No. 343. 1 2 3 Q. Q'. Q. g3 P- f4 ( g6+ K c6 + K. X Q. 4 Kt mates. (A) 1 K. d5 2 Kt b4 4- K. e6 3 4 Q. B. g6 + K. e7 d.6 mate. No. 344. 1 E. e3 4- P. X E. 2 E. e7 + K. f6 3 4 Q. Q. fl anything. orB. mates. No. 345. 1 Kt. f8 Kt.xKt. 2 Kt. g5 anything. 3 Kt. or B. mates'. No. 346. K. g5 (A)(B) K. xQ. 1 Q. hi 2 Q. b.4 + 3 Kt. mates. (A) 1 K. e3 2 B. xE. anything -3 Q., Kt., or B. mates. (B) l P— 2 Q. el anything. 3 Mates. No. 347. 1 Q. f3 B. gl (A) or E. g3 2 Q. fl anything. 3 Kt. d3 anything. 4 Kt. or P. mates. (A) 1 E. g4 2 Q. xP. E. xKt. 3 Q. a7 anything. 4 Q. mates. (B) 2 B. gl 3 Q. d3 + B. X Q. 4 P. mates. No. 348. Kt. e4 E. a7 Q. Xd8 Qi. mates. (A) Kt. f6 + E. g4 Mates. P. X Kt. (A) B. xE. anything. P. XB. K anything. No. 349. ■WHITE. BLACK. 1 B. b3 B. X B. (A) 2 B. c3 Kt. c4 3 Kt. d5 + P. x Kt. 4 K. g8 anything. 5 Mates. (A) 1 B. x P. 2 B. c3 Kt. c-4 3 B. X Kt. B. X P. 4 B. x P. -I- K. f5 5 P. mates. No. 350. t Q. e3 E. i2 2 Kt. b5 anything. 3 Mates. No. 351. IB. f8 IE. anywhere. 2 Kt. at c6 mates. 1 KtxE 2 Q. X Kt. mate. 1 h5 g7 or Ktx f4 2 Q. h7 mate. No. 353. Numbered 351 in error. bl K. d5 (A)(B) b4 + K c3 1 Q. 2 Kt. 3 Mates. (A) 1 K. 2 E. XB. + K 3 Q. el mate. (B) 1 Kt 2 Kt. e3 + anything. 3 Q. mates. (C) 1 B. b6 or a7 2 Kt. a3 + &c. (D) 1 B. d6 &c. 2 Kt. e3 + &c. No. 353. IE. c4 - Q. .1-! 2 Q. e4 + 3 E. mates. K. orQ. X Q. No. 354. 1 Q. f6 Q. a2 2 Q. X f5 Q. or B. + 3 P. b5 + Q. or B. X P. + 4 Q. retakes, mate. No. 355. 1 B. d6 E. f8 (A)(B) 2 Kt. d2 anything. 3 Q.orP. mates. (A) 1 E. e6 2 B. X R. + .Q. X B. 3 P. mates. 51 "WHITE. BLACK. (B) 1 P. or KtX Q. 2 E. f6 + Q. X E. 3 P. mates. (C) I Q.,E.,orKtxB. 2 Kt. retakes + anything. 3 P. mates. (D) 1 B. X d5 or E. g8 2 P. g4 + K. ±'4 3 E. f 6 mate. No. 356. 1 Q. b4 Ktx Q. (A)(B) 2 Kt. e3 dis. + K. X Kt. 3 B. mates. (A) 1 K. d5 2 Q. c4 + K. X Q. 3 Kt. mates. (B) 1 K. X P. 2 Q. f8 + K 3 Q,., B. or Kt. mates. No. 357. Q. n3 E. XP. + (A)(B) Q.XE. + K. X Kt. or KtX Q. E. or Kt. mates. (A) 1 Kt. e6 + K. X Kt. Q.xKt + Kt. mates. (B) 1 Q. f3 or E. e3 or P. X Kt. Q. orE.xKt. d5orP.xP. X Kt. or e5 Kt. e4 + Q.orE. mates. (0) 1 Q. Kt. b5 + K K Q. orB. mates. (D) 1 K. X Kt. Q. c3 + K. d6 Kt. mates. No. 358. E. h2 Q. c2 (A) E. X Q. E. X P. (a) E. c5 E. X Q. E.xKt+ P. X E. Kt. c5 mate. If 3 R. X B. Q. g2 + Q.orKt. mates. If 3 P. d2 Q. bl + Kt. gov. Q. hi mate. (a) 2 Kt. f3 (b) KtxKt. P. d5 Kt. c5 + K. f5 Kt. h4 mate. If 3 P. d2 KtxP. + Q. mates. AVHITB. Q. Kt. If 3 f7 + d2 mate. (b)2 E. c5 Q. d5 + B. xB. mate. (o)2 Q,. e6 Kt. c5 + Q. X Kt. mate. (d)2 E. c5 B. d5 + Q. or P. mates (A)l KtxKt B. h7 + Kt. f6 + Kt. b.4 mate. BLACK. K. f5 K. e4 R a5 (e) Kt. f3 B. X Q. f8 g6 (d) Kt X Kt. P. X Kt. e5 g6 KtX h4 anything. Kt. Q. P. K. f3 cl,e2 or c3 g6 15 Several other variations on Black's second move. No. 359. 1 B. b5 1 Q. X B. or Kt. d2 2 Kt. f2 mate. 1 Kt. P. 2 KtxP. mate. No. 360. c3 or d4 e5 1 Kt. g3 K. 2 Q. d6 mate. IK. e3 2 Q. b6 mate. 1 K. c4 2 Q.. d3 mate 1 K. d5 or B or P 2 Q. e4 mate. No. 361. 1 E. e8 + E. X E. 2 E. a6 + E. X E. (A) 3 Q. XE. + ! md draws by per- petual check. (A) 2 K. d5 3 B. d6dis. + K. e4 4 Q. e2 -+- K. f5 5 Q. h5 + K. e6 6 B. e5 dis. + &e. as before. No . 362. 1 E. g6 + K. h3 2 E. gl P. X E. (aQ.)+ 3 K.xQ. K. g3 4 K. fl K. f3 5 P. e8 (aQ .) winni as:. No. 363. WHITE. BLACK. 1 E. el P. queens. (A) 2 Kt. d6 + P. X Kt. 3 B. XP. + K. XB. 4 Kt. d4 mate. (A) Black has other moves, but he cannot prevent the mate in four. No. 364. 1 K. gl P. x Kt. 2 B. d3 B. x E. (A) 3 Q. X h8 anything. 4 Q. mates. (A) 2 E. X B. 3 Q. c3 &c. No. 365. 1 Q.xKt. p. x a 2 B. h7 anything. 3 Mates. No. 366. 1 Q. al Kt. c6+(A)(B) 2 E.xKt. P. e5 (a) 3 Q. c3 K. e6 or anything. 4 or Q.XP.) . Q. ci \^^ (a) 2 If K. X E. 3 Q. a5 &c. 2 If P. X E. 3 Q. a4 &e. (A) 1 Kt X E. 2 Q. c3 3 Q. orKt. mates. (B) 1 P. e5 2 R. c5 + P. X E. 3 Q.XP. &c. No. 367. 1 E. d2 E. x R. 2 KtXf4 E. d5 (A) 3 Q. dl anything. 4 Q. or P. mates. (A) 2 Kt. e6 3 Q.xKt. + 4 P. mates. No. 368. 1 E. el K (A) 2 Kt. d5 anything. 3 Mates. (A) 1 B. + 2 KtxB. + Kt X Kt. or K. c6 3 Q. c5 mate. No. 369. 1 Q. cl 1 B. b5 K. e6 (A)(B) 2 b7 co 2 Q. f5 + K, x Q. or 3 P. e3 K j 4 Kt. mates WHITE. BLACK. 3 B.or Q. mates. (A) 1 K. d6 2 Q. f7 &c. (B) 1 K d8 2 Q. xh7 &c. (C) 1 R. a6 or E. fl 2 Q. XP. &c. No. 370. 1 E. xKt P. x R. (A) 2 Kt. e5 P. x B. 3 Q. d2 anything. 4 Q. mates. (A) 1 Q. al + 2 E. X Q. E. a5 3 E. X R. anything. 4 E. a8 mate. No. 371. 1 Q. X B. Q. x E. (A) 2 P. f 4 + K. e6 3 B. b7 anything. 4 B.or Q. mates. (A) 1 Kt X Q. 2 E.xKt + K. e6 (B) 3 K. f8 anything. 4 Kt. g7 mate. (B) 2 K. e4 3 E. h4 + K. f3 4 E. d2 dis. mate. No. 372. 1 Q. f4 E. x Q. (A) 2 Kt. e3 + K. e4, e5, or e6 3 E. e7 mate. (A) E. X Kt. at d8 or moves. 2 Q. f5 + E. cov. 3 Q. X E. mate. If 1 any other move. 2 Q. d6 mate. No. 373. 1 B. e5 Q. f8 (A) 2 Kt. e3 + K. x B. 3 P. mates. (A) 1 P. c5 2 Q. c4 + K. X Q. (B) (0) 3 Kt. e3mate. (B) 2 K. x B. 3 Q. X c5 mate. (0) 2 K. c6 3 Q. x E. mate. No. 374. E. x Q. P. f3 anything. No. 375. 3 Q. WHITE. E. g2 Q.XP.+ P. mates. If 2 h3 mate. BLACK. E. g5 K. X Q. K. x Kt. 376. KtxE. (A) Ktx Q. No. 1 Q. c8 2 Q. h3 + 3 P. mates. (A) 1 E. moves. 2 E. h3 + Kt. X E. 3 Q. X Kt. mate. No 1 B. f8 2 Kt. c8 3 Kt. mates. (A) 1 E. x B 2 Kt. g4 3 Kt. mates. 377. B. x B. (A) anything. anything. No. Q, c2 E.X d8 P. e6 P. e7 Mates. 378. P. x Q. Ktx R. P. x.Kt. anything. No. 379. B. E. g4 + e8 + E.Xg6 Kt. B. fo f3 + K. e4 B. x B. B. x E.* B. x Kt.* K. x B. Stalemate. •If Black make any other move he will lose the game. No. 380. E. h6 Kt. e2 + Q. X Kt+ B. mates. (A) Kt. e6 + E, h5 Mates. Q. X E. (A) K K Q f6 Kt X Kt. anything. No. 381. Q. Q. Kt, 2 Q. c8 fo b5 mate. (A) I fo + K. d4 (A) anything. Q. X B. K. X Q. or K 3 Kt. mates. No. 382. 1 Kt. c4 dis. + B. cov. 2 Q. a6 Kt. f7 Kt. e5 + Q. XE. + K. al Kt x Kt. Kt x Q. + P. b2 mate. No. 383. 1 E. c7 Q. c3 + (A)(B) 2 E. xQ. K. or KtxE. or anything. 3 Q. b2) Q.xP. > mate accordingly, or Q. e5 ) (A) 1 Q. X Kt. 2 E. c4 + P. X E. 3 Q. e5 mate. (B) 1 K. e3 2 E. c3 + Kt X E. K. X Kt. or K AH 3 Q. Q. or Q. g3 )■ mate accordingly. e5j No. 384. B. Q. 3 bl 4 E. E. Q. 8 Ktx P. g' a8 + b2 a2 + a4 + d8 e7 + B. b8 B. cov. Kt. f2 K. xE, K. c5 B. b8 B. cov. B. X Q. mate. No. 385 1 Q. el P. x Kt. (A)(B) 2 Q. XP. K. f5 3 Q,. g5 mate. (A) 1 K. X Kt. 2 Kt. it P 3 CJ. c4 mate. (B) 1 K. d6 2 Q. cG + K. e5 3 Q. xP. mate. (C) 1 K. fo 2 Q. f4 + K. g6 3 Q. f7 mate. No. 386. 1 K. gl P. g2 (A)(B) 2 Kt. c6 K. X either Kt. 3 Q. c2 or c8 mate. (A) 1 K. b4 2 a bl + K. x P. orK 3 Q. b3) , or a b6 | mate - (B) 1 K. d4 2 Q. f6 + K 3 Q. mates. No. 387. WHITE. BLACK. "WHITE. BLACK. No. 391. 1 B. a4 + K. d5 1 K. f4 K. d4 (A) 2 E. hi anything. 2 B. fl K. d5 3 E. dl anything. 3 B. c4 + K. d4 4 B.orB. mates. 4 B. e5 mate. No. 388. (A) IP. e5 + 2 B. XP. P. h5 1 E. d8 + ' K. e6 3 K. f5 P. h4 2 d8 X P. + K. e5 4 B. g2 mate. 3 E. h4 dis. + E. cov. 4 B. d3 E. a3 or No. 392. 5 E. 6 Kt. 5 E. anywhere, or B. f7 (A) (B) h5 + B. X E. f3 + B. X Kt. mate. (A) 4 B. g6 e4 + B. x E. 1 2 3 or E. c3 K. f3 (A) P. queens -4- K. e5 or K. xB. a x g 7 l mate - 6 Kt. f3 + B. X Kt. mate. (B) 4 B. d7 2 3 or (A) 1 any other move. Kt. f3 anything. B. f6 -> P. f8 (a Kt.) Vmate. 5 Kt. 6 Kt. g4 + B. X Kt. f3 + B. x Kt. mate. (C) 4 Kt x Kt. or or P- g3 J 5 Q. Ktx e7 g5 + Kt. cov. No. 393. 6 Q. g3 + Ktx Q. mate. 1 Kt. e2 P. x Kt. (D) 4 Kt x f 6 " 2 E. d7 B. x E. 5 Q. e3 + Kt. cov. 3 Kt. f7 anything. 6 Q. g3 + Kt X Q. mate. 4 B. or Kt. mates. No. 389. No. 394. 1 b3 a5 + K. a8 2 Kt b6 + Kt X Kt. 1 e8 X B. P. x E. 3 P. h8 queens + Kt. c8 2 Kt. g5 P. x Kt. 4 Q. xKt + Q. x Q. 3 B. e8 B. x B. (A) 5 K. e3 dis. + K 4 Q. h7 B. X E. * 6 K. e4 dis. + K 5 Q. a7 mate. 7 K. d4 dis. + K * 4 K. x E. 8 K. d5 dis. + K 5 Q. e7 mate. 9 K. c5 dis. + K (A) 3 If any other 10 K. c6 dis. + B. i-2 move, 11 B. XB. + K 4 E. + 12 K. b6 dis. 4- B. g2 5 P. mates. 13 B. XB. + Q. cov. + 14 B. xQ- mate. No. 395. No. 390. 1 Q. a7 B. x Kt. 1 B. 2 B. 3 Q. 4 Q. 5 Q. f2 dis. + K. g2 a 7 dis. + K. f3 f2 + K. e4 X P. + K. d3 e3 + K. X P. 2 3 4 5 6 Q. gl + B. fl E. a8 P. g3 E. a2 P. g2 K. c4 K. X Kt. Q. h2 K dis. mate 6 Kt 7 Q. a3 + E- X Kt. d4 + K. b3 No. 396. 8 P.xKt.(aB.)+E. c4 1 B. g8 R. x B. 9 P. b8(aE.)4-B. b7 2 Kt. b2 Q. x P. + 10 Q. al P. g6 3 Kt. d3 mate. 11 B. gl P. X E. 12 K. Xh5 P. f4 No. 397. 13 K. g4 P. f3-dis. + 14 K. Xh3 P. f2 1 E. c7 E. e8 15 Q. b2 + K. X Q. dis. 2 Q. el anything. mate. 3 Mates. No. 398. WHITE. BLACK. 1 R. f 8 E. x R. (A) 2 P. f4 R. X P. 3 Kt. g3 mate. (A) 1 P. e5 2 R. f4 + P. X R. 3 Kt. f 6 mate. No. 399. 1 Kt. g6 Q. b8 2 Q. XB. Q. orKt. X Q. 3 Kt. mates. If 2 Q. e5 3 Q.. e5 mate. No. 400. IP. c4 + K. e6 2 g6xKt+ B. X R. 3 KtX P. + R- X Kt. 4 Q. b3 anything 5 Mates. No. 401. 1 Kt. d4 1 Kt X Kt. 2 P. mates. 1 K. X Kt. 2 Q. d6 mate. 1 K. d5 3 CJ. c5 mate. 1 Kt. 2 Kt. c6 mate. No. 402. 1 Kt. c5 1 K. f6 2 Kt. d7 mate. 1 K. d6 2 0,. e6 mate. 1 K. d4 2 KtxKt. mate. 1 K. f4 2 B. c7 mate. 1 Kt x Kt. 2 Q. d5 mate. No. 403. 1 Kt. g7 1 Kt X P. + 2 B.xKt. mate. I Kt. anywhere else. 2 P. d4 mate. 1 K. e4 2 R. e6 mate. No. 404. 1 B. b6 K. e7 2 Kt. e5 Kt X Kt.(A) (B) 3 Q. f6 + K. X Q.orKt. 4 B. or Q. mates. WHITE. BLACK. 3 4 3 4 (A) 2 K. X Kt Q. f8 + K. X Kt Q. f4 mate. (B) 2 B. X B. Q. f8 + K. X Q. Kt. g6 mate. No. 405. 1 B. f4 P. X B. 2 P. d4 R. X P. 3 R. e5 + K. X R. 4 R. e7 mate. No. 406. 1 Kt. b6 P. f3 2 B. cl Kt X B. 3 Q. XP. P. X Q. 4 Kt. g6 mate. No. 407. 1 Q. b5 Q. a5 2 hi f2 P. X Kt. 3 Q. e5 + Q. X Q. 4 B. mates. No. 408. 1 R. h7 Q. f4 2 Q. cl anything. 3 Mates. No. 409. 1 B. c8 Q. X P, (A) 2 Kt. iS Kt X Kt. * 3 Q.. c3 anything. 4 Q., B., or P. mates. *2Q. c3 3 Q. d4 + anything. 4 Kt. mates. * 2 Q. X c4 K. X Kt. or B. X Kt. 3 P. f 3 + anything. 4 Kt. or P. mates. (A) 1 Kt X f2 2 Q. e5 + B. X Q. 3 B. X P. + K. e3 4 Kt. c2 mate. No. 410. 1 Ktxb5 Q. d7 + (A) 2 Kt d6 Q. X Kt. + 3 K. e4 dis. + Q. cov. + 4 B. X Q. mate. (A) 1 Kt. f2 + 2 K. c3 Kt. e4 + 3 Q. X Kt. B. X Kt. + 4 Kt. cov. mate. (B) 1 Kt. b2 + 2 K. c3 ' Kt. dl + WHITE. BLACK. 3 K. d2 Q. d7 + 4 Kt. cov. mate. No. 411. 1 Kt. e6 P + 2 K. e4 P. g 3 3 E.xP P. g5 4 B. c4 P. g4 5 K. d3 anything. 6 Mates. No. 412. 1 K. c3 R. d7 (A)(B) 2 Q. X Kt. B. x Q. 3 B. d6 anything. 4 Mates. (A) 1 Kt X B. 2 Q. h8 K. xE. 3 Kt. f6 + anything. 4 Q. mates. fB) 1 Kt X P. 2 KtxB.dis. + Kt xB. 3 KtxE. anything. 4 Kt. mates. No. 413. 1 E. h3 K. f5 (A) 2 Q. g4 + K. X Q. * 3 B. e6 mate. * K. e4 3 E. e4 mate. (A) 1 B 2 Q. e2 + &c. No. 414. 1 Q. g3 + P. X Q. (A) 2 Kt. d6 dis. + P. X E. 3 B. g2 anything. 4 Mates. (A) 1 K 2 Q.X P. + K 3 R. x P + Q. X E. 4 Q. mates. No. 415. 1 Q. f8 Kt. a4 (A) 2 Q. f4 Q. X Q. * 3 Kt. d6 + Q. X Kt. 4 Kt. mates. * 2 Kt. c3 + 3 JTt X Kt. anything. 4 B. mates. (A) 1 Kt. d3 2 P. X Kt. + K. X P. 3 Q. fl + K 4 Q. mates. No. 416. 1 KtxP. 1 B. X P. + 2 Kt. f6 mate. ■WHITE. BLACK. 1 B. X Kt. 2 B. f7 mate. 1 Kt X Kt. 2 Q. a2 mate. 1 K. e6 2 Kt. c5 mate. No. 417. 1 B. c2 B. X B. 2 E. e8 E. X E. 3 P. d4 anything. 4 Kt. mates. No. 418. 1 Q. e8 E. f4 2 Q. e7 anything. 3 Q. or Kt. mates. No. 419. 1 Kt. d3 E. b5 2 Q. e7 + E. c5 3 Q. X E. + E. X Q. 4 B. mates. No. 420. 1 B. b6 1 P. X B, 2 Q. d3 mate. 1 P. X B. or P. c6 or c5 2 Kt. d6 mate. 1 e3 moves. 2 Q. d5 mate. 1 a3 moves. 2 B. c2 mate. 1 B. e3 2 E. X B. mate. 1 P. h.6 or h5 2 Q. h7 mate. I E. f4 2 Kt. g5 mate. 1 R. g4 2 Q. X E. mate. 1 E. anywhere else. 2 Q. g4 mate. No. 421. 1 E. el 1 P. e2 2 Kt. c6 + anything. 3 Q. mates. 1 K. d6 2 Ktxb5 + 3 Q. mates. 1 K. X B. 2 Q. f5 + K. X Kt. 3 E. mates. 1 P. f4 2 Q. e7 + 3 Kt. or E. mates. WHITE. BLACK. 1 B. x E. 3 Q. e6 + 2 Q. or Kt. mates. No. 422. 1 B. e7 Kt. d6 (A)(B) 2 Q. c4 Kt. X Q. # 3 E. f5 + K. x Kt 4 B. mates. * 2 B. X E. 3 Q. c5 + K 4 Kt. e2 mate. (A) 1 E. x B. 2 Q. g5 + K. d6 * 3 E. f6 + 3 Kt. mates. * 2 K. X Kt. 3 P. c3 + K 4 Q. e2 mate. (B) 1 B. X E. 2 Q. b5 + K. f4 *• 3 Kt. e2 + K 4 B. mates. * 2 P. f5 3 Q. x e4 + K 4 Kt. mates. (C) 1 K. x Kt. 2 Q. b5 E. x B. 3 P. c3 + K 4 Q. mates'. No. 423. 1 G. x c6 B. X Q. (A) 2 B.XP. + K. X B. 3 Kt. mates. (A) 1 K. X E. 2 Q.XP. + K 3 Q. e3 mate. No. 484. 1 Kt. d2 dis. + K. g3 2 K. Ill P. X P. 3 Q. g8 E. g5 4 B. d6 + either E. cov. 5 B.xE. + E. X B. 6 Kt. f5 + E. X Kt. 7 Kt. fl + E. X Kt.mate. No. 425. B. xE. (A)(B) Kt X Q. 1 Q. al 2 Q. a8 + 3 B. mates. (A) 1 K. X E. 2 B. c4 + K 3 Q.xB. mate. (B) 1 R. h8 2 Q,. e5 anything 3 Mates. (C) IB. cl WHITE. BLACK. 2 Q. d4 anything. 3 Mates. (D) 1 B. g2 2 E. e3 + anything. 3 Mates. (E) 1 B. h3 2 Q. hi + anything. 3 Q. or B. mates. No. 426. 1 B. f6 E. x B. (A) 2 h5 g7 E. x Kt. * 3 Q. X P. + K — 4 Kt. mates. * 2 P. e3 3 Q. c5 + K — 4 Kt. mates. (A) 1 P. e3 2 e6 X P. + K. e4 3 Q. c6 + K — 4 Q. mates. No. 427. 1 Kt. d7 + K. d4 2 Q. hi Black has choice of five moves, but in any case 3 Q. or E. mates. No. 428. 1 E. f4 B. X B. (A) 2 Q. h8 E. X B. * 3 Q,. a8 anything. 4 Q. mates. * 2 K. X E. 3 Ktx P. + K 4 Q. mates. (A) 1 E. x B. 2 Kt. f3 + P. x Kt. 3 0. h8 + K 4 Q. mates. No. 429. 1 B. h3 K. d5 (A) 2 Q. c5 + E. x Q. * 3 Kt. e7 + K. c4 4 Kt. d6 mate. If 3 K. X Kt. 4 B. f5 mate. * 2 K. X Kt. 3 Kt. e7 B. c4 or E. x Q. 4 B. f5 mate. If 3 Kt. g3 or g7 4 CA. d5 mate. (A) i a x Kt. or B. x Kt. or B. fl 2 Q g8 + K. d7 3 Q. f7 + K. c8 4 Kt. d6 dis. + No. 430. WHITE. BLACK. I lit. f5 1 K. X Kt. 2 E. X 1'. mate. 1 R. e8 2 B. X Kt. mate. 1 E. X Kt. ]'. X Kt. Kt orB 2 Q. c3 mate. No. 431. 1 Kt. d3 E. c4 (A)(B) 2 Q. d4 + K. X Q. * 3 E. X Kt. mate. 41 2 E. x Q. 3 E. c5 mate. ■ * 2 K. X E. 3 Kt. e5 mate. (A) 1 B. X Kt. 2 Q. d7 anything. 3 Q. X Kt. mate. (B) 1 B. X E. 2 Q. e5 + K. c4 3 Kt. b2 mate. (C) 1 Kt. f7 2 Q. X Kt + anything. 3 Q. mates. (D) 1 K. x E. 2 Q. c7 + 3 Q. mates. (E) 1 Q. X f2 2 KtX f4 + K. x E. 3 Q. c7 mate. No. 433. 1 P. go + 1 K. x P. o Q. X f7 2 B. X Q. * 3 E. eo + 3 K. f6 4 Kt. d7 mate. If 3 K. f4 4 Kt. d3 mate. * 2 E. X Kt. 3 Q. e7 + 3 K. X B. 4 Q. X P. mate. If 3 K. f5 4 B. g4 mate. * 2 B. X B. 3 E. e5 + K. g4 4 Q. f5 mate. * 2 K. X B. 3 Q. f3 + K. g5 4 Kt. e6 mate. * 2 B. f5 3 E. g4 + B. X E. 4 Kt. e4 mate. * 2 Q. X B. 3 Kt. e6*+ K 4 Q. x P. mate. WHITE BLACK. * 2 Kt X Kt. or Kt. d8 3 E. ej + B. cov. 4 Q. x B. mate. * 2 P. queens, or Kt. d6 or Kt. c6 3 Kt. e6 + K. x B. 4 Q. f3 mate. No. 433. 1 Q. a7 K. d4 (A)(B) 2 E. e2 K. e5 * 3 Q. g7 mate. *2 KtX Kt. 3 P. xKt. dis. mate. * 2 anything else. 3 Kt. b5 mate. (A) 1 Kt x Kt. 2 Q. g7 + K. f5 3 E.XE. mate. (B) 1 Kt X P. 2 B. x E. + K. f6 or d4 3 Q. orKt. mates. (C) 1 Kt x B. 2 Q. g7 + E. f6 3 Q. X E. mate. (D) 1 K. fC 2 B. X Kt. moves. 3 Q.orKt. mates. K., P., or E. No. 434. 1 B. a4 Kt X E. (A) 2 Kt. e5 K. X Kt. 3 B. g7 mate. If 2 K. X P. 3 Kt. b3 mate. If 2 K. c3 3 Kt. f3 dis. mate. If 2 B. X Kt. 3 Q. X Kt. mate. (A) 1 B.xE. 2 Kt. b3 + K. e4 3 Q. hi mate. No. 435. 1 Q.xP. E. g6 (A) 2 Kt. c8 anything. 3 Q. , R., or Kt. mates. (A) 1 B. X E. 2 B. e4 + K. c5 3 Kt. c8 dis. mate. (B) 1 Kt. f5 2 Q. b7 + K. x Kt. 3 B. e5 mate. (0) 1 Kt. c5 2 B. e4 4 Kt X B. 3 Q. e5 mate. .52 No. 436. WHITE. BLACK 1 Q. o4 B. X Q. (A) 2 K. d7 + K 3 P. mates. (A) 1 K. x E. 2 Q. c5 + 3 Q. mates. (B) 1 B. X E. 2 Q. X f i + K. c-5 3 Q,. b4 mate. No. 437. 1 Q. g4 1 B. x Q. orB 2 E. d4 mate. 1 K. xKt. 2 Q. f3 mate. 1 K. x E. 2 Kt. e3 mate. 1 Kt 2 Q. x B. mate. 1 K- 2 E. d.4 mate. 1 P. f3 2 Kt. b4 mate. No. 438. 1 Q. 08 + K. h7 2 R. c7 + K. h6 3 E. g6 + K. h5 4 E. a6 dis. + K. g5 5 Kt. f3 + K. f5 6 B. c2 + E. cov. 7 Q. f7 + Q. cov. 8 K. x P. Q. X Q. (forced) 9 E. c5 + Q. cov. 10 B. bl Q. e5 11 E. d5 Q. X E. mate. No. 439. 1 Kt. b3 K. X Kt. (A)(B) 2 Kt. a5 dis. + K 3 Kt. or B. mates. (A) 1 K. e5 2 B. g3 + K. f6 3 Kt. d5 mate. (B) 1 E. X E. 2 Kt. d5 + K 3 Kt. or B. mates. No. 440. 1 Q. g8 Q. x Q. (A) 2 KtxP. + Kt. or P. X Kt. 3 Kt. mates. (A) 1 B. f7 2 Q. x Kt. anything. 3 Q. a2 mate. No. 441. WHITE. BLACK. 1 E. f4 B. orP.xE.(A) 2 Q. f5 + K. X Q. 3 Kt. mates. (A) IK. d7 2 Q. c8 + K. X Q. or K. X Kt. 3 Kt. mates. No. 442. IP. d3 + K. d4 2 Kt. f5 + E. X Kt. 3 Ktxc7 K.xE. (A)(B) 4 B. e3 mate. (A) 3 E. X e5 4 E. c4 mate. (B) 3 E. f6 or E. e8 orB. X P. 4 R. d5 mate. (C) 3 P. f2 or E. X b5 4 Kt. e6 mate. No. 443. 1 f5 x B. + K. e8 2 Q. X b4 B. X Q. 3 B. b5 mate. If 2 E. X Q. 3 K. X P. dis. + If 2 Q. X Q. orQ. X B. 3 P. c8 queens + If 2 Q. X P. + 3 K.X Q. dis. mate. No. 444. 1 B. h4 + E. X B. 2 Q. a7 K. X R. 3 Kt. e6 + + K. has five squares open to bim, but where- ever he goes 4 Q. mates at g7 No. 445. 1 Q. f2 E. b7 (A) 2 Q. X B. Q. d4 3 Q. d6* Q. gl 4 Kt. f 2 + Q. x Kt. 5 Q. h2 mate. (A) 1 B. d7 2 Q.xB. Q. d4 3 Q.xQ. E. fl 4 Q. gl + E. X Q. 5 Kt. f2 mate. * If white here play 3 Q. X Q. he will be foiled by 3 E. b2. No. 446. WHITE. BLACK. IE. d7 + Kt X E. 2 Kt. e6 anything. 3 K.orKt. mates. No. 447. 1 E. h4 B. X E. (A) 2 Q. d6 P. c6 * 3 Q. f8 + R. x Q. 4 Kt. d6 mate. * 2 Kt X P. 3 Q.xKt+ K. d8 4 Q. c8 mate. (A) 1 B. IB 2 E. X E. anything. 3 Q.orE. mates. No. 448. IB. b3 + 2 E. a4 + 3 E. c4 + 4 Q. b5 + 5 Q. c6 + 6 Kt. ib + 7 Q. d6 + K. e8 8 Q. d7 + 9 Q. e7 + 10 E. h4 dis. + 11 Q. e8 + B. cov. 12 P. mates. No. 449. 1 Kt. g6 + P. X Kt. 2 Q. h2 + Kt. h6 3 Q. X Kt + P. X Q. 4 B. X d4 + K. h.7 5 Kt. g5 + P. X Kt. 6 E. hi + Q. X E. 7 E. X Q. mate. No. 450. 1 Q. X a7 + 2 Kt. b6 + 3 B. d6 + 4 E. c8 + 5 E. a8 + 6 Kt. d5 + 7 E. c8 + 8 E. X Q. mate. KtX Q. K P. X B. KtX E. K. c7 K. c6 Q. cov. No. 451. 1 B. X c7 + K. X B. 2 P. . b6 + K. b8 3 E. a8 + K. X E. 4 E. a4 + K. b8 5 E. a8 + K. X E. 6 Q. a I + anything 7 Q. mates. No. 453. WHITE. BLACK. 1 E. f8 + B. XE. 2 Q. g8 + K. xQ. 3 Kt. f6 + K. h8 4 E. XE. mate. No. 453. 1 E. b8 + B. cov. o Q. h5 + K 3 P. e7 + K- 4 Q. g6 + K 5 B. f6 + Q. x B. 6 d7 Xd8 + Q. cov. 7 P. d6 E- + 8 K. fl Q. xE. 9 P. c5 Q. g 8 10 P. ffi Q 11 0. h6 + K. g8 12 P. b4 mate. No. 454. 1 Q. a6 + K 2 E. b5 + K 3 Kt. c2 + K 4 E.xg5 + K 5 Q. e2 + K 6 R. g3 + Kt X B. 7 Q. fl + K 8 Q. el + K 9 K. f4 + P. x E. 10 B. fl mate. No. 455. 1 Kt. c6 + K 2 E. b6 + P.xE. 3 P. xP. + K 4 Q. a7 + K.xKt. 5 B. d7 + K 6 Q. a2 + K 7 Q. g2 + K 8 Kt. g6 mate. No. 456. 1 Kt. b5 + K 2 Q. a7 + B. x Q. 3 Kt. c7 + K 4 Ktxa6 + K 5 Kt. c7 + K 6 Ktxd5 + K 7 Kt. b6 + B. X Kt. 8 P. xB. + Kt. cov. 9 E. XKt.mate. No. 457. 1 R. g8 + E. cov. 2 Ktxd6 + P. x Kt. 3 Q. f7 + K, x Q. 4 E. g7 + K. e8 5 B. d7 + Kt X B. 6 P. x Kt. mate. Wo. 468. WHITE. BLACK. 1 B. g8 + K 2 B. f7 + K 3 B. S 6 + E. x B. 4 P. XE. + K. X P. 5 Q Xh6 + P. xQ. 6 E. g8 + K 7 Kt: mates. No. 459. 1 E. xP. + Kt x E. 2 Q. x Kt + Kt X Q. 3 Kt. c6 + K. b7 4 E. b8 + K. x Kt. 5 E. X b6 mate No. 460. 1 KtxE. + .K. b7 2 K. 13 + 3 B. aG + 4 B. x B. + SB. d5 + 6 B Xe6 + 7 B. xE. + 8 E. b8 + 9 E. e8 + 10 E. e6 + 11 E. xg6 + 12 E. g4 + 13 E. xB. + 14 E. g4 + 15 E, gl + 1G Kt. mates. No. 461. 1 Kt. go + K 2 E. h8 4- B. xE. 3 K. g8 Kt. d6 4 K.xB. 5 Kt. mates. No. 462. 1 Kt. c3 P. a2 2 E. al K. a3 3 Kt. c2 4- K. b3 4 E. X P. B. f6 5 E. a3 + K. b2 6 Kt. dl 4- K. bl 7 B. x c4 P. XB. 8 Kt. c3 4- K. b2 9 E. a2 + K. b3 10 K. ct K. X Kt. or anything. HE. a3 mate No. 463. 1 B. c3 K. d3 (A)(B) 2 Kt, e5 K. x B. 3 Q. b2 mate. WHITE. BLACK. (A) 1 B. X P- 2 Kt. e5 4- E. f4 orB 3 Q. f3 mate. (B) 1 K. f4 2 Kt. e5 ' K. f5 3 Ct. g4 mate. No. 464. 1 Q. d8 Q. f3+(A)(B) 2 B. g4 + 3 Q. or Kt. mates. (A) 1E.XP.+ 2 B. f5 + 3 Q. or Kt. mates. (B) 1 E. d7 2 B. b7 + 3 Kt. mates. (C) 1 B. X Kt. 2 B.X a6 + 3 Q. X E. mate. No. 465. I 2 3 4 Q. c5 B. e6 -f Q. X E. + Q. mates. 1 2 3 4 N< Q. d6 Kt do Q. X Kt. Q.orKt. m 3 4 2 3 4 2 3 4 2 3 4 Kt. g6 KtX E. 466. Kt. e6 B. x Kt. anything. No. 467. Q. h.4 Kt. c6 E. X P. 4- E. b8 mate. ^ x> (A ) l Q.xB. + Kt. a6 + e8 mate. (B)l c6 d8 + B. X B. mate. (0)1 KtXaG E.xB. KtX c5 mate. E. x Q. (A) (B) P. a6 B. xE. Q. Kt. Q. B. X K. K. E. B. x E. X B. b8 cS dG Kt. Q. P. a6 K. X Q. E. X B. No. 468. Kt. Q. Q. e7 + d5 + f7 Q. X h7 + B. f8 mate. B. x Kt. K. hS E. g8 k. x a No. 469. No. 478. WHITE. BLACK. WHITE, BLACK. 1 2 3 4 5 Kt. h5 + E. d5 + P. X P. + E. gl E. dl mate. P. X Kt. K. e6 K. xE. anything. 1 2 3 Q. f4 + K. x Q. Kt. do + K. x P. B. g6 mate. No. 477. No. 470. 1 Q.x P. + P. x Q. 2 1>. c7 Kt. e7 or Q. x h3 3 B. b7 + K. X B. 4 P.xKt(aQ)+K c6 5 Q. c7 mate. No. 471. 1 Q X Kt + E. X Q. 2 E. f7 + K 3 Kt. c6 + K. X Kt. 4 P.XP. + K. X Kt. 5 P. d8(aKt)+B. X Kt. 6 P. b8(aKt) mate. No. 472. 1 Kt. b5 + Kt X R. + 2 K. a6 Kt X b6 3 P. xKt. E. b8 4 P. b7 + R. X P. 5 P. queens. + E. eov. 6 Q. c6 + E. eov. 7 Q. yE. mate No. 473. I K. e5 + K. b4 2 P. a3 + K. a5 3 P. b4 + K. a6 4 Kt. e6 B. d8 5 Kt. d7 Kt X P. 6 P. XP. R. c8 7 P. f5 E. a8 8 p. m E. c8 9 P. f7 E. a8 10 P. f8(aQ.; E. c8 11 Q. c5 KtX Q. 12 KtxKt + Kt X Kt. 14 KtxKt mate No. 474. 1 Q. d8 + Kt x Q. 2 Kt. d6 + K. f8 3 P. e7 + K. g8 4 Kt. f7 + Kt X Kt. mate No. 475. 1 P. f4 + K - 2 Q. d5 + K. XQ. 3 Kt. g5 mate. 1 Q. & + K. x eo 2 Q. f6 + K. X Q. 3 Kt. e4 mate. No. 478. Q. g3 + Kt. e3 + Q. X P. + B. b6 mate. K K Kt x Q. No. 479. 1 Kt. b6 + B. x Kt. 2 Q. b8 + E. X Q. 3 E. a7 + B. X E. 4 Kt. c7 mate. No. 480. d7 + 1 B. 2 Kt, c4 + 3 Kt. f6 4- K. c5 4 P. b4 mate. K. X E. E. x Q. No. 4S1. 1 Q. c6 + K 2 B. a5 + K. x B. 3 P. c3 anything. 4 P. b4 mate. No. 482. 1 E. d5 + Kt X E. 2 Q. c6 + K. x Kt. 3 Kt. f3 + K. x P. 4 Q. c2 + K. X Kt. 5 Q. g2 mate. No. 483. 1 Kt. a7 + K. d7 2 P. e6 + K. d6 3 Kt. b5 + K. e5 4 K. g5 Kt. d5 5 B. c7 + Kt. x B. 6 P. f4 mate. No. 484. 1 E. al + K 2 Kt. d5 + K 3 E. cl + K. X Kt 4 E. e5 + P. xE. 5 R. c5 + E. X E. 6 Q. d7 mate. No. 485 WHITE. BLACK. 1 Kt. C6 + K 2 B. e2 + K. X Kt. 3 B. f 3 + K 4 Q. f5 + K. x Q. 5 Kt. d4 + K. X P. 6 B.;X e6 mate. No. 486. 1 Kt. bo + K 2 Kt. d6 + K. X B. 3 Q. a7 + K. X Kt. 4 b4 d4 + P. x E. 5 Q. e7 + K 6 B. e6 + K. X E. 7 B. x g8 + K 8 Q. e6 + K 9 Kt. f3 + K 10 P. mates. No. 487. 1 B. el Q. x Q. (A)(B) 2 P. d4 c4 X P. 3 Kt. g3 + 4 Kt. or E. mates. (A) 1 E. X Q. 2 P. d3 + E.,Kt.,orP.xP. 3 Kt. g3 + 4 Kt. or E. mates. (B) 1 G. g7 2 Q. d4 + Q. X Q. 3 Kt. g3 + 4 E. mates. No. 488. 1 R. b5 + R. X E. 2 Kt. c6 + K. fo 3 E. e3 P. e5 4 E. X P. + R. or Kt x E. 5 Kt. d4 mate. No. 489. 1 2 3 4 5 Kt. B. K. K. • E. e3 P. X Kt. f 1 P. X Kt. c2 P. e7 1 Q. c2 2 Q. c3 + 3 Q. a5 + 4 Q. c7 mate. (A) 1 P. f3 2 Q. h2 + 3 Q,. h5 + 4 Q. h8 mate. ■WHITE. BLACK. (B) 1 P. e3 2 Kt. d3 + K. e4 3 Kt. 1)4 dis. + K. f3 4 Q. g2 mate. If 3 K. eo 4 Kt. c6 mate. No. 699. 1 B. K x E. or B. d4 2 Q. e7 mate. 1 B. b2 or c3 2 Q. x B. mate. 1 Q. B 2 Kt. d7 mate. 1 Kt X E. 2 Kt. g6 mate. 1 Kt. elsewhere. 2 Q. d6 mate. 1 E. U 2 E. e6 mate. 1 E. elsewhere. 2 Kt. g4 mate. 1 g3 moves. 2 E, f5 mate. No. 700. 1 Q. f7 1 B. x Q. (A) 2 P. c4 3 Mates. (A) 1 K. X Kt. 2 P. a4 + K. X P. or K. a6 3 Q. mates. No. 701. 1 Q. b8 Kt x B. 2 Q.xb2 3 Mates. No.. 702. 1 B. a4 K. X Kt. 2 B. b3 mate. 1 P. d6 2 bo c7 mate. 1 P. f6 2 d5 c7 mate. IP. f5 2 Q. g8 mate. 1 P. e4 2 Q. X e4 mate. No. 703. 1 B. dl K— 2 B. c2 3 Q. or B. mates. No. 704. 1 Q. d6 B. 2 E. g3 + K. e5 + 14 WHITE BLACK. 3 P. b.6 Q. p 4 E. a4 anything. 5 Mates. No. 705. 1 E. g2 B. X P. 2 Q. gl anything. 3 Mates. No. 706. To mate. 1 E. X b3 + P. X E. 2 Q,. X B. mate. To sui-mate. 1 E. Xa2 + P. xE. 2 Q. c3 + B. X Q. mate To stalemate. 1 Q. X B. Kt 2 Kt X Kt. stalemate. To sui-stalemate. 1 Kt. c2 + P. X Kt. 2 Q. c3 + B. X Q. sui-stalemate. No. 707. 1 Q. g2 K. f6 (A) 2 Q. hi K. e5 * 3 Q. al mate. * If 2 P. e5 3 Q. c6 mate. If 2 Kt. f4 3 Q. h8 mate. If 2 Kt. e4 3 Kt xP. mate. (A) 1 h3 (B) 2 KtxP. + 3 Q. X Kt. mate. (B) 1 f2 2 KtxP. + 3 Q. e4 mate. (C) 1 K. d4 2 Ktxb4 Kt. f4 * 3 Q. X Kt. mate. * If 2 Kt. dl 3 Q,. e4 mate. • If 2 P. e5 3 Q. d5 mate. If 2 Kt X Kt. 3 Q. e4 mate. No. 708. 1 Q. a8 K. g4 2 Q. a4 anything. 3 Q. d7 01 Q. dl mate accordingly. No. 709. WHITE. BLACK. 1 Q. h5 K. do 2 Q. dl K. xE. 3 Q. b3 mate. If 2 E. x Q. 3 Kt X e3 mate. No. 710. 1 R. e7 E. x E. 2 Kt. f7 E. x Kt. 3 P. e4 -f P. x P. en pass. 4 E. d4 mate. No. 711. 1 Q. h2 K. x Kt. 2 Kt. d3 dis. mate. 1 K. x E. 2 Kt. e6 mate. 1 Kt X Kt. 2 B. X B. mate. No. 712. 1 Kt. d4 B. x Kt. (A) 2 E. e5 K. x E. 3 Q. d6 mate. (A) Kt X Kt, 2 Q. d6 + K. f3 3 Q. xE. mate. No. 713. 1 Q.xB. P. x Q. 2 B. b5 + K. X B. 3 Kt. b4 Q. x Kt. or B. x E. 4 P. a4 + K. x Kt. 5 B. d2 mate. No. 714. 1 E.X eo B. e7 (A) 2 R. x f3 P. X E. 3 KtxB. mate. (A) 1 B. d5 ,2 Q. b6 + K 3 Kt or P. mates. (B) 1 B. X E. 2 Q. b6 + K 3 P. b3 mate. No. 715. 1 B. bl 2 Q.xB. 3 Kt. g3 + 4 Mates. (A) 2 E. X Q Q. X B. Kt. d7 (A)(B) anything. "WHITE, BLACK. 3 R. g5 + B. X E. 4 Kt. mates. (B) 2 Q. e4 3 Q.XP/+ R. g4 4 Q. x E. mate. No. 716. 1 Q. g5 K. x P. 2 Q. ft mate. 1 P. X P. 2 B. g7 mate. 1 K. c5 2 Q. bb mate. 1 K. e3 1 Q. d2 mate. 1 Kt. moves. 2 Q. X P. mate. No. 717. 1 Q. d6 P. d4 (A) 2 B. f4 anything. 3 Mates. (A) 1 Kt. b7 2 Q. b4 + P. d4 3 Q. xKt. mate. (B) 1 Kt. c6 2 Q. xKt. + &c. (C) 1 Kt. o4 2 Q. a6 &c. No. 718. 1 B. h8 1 K.,E.,B., or h3 moves. 2 Q. al 3 Q. mates. 1 P. X B. 2 Q. e8 3 Q. mates. 1 P. b3 2 Q. d6 3 Q,. mates. No. 719. 1 B. c3 B. X P. 2 Q. a6 + B. X Q. or K 3 P. or Kt. mates. No. 720. 1 B. c3 B. x, B. 2 B. g2 anything except (A) 3 Kt. d3 + P. X Kt. 4 P. f4 mate. (A) 2 Kt X P. 3 Kt. d6 4 Kt. mates. No. 721. Ghuznee. No. 148. Corrected. WHITE. BLACK. 1 Q. X Kt. P. x Q. 2 R. b7 Kt. f4 + 3 K. f 1 (A) Kt x Kt. 4 R. e7 + K. d8 5 KtX P. + K. c8 6 P. c4 * Q. d8 (best) 7 P. queens. Q. x Q. 8 P. + K. c7 9 P. queens. + K. x Kt. 10 Q. d7 + K. b6 11 Q. b5 mate. * This is the move which appears to have remained nndiscovered for so many years. (A) If white play 3 Kt x Kt. which seems to be the natural move, Black will escape, thus — 3 Q. X P. 4 e5 g6 (best) Q. X Kt. 5 KtxQ. Castles! and White is foiled. It appears that in a letter from Bolton to Lewis, dated 9th September, 1843, after noting certain points of attack and defence in this problem, he continues thus — " and, if necessary, Black can afterwards Castle." No. 723. 1 K.x e4 K. x E. 2 B. e5 K. X E. 3 B. c2 mate. If 2 B. X E. 3 B. b5 mate. No. 723. 1 K. a2 P. *b4 2 Q. ' f8 + B. b8 dis. + 3 Q. f3 mate. No. 724. 1 E. b5 P. x B. (A) 2 Ktx P. K x P. 3 Kt. e4 + K. x Kt. 4 E. e5 mate. (A) 1 K. X P. 2 B. e4 + K. c4 3 E. b6 B. X Kt. 4 P. X B. mate. (B) 1 B. X Kt. 2 E. b6 + K. x P. 3 P. e4 + B. X P. 4 B. X P. mate. No. 725. 1 B. c6 K. d3 (A) 2 B. xP. + K. e4 3 Kt. e7 dis. mate. (A) 1 B. X P. 2 KtX e3 dis. + K. X Kt. 3 Q. a3 mate. (B) 1 K. f5 2 Q. e8 3 B. d7 mate. (0) 1 P. queens. 2 Kt X f4 dis. + K. X Kt. 3 Q. f8 mate. No. 726. 1 E. g8 Kt X Kt. (A) 2 Q. g6 K. c5 v;. 3 Q. X P. + K. X Q. 4 E. c8 mate. (A) 1 B. c3 2 P. e3 + ' K. e4 3 Kt. f 6 + E. X Kt. 4 Q. X P. mate. No. 727. 1 E. b3 Kt X Q. (A) 2 B. e4 + K. X B. ; 3 Kt. mates. (A) 1 Kt. d7 2 Q. X e6 + K. X Q. 3 B. mates. No. 728. 1 E. g4 P. X E. 2 E. g5 P.orB. X R. 3 Kt. f5 mate. If 2 K. X E. 3 B. cl mate. No. 729. 1 Q. f6 + K. x R. 2 Q. b2 Kt X Q. 3 Kt. b3 mate. No. 730. 1 Q. f2 P. e5 (A) 2 Q. fl P. c5 3 E. xP. + K.,B.,orP.xE. 4 Q,. h3, f8, or a6 mate. (A) 1 P. c5 2 Q. X B. P. X E. 3 B. f4 + 4 Q. mates. (B) 1 P. x E. 2 Q. e3 + 3 Q. d4 4 Q,. g7 mate. No. 731. 1 Q. d2 h7 h6 (A)(B) 2 Kt. f2 K. g3 * 3 KtX e4 + K. f4 5 Kt. 4 Kt. g3 K. g3 or K. x P. 5 Kt. fl or Kt. g4 mate. If 4 P. X e5 5 Q. f2 mate. If 4 B. g3 5 Q. d4 mate. * 2 P. X eo 3 f2 dl P. d6 4 Kt. c3 K. g3 5 Kt. fl mate. If 4 anything else. e2 mate * 2 K. X P. f2 X g4 + K. f4 Q. b4 anything. Q. d6 mate. * 2 P. g3 Kt. g4 dis. + K. i'5 KtXh6 + K. X B. i'2 g4 mate. (A) 1 P. g3 c4 dis. + K . g4 e2+ K e3 anything. g4 mates. (B) 1 B. el Q. X B. ' K. X P. Q. fl K. d4 g4 anything. c4 mate. (C) 1 K. X P. c3 + K. f4 f6 + K. g3 d6 mate. (D) 1 B. f2 Q.XB. + K. X P. K. c3 anything. Q. f5 mate. No. 732. 1 Ktx eo P. X B. 2 d4 e6 B. X B. 3 Kt. e7 + K 4 Q. b4 or b6 mate. An extremely difficult Problem ; the variations on which I leave as an exercise for the reader. Kt. Q. dl Q. Kt. Q. Q. Q. Q. Q. Q. Q. Kt. No. 733. fl K b5 + K b8 + K. X Q. a6 mate. No. 734. 1 Kt. 2 Q. 3 Kt. 3 Kt. g5 c5 + e4 mate. If 2 K d3 mate. E. f6 (A) K. X Q. ■WHITE. BLACK. (A)l E. f8 2 Q. X E. + K. x E. 3 Kt. d3 mate. (B)l K. XE. 2 Kt. d3 + K. d6 3 Q. c5 mate. (C)l B. X E. 2 Kt. f7 + K. e6 3 Q. f5 mate. (D)l P. d4 2 Kt. e4 + K. XE. 3 Q. io mate. (E)l Kt. d4 2 Kt. f7 + K. c5 3 Q. X Kt. matt t No. 735. 1 K. d2 K. hi (A) 2 Q. a8 P. gl dis. + 3 Kt. f2 mate. If 2 K. gl 3 Q. a] mate. (A)l E. hi 2 Q. b8 anything. 3 Q. mates. No. 736. 1 Kt. c5 + + K. b5 2 Q XP. + K. X Kt. 3 Q. b4 + K. d4 4 B. XB. + K. X P. 5 E. d2 + K. e4 6 Q. XE. + E. cov. 7 Q. X c6 + ; J 8 Q. Xe6 + >i 9 Q. X c6 + ? J 10 Q. xKt + } ' 11 Q. c6 -4- ') 12 P. XB. + K. XP. 13 B. XQ. K g2 14 Kt. g5 K. XE. (A) 15 B. c5 P. H3 16 B. Xh3 P. h4 17 Q. a8 P. c6 18 E. dl + E. XE. mate. (A) 14 P. h3 15 B. X P. + K. XE. 16 B. c5 P. M 17 Q. a8 P. c6 18 E. dl + R. X E. mate. xE. Solutions of the Problems on the Frontispiece and the Vignettes. Camtlla. WHITE. BLACK. 12 Kt d6 K a8 WHITE. BLACK. 13 Kt . c8 K. b8 1 Q. f4 + * K. e8 14 Kt a7 K. a 8 2 Q. a4 + K. f8 15 P. b4 K. b8 3 Q. a8 + E. d8 (A) 16 P. b5 K. a8 4 Q. xKt + B. cov. 17 P. b6 K. b8 5 Q. X B. + K. e8 18 Kt c-6 4- K. moves. 6 Q. g4 K. f8 (best) 19 P. mates. 7 Q. f 5 + K. e8 8 Q. Ii5 Q. X R. + The Propeller. 9 Q. X Q. + K. f8 10 Q. f7 mate. i a f3 4- K. a7 (best) (A), 5 B. d8 2 Kt c6 4- 4 Q. xKt + K. e8 3 Kt d8 dis. + 5 Q. h5 E. x e6 (best) 4 P. xP. 4- K. XP. 6 E. g8 + K. d7 5 Q c6 4- 7 Q. e8 + K. c8 6 Q. c5 + 8 Q. x B. + K. b7 7 Q. d5 4- 9 Q. a 8 mate 8 a d4 4- • * 1 Q. x Kt . 4- will not win 9 Q. e4 + though it looks so tempting. 10 Q. 11 Q. e3 4- f3 4- f2 4-